Thread Rating:
  • 0 Vote(s) - 0 Average
  • 1
  • 2
  • 3
  • 4
  • 5
some helpfull notes - sami2004
#1
Cram-Facts:
CARDIOLOGY:
1. Stable angina – chest discomfort, can be felt in back/arms/jaw/abdm, occurs c stress/emotion, relief c rest, dx c stress test. Tx c nitrates, bb, Cabs, heparin, aspirin, if 3 vessels or L main do CABG
2. Unstable angina – unpredictable at rest or abruptly worsening pattern of angina, prolonged duration (>20), dx c ECG (st depression/t inversion) or cath shows CAD, but negative cardiac markers, tx c nitrates, cabs, bb, heparin
3. Variant/Prinzmetal Angina – chest pain at rest, ST elevation (note the 3 causes of ST elevation are MI (inferior (LDA) is II, III AVF; lateral (circumflex a)is I, AVL, V5, V6; anterior is V1-V4), Pericarditis (diffuse, meaning every lead has it), and Variant Angina) with negative markers. Treat with Ca-channel blockers (Cabs) or nitrates.
4. Acute MI – chest discomfort, crushing pain without warning (females and diabetics get atypical chest pain, which is abdm pain, fatigue, neck pain or weakness), prolonged duration (hours), ECG may be abnormal (st elevation or depression), increased markers, tx c MONA, ACEI, heparin, bb, tPA’s if 60, subtract one). >2 risk factors: diet if >160, drugs if >190; 2 or more risk factors: diet if >130, drugs if >160, pt has CHD: diet if >100, drugs if >130.
6. Causes of high output heart failure – severe anemia, thyrotoxicosis, acute beriberi, paget’s dz, large AV fistula
7. Acute Pulmonary edema – tx – 1st upright position and O2, 2nd loops, nitrates, morphine, and 3rd intubate if severe.
8. HOCM – tx – 1st avoid dehydration, 2nd strenuous activity prohibited, 3rd BB, 4th Cab’s, 5th surgical myectomy. Best dx is history (screen family) and physical, then Echo.
9. Restrictive CM – JVD, edema and ascites, dx c echo, tx 1st diuretics/ decrease salt
10. Myocarditis – history or URI (coxsackie) then fever, dyspnea, CP, edema, tachy
11. Acute Pericarditis – positional CP, tx c NSAIDS
12. Pericardial effusion – pericardial friction rub, tx c pericardiocentesis
13. Tamponade – becks triad (JVD, muffled heart sounds, pulsus paradoxicus c hypotension), tx c pericardiocentesis
14. Constrictive pericarditis – pericardial knock, kussmaul breathing, CXR shows pericardial calcification, tx c diuretics
15. Acute RF – PECCS (polyarthrtitis, erythema marginatum, carditis, chorea, subQ nodules) in kids 5-15yo due to group A strep. Tx is Abx, bed rest, salicylates, sedatives for chorea, steroids for carditis.
16. Mitral stenosis – most associated c RHD, LA enlargement à hoarseness, dysphagia, and A.fib, diastolic rumble at LV apex, tx c diuretics, coumadin for a.fib, endocarditis prophylaxis, balloon vulvoplasty
17. Mitral regurge – a/w marfans, RHD, myxomatous change, high-pitched holosystolic murmur at left sternal border, tx c diuretics, dilators, endocarditis prophylaxis, mitral valve respacement/repair
18. Aortic regurge – congenital, marfans, trauma, aortitis, high-pitched decrescendo diastolic murmur at left sternal border and/or apex and wide pressure, tx c valve diuretics, dilators, endocarditis prophylaxis, valve replacement (last)

19. Aortic stenosis – calcific in elderly, bucuspid in congenital, angina, dyspnea, syncope, mid-late systolic murmur at base radiating to carotids, tx c replacement (1st step)

20. Endocarditis – if dental procedure give amoxicillin (clindamycin if allergic), if GI/GU procedure give amoxicillin c gentamycin (vanco with gentamycin in allergic)
21. VSD – membranous septum, harsh systolic murmur at L sternal border, spontaneous closure in 30-50%, tx – for small vsd observe, for large vsd and significant shunt, surgical repair and endocarditis prophylaxis.
22. ASD – wide, fixed splitting S2, tx – if small observe, if large surgery
23. PDA – machinery murmur, wide systemic pulse pressure, tx c indomethacin then surgery
24. Aortic Coarctation – UE HTN c LE hypotension, rib notching, LE claudication, HA, dx with MRA or contrast aortography, tx is surgery (best at 4-8yo).
25. Tetralogy of Fallot – PROVe (Pulm HTN, RVH, Overriding aorta, VSD), kid squats to increase systemic resistance, thus decreased R to L shunt, cyanosis in kid >1yo, CXR c boot shaped heart, confirm dx with cath, tx is surgery, endo prophylaxis
26. Transposition of great vessels – MCC of cyanosis in 24hrs of birth, tx c surgery
27. Initial Tx’s: CHF à thiaz, bb, acei, arb, aldo ant; Post-MI à bb, acei, aldo ant; DM àacei, bb, thiaz, arb; recurrent strokes à thiaz, acei
28. Hyperaldosteronism – hypokalemic met alkalosis, PRA ratio, captopril-suppression test, high aldo level, 24hr urinary aldo, salt loading test
29. Pheochromocytoma – 24hr urine collection for VMA, MRI to visualize adrenal tumors, MIBG if chemistries positive by CT/MRI are negative.
30. Renal artery stenosis – renal U/S c Doppler, captopril scanning, CT/MRA, high renin, ACEI contraindicated if B/L
31. Urgent v Emergent HTN – Urgent is just one high reading (give nitroprusside or lobetolol, wait til BP goes down and d/c home). Emergent is when there are signs of end-organ damage (must admit and do workup).
32. PAD – claudication, rest pain, ulceration at medial ankle, dx c ankle-brachial index before/after exercise, angiography, MCC is atherosclerosis, tx c meds (pentoxyfylline, cilastazol, cab’s), angioplasty/stenting, avoid constricting drugs (bb)
33. Temporal Arteritis - >55yo pt c HA, scalp tenderness, visual s/s, next step is low-dose steroids (before temporal a biopsy or getting ESR).
34. Polyerteritis – HTN, abdominal pain, numbness in legs, skin findings, cns s/s, dx c biopsy, tx with steroids.
35. AV Fistula – thrill/bruit over fistula (buzzing sound), dx c angiography, tx c surgical excision, if congenital do conservative management instead.
36. Varicose veins – pain, pigmentation, superficial ulcer, tx c elastic stockings
37. Superficial thrombophlebitis – pain, erythema, embolism is rare, tx c warm compression, limb elevation and NSAIDS.
38. Deep vein thrombophlebitis – pain, swelling, fever, + Homans sign, PE is risk, so must do plethysmography or Doppler, tx c heparin/warfarin, filter if recurrent.
39. Dissecting aortic aneurysm – sharp CP radiating to back, dx c CT, TEE or MRI, tx – 1st decrease BP (nitroprusside), 2nd - If ascending aorta (up to aortic arch) do surgery, if descending aorta use meds
40. Abdominal aortic aneurysm – bruit, dx with U/S, see abdominal notes
41. Aneurysm of thoracic aorta (nondissecting) – may compress adjacent structures causing CP, dysphagia, hoarseness, dx c aortography, Atherosclerosis is MCC, also due to cystic medial necrosis. Tx c surgical graft replacement.


SKIN:

42. HSV – type 1 at mouth, type 2 in genitalia. Recurrent erythema nodosum is characteristic. Dx c Tzank, tx c acyclovir
43. Herpes zoster (shingles) – dermatomal, reactivated at dorsal nerve root, tx c acyclovir
44. Varicella (chickenpox) – lesions in all stages of development, tx c benadryl. In 1st TM, causes microcephaly, chorioretinitis, IUGR and cataracts. Treat neonates with VZIG if mom contracted varicella within 5 days of delivery.
45. Impetigo – honey-crusted lesions. S aureus and B-hemolytic strep. Tx c muciprocin
46. Rubella – 3 days of cervical/suboccipital/postauricular node enlargement, prevention best c immunization before 1st TM to prevent triad: visual (cataracts), hearing loss, heart (PDA) defects.
47. Measles (Rubeola) – looks like spilled red paint over your head (rash spread beind ears and over forehead to neck to trunk and extremities), prevent c immunization
48. Roseola – 3-5 days of fever, and THEN rash after (never together). No tx
49. Erythema infectiosum – 5th dz – slapped cheek appearance, parvo B19, causes aplastic crisis in sickle cell patients, no tx
50. Rocky Mountain Spotted Fever – fever, rash on wrists then palms and soles, dx c weil-felix test, tx c tetracycline (chloramphenicol if pregnant)
51. Lyme dz – erythema chronicum migrans c central clearing, tx is doxycycline (amoxicillin if pregnant and children 60yo, large tense blisters, - nikolsky, IgG/C3 at dermal-epidermal junction, tx c prednisone, tetracycline, azathioprine (remember BCDE – Bullous pemphigoid, C3 at Dermal Epidermal junction)
57. Pemphigus Vulgaris – 40-60yo, multiple flaccid bullae, + nikolsky, biopsy shows acatholysis, antibodies to epidermal Ag, tx is prednisone, fluids, tetracycline
58. Dermatitis herpetiformis – itchy papulovesicular eruption usually on shins, - nikolsky, a/w celiac sprue, tx c gluten-free diet and dapsone (r/o G6PD first)
59. Factitial Dermatitis – no rash in nonreachable areas (midback, butterfly sign)
60. Acne Vulgaris – common acne. Tx c 1st benzoyl peroxide, 2nd topical/oral Abx, 3rd Topical retinoids, 4th Isoretinoin (r/o pregnancy first)
61. Hereditary angioedema – AD, C1 esterase inhibitor deficiency, subQ/mucosal edema
62. Pilonidal cyst – swelling, tender sacral mass, tx c antibiotics, I&D
63. Epidermoid cyst – contains keratin, asymptomatic, if infected (I&D, abx), if not excise
64. Capillary Hemangioma – strawberry nevus, reddish-purple hemangioma, tx c pulse dye laser therapy
65. Cavernous Hemangioma – purplish vascular anomaly, tx c reassurance, compression
66. Seborrheic keratosis – benign skin tumor in elderly, brown flat macule that appears “stuck-on”. Observe unless eruption is multiple then do shave excision and curettage, cryotherapy
67. Port-wine stain – a/w sturge-weber syndrome, brain calcfications, seizures
68. Actinic Keratosis – precursor to SCC, sun induced kyperkeratotic coarse lesions that are hard to remove. Tx c cryosurgery, 5FY, excision
69. Squamous cell ca – generally from the lower lip down. Ulcer that won’t heal. Tx c surgery or radiation
70. Basal cell ca – generally from upper lip up. Pearly nodule c rolled border. Surgical removal has high cure rate.
71. Melanoma – ABCD (asymmetry, borders irregular, color variation, diameter >6mm), MC is superficial spreading type, dx c total excision, loves to metastasize
72. Behcet’s syndrome – apthous ulcers, genital ulcers and uveitis, tx c d/c abx, chlorambucil
73. Dermatomyositis – difficulty rising from chair, proximal weakness, gottrons sign (purple papules on knees and knuckles), dx c mucle biopsy, tx c prednisone
74. Lofgren Syndrome – fever, erythema nodosum (LE nodules), and sarcoidosis.
75. Amyloidosis – macroglossia, waxy papules on face, congo red stain on biopsy
76. Scleroderma – raynauds, dysphagia, masklike face, tight skin, dx c skin bx, tx symptomatically or c D-Penicillamine, a/w CREST syndrome
77. Tuberous sclerosis – retinal phacomas, seizures, MR, sebaceous adenomas, ash-leaf hypopigmented macules, tx c seizure control.
78. Porphyria Cutanea Tarda – no abdm pain, but + red urine and vesicles on back of hand after having alcohol, drugs, estrogens, a/w Hep C, tx c 1st stop EtOH then phlebotomy
79. Acute Intermittend Porphyria – abdm pain, weakness in shoulders/arms, change in behavior. Blocks porphobilinogen deaminase, high ALA in the stool.
80. Acathosis Nigrans – black axillary/neck patches, a/w PCOS, DM, obesity and abdm adenocarcinoma. Next step is get fasting glucose to rule out insulin resisitance.
81. TTP – fever, thrombocytopenia (causing petechia/purpura), MAHA, renal problems (hematuria) and CNS symptoms (depression, HA, psychosis). Tx c plamapheresis
82. DIC – all labs messed up (BT, PT, PTT, fibrinogen, fibrin split products) causing cutaneous hemorrhage and ecchymosis. Tx – 1st treat primary cause, 2nd heparin
ENDOCRINE:
83. Thyroid nodule – 1st do TSH, then do FNA (preferred) or scan to see if its hot or cold (cold is malignant, if hot, observe – do not biopsy). MC benign is follicular adenoma, MC malignant is papillary (psammoma bodies), must as h/o radiation, worse if pt is male, >40 or young, distant mets. If results turn out that it’s a cyst, aspirate it and follow-up, if cancer, surgery c radioiodine (if papillary or follicular).
84. Goiter – high or low iodine uptake, lithium/amiodorone use, familial, tx c levothyroxine. Do not d/c drug, just continue the drug and add levothyroxine.
85. De Quervains (subacute) thyroiditis – painful thyroid, tx is NSAIDS
86. Sick Euthyroid Synd – low T4/T3, normal TSH. No s/s, just a goiter. Tx - nothing
87. Riedel’s – tracheal compression due to sclerosing fibrosis (rare)
88. Hashimoto’s – antimicrosomal ab, tx c levothyroxine
89. Congenital hypothyroidism (cretinism) – jaundice, lethargy, umbilical hernia, low T4, high TSH, tx c synthroid (levothryoxine)
90. Adult hypothyroidism – fatigue, myxedema, cold intolerant, wt gain, eyebrow thinning, high tsh, low T4, MCC is hashimotos, but also d/t prior graves tx, sheehan’s, amiodorone, lithium, tx c synthroid
91. Graves – low tsh, high T4, tachy, palpitations, weight loss, opthalmopathy, smooth goiter, A. fib, tx c BB’s (tremor and tachy), PTU, methimazole, radioactive iodine or subtotal thyroidectomy. In pregnancy, PTU can be used, as well as surgery if appropriate. Pt 40yo get radioactive iodine.
92. Toxic Nodule – high RIAU, no eye s/s, nodular goiter, on scan there is ONE area of increased uptake, whereas the rest its decreased (in toxic multinodular goiter (plummers disease), there are several areas of increased uptake and in Graves the entire gland has increased uptake)
93. Thyroid storm – very high fever, delirium, n/v, abdm pain, high t4, low tsh, tx c supportive care first (decrease temp, arrhythmia, BP), BB, glucocorticoids
94. Type 1 DM – polyuria/dypsia/phagia, islet cell ab, HLA DR3/4, low C-peptide, tx c insulin. If having surgery, give 10 units insulin in AM, and then 0.1U/kg/hr infusion.
95. Type 2 DM – polyuria/dypsia. Fasting glucose >126, random >200 on 2 visits. Tx first with diet/weight changes (decrease calories and carbs), oral agents, insulin. HBA1c to monitor glucose over 2-3 months. For retinal neovascularization, give laster photocoagulation therapy. For nephropathy, check for microalbuminuria (1st sign) and give ACEI. For neuropathy, give foot care and analgesia.
96. DKA – lethargy, n/v, polyuria, abdm pain, confusion, kussmaul breathing, fruity breath, glucose 400-600, anion gap met acidosis. Tx Isotonic fluids with insulin, replace K+ if needed (prevent cerebral edema).
97. Hyperosmolar coma – dehydration, lethargy, confusion, coma, high glucose without ketones, tx c fluids, insulin and electrolyte replacement.
98. Lactic Acidosis – coma, confusion, hyperventilation, no ketones, anion gap met acidosis, rare a/w metformin, tx etiology (starvation).
99. Pt with high blood glucose in the morning? Get 4AM blood glucose. If its high (Dawn effect), then increase morning NPH, if its low (Samogi effect) then decrease night-time NPH.
100. Insulinoma – lethargy, diplopia, HA, glucose 50yo every 3-5yrs; FOBT - >50 annually; DRE - >40 annually; PSA - >50 annually in normal risk, >40 annually in high risk; Pap smear – onset of sexual activity or 18yo annually for 3 consecutive years then however often; Pelvic exam – 18-40yo every 1-3yr, >40 annually; Endometrial biopsy – menopause/high risk annually; Self breast exam - >20 monthly; Clinical breast exam – 20-40 every 3 years, >40 annually; Mammogram – 40-49 every 1-2 yrs, >50 annually.
168. Cancerous Occupation Hazards – aromatic amines c bladder ca, arsenic c lung/skin/liver ca, asbestos c mesothelioma (bronchogenic MC), benzene c leukemia, mustard gas c lung/larynx/sinus cancer, vinyl chloride c liver cancer
169. Hodgkin’s – fever, night sweats, chills, weight loss (like TB), and painless cervical adenopathy. Dx c CT chest/abdm and then lympangiography and then biopsy (for treatment purposes). Reed-sternberg cells. Tx – If no B s/s (fever, wt loss, sweats) give radiation alone. If B s/s give chemotherapy (MOPP or ABVD)
170. Non-Hodgkin’s – variable nodes, monoclonal B/T-cell proliferation, dx c CT chest/abdm/pelvis then other stuff like BM bx, PET scan, gallium scan. Tx c radiation and chemo (CHOP) c Rituximab (CD20 Ab).
171. Acute Lymphocytic Leukemia – kids, blasts, tx c intrathecal chemo (MTX)
172. Acute Myelogenous Leukemia – M3 causes DIC, Aeur rods, blasts, add All-trans retinoic acid (Vit A) to tx.
173. CML – high WBCs, high PMNs, splenomegaly, LUQ pain, fullness and early satiety, decreased LAP, dx c phili chromosome (t9;22 of brc:abl) in BM, tx c Imatinib (Gleevac).
174. CLL – elderly, high WBCs, high lymphocytes, splenomegaly, dx c smudge cells, no tx if no lymphocytosis, if + lymphocytosis give fludarabine or chlorambucil.
175. Hairy Cell Leukemia – CD10+ and TRAP+ (tartrate-resistant acid phosphatase), tx c cladribine
176. Mycosis fungoidis – cutaneous T-cell lymphoma (look at 1st aid picture), lion-like facies, tx c PUVA chemotherapy. If affecting peripheral blood, its Sezary syndrome.
177. Multiple Myeloma – high calcium, high OAF, high uric acid. Best initial test is X ray if bone pain or electrophoresis if high protein. Most accurate test is >10% plasma cells. Tx: 70yo get Meiphelen or Thalidomide
178. Aplastic Anemia – low rbc/wbc/platelet, drugs (chloramphenicol), parvo-B19 (sickle cell), tx: 50yo get cyclosporine + anti-thymocyte globulin
179. If pt has neck + pelvic mass after chemo the mass gets smaller, wheat test checks content of the lymph node? PET scan. So in a nutshell, a lymphoma gets excisional biopsy of the node, then PET scan, and chemo if they have B symptoms.
180. Adverse effects of chemo: Vincristine/blastine – peripheral neuropathy, cyclophosphamide – hemorrhagic cystitis, Busulfan/Bleomycin – Irreversible Pulmonary Fibrosis (that’s why Lance Armstrong refused it), Cysplatin – renal dz, ototoxicity, anemia. Overall MC adverse effect with chemo drugs is sterility.
181. Lung cancer – chronic cough (MC s/s), wt loss, smoker, hemoptysis. Dx: 1st CXR, then biopsy. Tx: Small cell get chemo only, Non-small cell – chemo c radiation. Horner’s syndrome – unilateral ptosis, meiosis, anhidrosis due to compression of ipsilateral superior cervical ganglion by lung tumor, particularly SCC. SVC syndrome – obstruction of SVC causes facial swelling/plethora, dyspnea, cough, JVD. Pancoast syndrome – tumor of the superior sulcus causes brachial plexus s/s. Small cell causes Cushings syndrome (ACTH) and SIADH, SCC causes hypercalcemia (PTH-like peptide)
182. Solitary nodule – 1st step get old x-ray. If present and same size, its benign (send home), if increase in size its probably cancer. If it wasn’t there, assess risk (high is smoker and >35, low risk is nonsmoker and 35 except if B/L, lumpy and s/s only occur c menses. If 35yo. If mammo was already done, get FNA. If after biopsy, mass goes away, send pt home. Tx: tamoxifen, mastectomy, radiation, axillary dissection, chemotherapy (c platinum) if + nodes.
184. Prostate cancer – s/s of BPH c hematuria, high PSA (only to screen/monitor, not for dx), irregular/boggy, back pain. Tx c surgery. If +mets, then do orchiectomy, leuprolide, flutamide, DES, but no chemo. Only do TURP and radiation of mets is local (bone).
185. Colon cancer – R sided bleeds (bloody stools), L sided obstructs (constipation), wt loss. Dx c colonoscopy. Tx c surgery and 5-FU and then f/u CEA levels. If mets (MC is liver) to liver do surgery, but anywhere else do chemo.
186. Pancreatic cancer – 40-80yo male smoker c jaundice, wt loss and vague abdm pain. May have migratory thromboplebitis (Trousseau’s syndrome) or palpable, nontender gallbladder (Courvoisier’s sign). Dx c CT, then FNA. Tx c whipples.


HEMATOLOGY:

187. Microcytic (MCV 10,000), granulocytes (post chemo), FFP (bleeding diathesis like DIC, warfarin poisoning, liver failure), cryoprecipitate (vWD and DIC). MCC of transfusion rxn is lab error. If it occurs, 1st step is stop transfusion.
205. Platelet problems = skin, gums, nose, gingival (ALL SUPERFICIAL), GI, CNS and vaginal bleeding; Factor problems – bleeding into join and muscles (DEEP), GI, CNS.
206. von Willebran Dz – high PTT, normal PT, high BT, normal plt/rbc count, AD (look for family history) (a platelet type of bleeding c a normal platelet count). Best initial test is bleeding time, then ristocetin level. Best tx c desmopressin (DDAVP)
207. Hemophilia A/B – really high PPT, normal PT, normal BT/plt/RBC, looking for delayed hemarthrosis in males only (A is factor 8, B is factor 9).
208. DIC – high PT/PTT/BT, low plt, low RBC, low factor 8.
209. Liver failure – high PT, normal/high PTT, normal BT, normal/low plt/RBC, jaundince, normal factor 8, do not give vitamin K (ineffective), give FFP’s.
210. Heparin – high PTT, thrombocytopenia. Tx c d/c drug
211. Warfarin – high PT, vit K antagonist (2,7,9,10), tx c FFP (fast) or vit K (slow), skin necrosis
212. ITP –low platelets, high BT, h/o URI, next step is steroids (just treat it), auto-platelet Ab, if platelets fall 15 in pt c no risk facts. If PPD is positive, proceed to CXR, if (-) take INH for 9 months, if + get sputum AFB. If PPD is negative, repeat it in 1-2 weeks to rule out false negatives. If pt had PPD in the past that was +, don’t do PPD again (it will always be positive), go right to CXR.
240. Endocarditis – fever and a murmur is key, h/o IVDA is s.aureas at tricuspid valve, #1 dx is blood culture (not ECHO), #2 dx is ECHO (TTE type, not TEE). For dental procedures (must be dental procedure c blood, cant be dental fillings) give amoxicillin or clinda if allergic, for GI/GU (strep bovis) procedure give amox + genta, or vanco + genta if allergic. Strongest indication for surgery is ruptured valve. So, 1st step is blood culture, 2nd step is start abx while waiting for results.
241. Thrush – oral candida, removable white mouth patches (Candida CAN come off, hairy leukoplakia cant). Tx c nystatin mouth rinse.
242. Lyme Disease –problems in joints, CNS (b/l bells palsy), heart (3 degree AV block). If its just a tick bite and no s/s, do nothing. If it’s a bite c lyme rash give amox (pregnant or kids) or doxy (not serology). If pt has b/l bells palsy get serology. If av block c cns s/s (except bells palsy) give ceftriaxone next.
243. HIV – 1st ELISA, 2nd western blot (in kids, 1st is PCR). Peripheral neuropathy c stavudine/didanosine, anemia c zidovudine, rash c tmp/smx (start dapsone), nephrolithiasis c indinavir. MC overall adverse effect is increase lipids and glucose levels. Prophylaxis: 5days, Edema, Erythema, Lymphadenopathy, Myositis, Conjunctivitis). Next step is Echo (r/o coronary aneurysms). Tx c Aspirin + IVIG
264. Takayasu arteritis – Chinese 30-50yo female c pulselessness on 1 side. Dx c angiogram of aortic arch (coronaries to r/o stroke). Tx c steroids, cyclophosphamide
265. Wegeners – nasal (sinusitis), lung (hemoptysis, dyspnea), kidney (hematuria), c-ANCA, tx c cyclophosphamide
266. Fibromyalgia – young female with pain all over, multiple points of tenderness, irregular sleep pattern, anxiety, exams all normal. Tx c antidepressant, NSAIDS
267. Polymyalgia Rheumatic – old female c pectoral/pelvic pain/stiffness, elevated ESR, normal biopsy, a/w temporal arteritis. Tx c steroids.
268. Polymyositis – 40-60yo female c proximal muscle weakness, elevated ESR/CPK, abnormal muscle biopsy, dx c 1st muscle biopsy, then EMG. Tx c steroids
269. Dermatomyositis – same as above, but c rash (heliotrope rash around eyelid).
270. Paget’s disease - >40yo male c pevic/skull damage, hats don’t fit anymore, deafness, paraplegia, bone pain, very high alk phos, normal calcium/phos, increased risk of osteosarcoma. X-ray shows thickened bones. Tx c NSAIDS, bisphophonates (Etidronate) and calcitonin.
271. Herniated disk – most at L4-5 (weak big toe), and L5-S1 (reduced Achilles reflex), positive straight leg test.
272. Carpal Tunnels – median nerve compression (thumb, pointer, middle finger), Tinnels sign (tapping wrist causes numbing), Phalens sign (flexing wrist), tx c rest, splint, workplace modifications, then NSAIDS.
273. Osgood-Shlatter – inflammation of tibial tubercle in boys. Tx c rest and immobilization.
274. Slipped Capital Femoral Epiphysis – Obese kid c painful limp. Dx c xray. Tx c surgical pinning >5yo.
275. Legg-Calve-Perthes – non-obese kid c a limp (d/t avascular necrosis @ hip). Tx c observation and pain relief, 2nd is bracing, 3rd is surgery.
276. Osteoporosis – risks include early menopause, alcohol, Caucasian, thin body, tobacco. Dx c DEXA >-2.5 (-1 to -2.5 is osteopenia). Tx 1st weight-bearing exercise, 2nd lifestyle (smoking, alcohol cessation), 3rd calcium/vit D, bisphosphonates, etc.
277. Patellar tendonitis – an NBME 3 test question, aka jumper’s knee, patellar tenderness due to overuse and jumping sports resulting in quadriceps contraction. Tx c rest, nsaids, quadriceps stretching.
278. Osteosarcoma – 10-25yo c knee pain, mass, limping, high alk phos. X-ray c sunburst appearance. Tx c surgery and chemotherapy
279. Osteoid Osteoma – bone pain worse at night and relieved c NSAIDS. Tx c nsaids
280. Osteochondroma – bone pain, xray shows pedunculated metaphyseal tumor at distal femur. Tx c surgery.
281. Ewing sarcoma – fever, pelvic/femur bone pain, swelling, xray shows onion skinning. Tx c radiation, chemo, surgery.
282. Reflex Sympathetic Dystrophy – burning pain, skin changes (color/temp), edema in a pt who had prior injury to that area. Tx c pain management (hard to do).
283. Nursemaids Elbow – from pulling your childs arm, he develops severe pain at elbow and will not use that arm. Tx c pushing back the head of the radius while the arm is supinated and flexed. Kid will feel much better immediately.

NEUROLOGY:

284. Migraine HA – 70% unilateral, throbbing, aura, photophobia, family history, possible risk of stroke, worse c OCPs/EtOH/chocolate. Tx c NSAIDS, triptans (contraindicated in heart disease), ergots. Prevent c BB 1st, cab’s 2nd, sodium valproate/SSRI/TCAs.
285. Cluster headache – same time every month/year, males mostly, tearing, redness, pain, rhinorrhea, feels like an ice-pick is shoved in your eye (old question). Tx c 100% oxygen 1st, steroids 2nd.
286. Temporal Arteritis – >50yo c unilateral temporal HA, scalp tenderness, vision changes, high ESR. 1st step is give steroids, 2nd step temporal artery biopsy.
287. Pseudotumor Cerebri – aka Benign Intracranial HTN – increased ICP, HA, visual changes, obese female, papilledema, no focal CNS findings, a/w vitamin A toxicity. Dx c MRI 1st then LP 2nd, tx c azetazolamide.
288. Trigeminal Neuralgia – pain whenever you touch your face @ 5th cranial nerve distribution. Tx c carbamazapine. Definitive treatment with surgical rhizotomy.
289. Essential Tremor – at rest and motion. Tx c propranolol. (Tremor at rest only is Parkinson’s or hyperthyroidism, tremor c motion only is cerebellar dysfunction)
290. Nystagmus/Vertigo – if + focal defecits, the problem is central (vertical nystagmus): cerebellum (CT/MRI), M. Gravis (MRI), Stroke (MRI/CT), phenytoin without an hearing loss or tinnitus. If no focal defecits, the problem is peripheral (in the ears), so pt will have hearing loss and tinnitus. If pt only has vertigo, its benign positional vertigo. If pt has hearing loss and tinnitus with it: Miniere’s disease (chronic disease), Acoustic Neuroma (look for ataxia), Labyrinthitis (acute viral infxn)
291. Epilepsy (as per Kaplan on what is important): do not treat 1st time seizures unless there is a family history, EEG is positive or pt has status epilepticus.
292. Status Epilepticus – Dx: 1st sodium, 2nd glucose, 3rd calcium, 4th hypoxia, toxicology, CT-head, EEG (last!). Tx: 1st Benzo (lorazepam IV), 2nd Phenytoin, 3rd Barbiturate, 4th Anesthesia (succinylcholine/propofol – these will just stop the shaking, wont stop the seizure).
293. Absence seizures – kid stares into space, doing poorly in school, eye blinking, lip smacking, EEG c 3/sec spike and wave pattern. Tx c ethosuximide.
294. TIA – focal, abrupt onset lasting less than 1 hour, symptoms resolve after 1 day. Risk of stroke in days to weeks. Amorosis fugax (curtain over an eye due to retinal dysfunction) needs Doppler U/S of carotids or MRA. Give heparin acutely (if no contraindications), then long term aspirin. If stenosis >70%, amaurosis fugax/TIA or small, non-disabling stroke do CEA (carotid endarterectomy) and give aspirin. If stenosis 50yo, high risk (COPD, cardiovascular, renal), women who WILL become pregnant in winter, household contacts of high-risk pt (to protect the high-risk pt). Pneumococcal - >65, comorbidities.
341. Formulas: [A = True Positive; B = False positive; C = False negative; D = True Negative] (positives always on top) Sensitivity = TP/TP+FN; Specificity = TN/TN+FP; PPV = TP/TP+FP; NPV = TN/TN+FN; Attributable risk (attrib = subtract) = (a/a+b) – (c/c+d); Relative risk (only for propective studies like cohort study)= (a/a+b) / (c/c+d); Odds ratio (only for retrospective studies like case-control) = ad/bc; attack rate (how many ppl get attacked c dz) = a+c/b+d.
342. Power = rejecting the null when its false (a good thing, like saying Viagra does not treat constipation, which it doesn’t do). However, sometimes FDA may not always make the right choice and end up approving something that doesn’t work, or not approving something that works. Type 1 error – rejecting the null when it’s true (saying Viagra does not treat erectile dysfunction). Type 2 error = acceptance of the null hypothesis when it is false (saying Viagra treats constipation). Generally, the only way to increase power is to increase the sample size.
343. Mean = average; Median = middle #, Mode = MC #.
344. Confidence Interval = [mean +/- Z score x standard error of mean], where Z is the standard score (If confidence interval is 95%, Z is 2, if CI is 99%, Z is 2.5) and standard error of mean is (S / square root of N), where S is the standard deviation and N is the sample size. For example, old TQ said the mean was 67%, standard deviation was 8% in a sample size of 16, calculate a 95% CI: (67 +/- 2 (8 / square root of 16) = (67 +/- 2 (8/4)) = 64 +/- 4. The answer was 63-71.
345. When they give you a chart with different confidence intervals, just look for the one that has 1 within the range (ie. 0.89-2.3, not 1.12-2.25 or 0.56-0.93). That one is NOT statistically significant, meaning the risk is the same. If 1 is not within the range, is is statistically significant. If it was over 1 (1.12-2.25 used above), there is an increased risk. If it was under 1 (0.56-0.93 used above), there is a decreased risk.
346. When given statistical scales and asked for the statistical test: Nominal is categorical (how many you can split into groups, like genders, ethnicities, etc), Interval is a measurement (height, wt, BP, etc.). Pearson correlation = 2 intervals; Chi-square = 2 nominals; t-test = 1 nominal + 1 interval. For example, if you want to find out if men do better than women on step 2. Men vs women is nominal, Step 2 is an interval, therefore one of each makes it a t-test.
347. If given the following data: After surgery: 90% survive 1year, 75% survive 2years, 50% survive 3years, 40% survive 40%, and asked: what is the life expectancy after surgery? Always pick closest to 50%, so the answer would be 3 years. If asked, if a pt survives 2 years, what is the chance of surviving 3 years? Always put the # ending on top, # starting on bottom, so it will be 50/75, or 67%.
348. Cohort study (think – Cohort to Go Forth) – a prospective study where people are followed for a period of time. Advantages are that incidence (# of new cases) can be determined, there is an accurate relative risk (remember RR with cohort), and less control group bias. Disadvantages are that it takes too long, expensive, the sample size can get too large, and you might run into an ethical problem.
349. Case-control study – a retrospective study where you start with an outcome and then check backwards to evaluate the risk or cause. Advantages are that it’s cheap and easy, small sample size and minimal ethical risk is involved. Disadvantages are that incidence (new cases) are not determined, RR is just approximated (not exact, just taking odds, remember OR c case-control) and that there is some control group bias. Kaplan says, if you have no idea which type of study it is, pick this one.
350. Confounding bias – when hidden factors affect the results. For example, an experimenter measures the # of ashtrays owned and incidence of lung cancer and finds that people c lung cancer have more ashtrays. He or she then concludes that ashtrays cause lung cancer. Smoking is the confounding bias here, because it increases both ashtrays and lung cancer. So how can you prevent this? Do multiple studies.
351. Lead-time bias – when you confuse the facts that early screening will increase life expectancy. Look for false estimates of survival rates. For example, if I diagnosed you with cancer at 18 and you lived until 30, you will think I treated you for 22 years. However if I didn’t diagnose you until 25 and didn’t treat you after, and then you lived until 30, you will think that you only survived 5 years. The difference is not that my drug treats you better, but that I am diagnosing you earlier, thus getting a good lead on time. The solution here is to measure the “back-end” survival (ie. Getting the age 30 as the age that they both die at, whether they were treated or not).
352. Recall bias – subjects can’t remember events in the past. Solution is to make them confirm information with other sources.
353. Late-look bias – subjects die before the end of the survey, so your information gets distorted. For example, a survey finds that AIDS pts only get mild symptoms. This is wrong because they die before the really bad symptoms occur. Solution here is to stratify the disease by severity.
354. Experimenter/Interviewer bias – aka Pygmalion effect - when the experimenters expectations are inadvertently communicated to subjects, who then produce the desired effect. Solution is to make the study a double-blind one.
355. Selection bias – aka sampling bias – when the sample selected is not a representative of the population. For example, taking the people from a health club and doing a survey on the lungs in the general population. Another cause is when a study uses hospital records to estimate population prevalence (Berkson’s bias). For example, a doctor says all the people in NY are sick because all day he works with sick patients in NY.
356. Measurement bias – aka Hawthorne effect – when being observed makes you change how you answer to questions. Also, when the way the information is presented makes you answer in a certain way. For example, asking a pt “you don’t like your doctor, do you?” The pt is likely to say no because of the way the question was presented. In the law world, its termed “leading.” Prevent this by having a control/placebo group.

________________________________________________________________________

OB:
357. Numbers to note: How many weeks in each trimester? 13; what is so special about 37 weeks? Lungs are muture b/c lethicin/sphingomyelin ratio is 2:1; what is the risk of having Down’s if mom is 35? 1/350; risk @ 40? 1/100; risk @ 45? 1/50, so you absolutely must recommend amniocentesis. Pregnancy weight gain is about 25 pounds (5 in first 20 weeks, 1 pd every week after). Uterine height: 8 weeks @ iliac, 14 weeks @ pubic symphisis, 20 weeks @ umbilicus, 38 weeks @ xiphoid process.
358. Dates to note: 6-8wks is prenatal workup. 15-18 weeks is triple screen. 18 weeks is ultrasound. 26 weeks is glucose challenge test for DM. 35 weeks is GBS culture.
359. Diagnostics: Ultrasound (noninvasive, no adverse effects, done at 18-20 weeks), Chorionic villus sampling (“CVS,” invasive, done at 9-12 weeks, best for early gestation so mom has the chance to choose an abortion, may be fatal, f/u c triple screen after), Amniocentesis (done at 15-20 weeks for genetic purposes or high risk patients, done at 24 weeks for Rh isoimmunization, done at 34 weeks for gestation age, pregnancy loss about 0.5%)
360. Diabetes workup: Done at 24-28 weeks in normal pt. Done c prenatal workup (6-8 weeks) if pt is obese or has h/o macrosomic baby, h/o DM or family h/o DM. The pt will come to your office fasting for 1 hour, her blood sugar should be >140. If 2cm (if none, pt has false (Braxton-hicks) contractions and send her home). MC risk factor is previous preterm labor. Dx c fetal fibronectin (if +, tocolytics and steroids, if (-), send home). Management: 1st L lateral decubitus position, bed reast, O2 and IVF. 2nd Start tocolytics (useless >4cm dilatation, r/o contraindications first), get cervical/urine culture before giving IV Pen G (for GBS), IM betamethasone and send home.
394. Tocolytics – 1st Mg Sulfate (calcium blocker that may cause resp depression, loss of DTRs and pulmonary edema. If so, give IV calcium gluconate). 2nd Ritodrine/Terbutaline (B-adrenergic agonists that may cause hypotension and tachycardia so don’t give in pt c heart disease or DM). 3rd Nifedipine (calcium blocker that may cause hypotention). 4th Misoprostol (prostaglandin inhibitor that may cause in utero ductus arteriosus closure, so don’t give if gestation age >32 weeks). Some contraindications to tocolytics include (conditions where you may need to deliver) abruption placenta, ROM, chorioamnionitis, fetal demise, late decelerations, eclampsia, severe eclampsia and cervical dilatation >4cm.
395. Post-date pregnancy (>40wk): complications include increased perinatal mortality, macrosomia, need for c-section, dysmaturity syndrome (mother’s support runs out). 1st step is to check dates (if dates still unsure, continue c conservative treatment and biweekly NSTs), 2nd step is induction of labor. If cervix is favorable (soft), begin aggressive tx c oxytocin and artificial ROM. If cervix is unfavorable (hard), give prostaglandins c oxytocin and wait for spontaneous delivery.
396. Transient HTN – unsustained high BP without proteinuria or edema. No tx.
397. Chronic HTN – high BP before 20 weeks gestation. Tx c methydopa, hydralazine.
398. Mild preeclampsia – mild HTN (140/90), petal edema, 2+ proteinuria after 24 weeks gestation. Management: 36 wk – deliver
399. Severe preeclampsia – sustained BP >160/110, >3+ proteinuria, edema, epigastric pain, HA, blurred vision, thrombocytopenia (r/o HELLP synd). Tx: prompt vaginal delivery c oxytocin, MgSO4 (to prevent convulsions) and IV hydralazine/lobetolol.
400. Eclampsia – HTN, proteinura, edema, seizures. Tx: 1st ABC’s, 2nd MgSO4 to stop seizure (do not deliver 1st, you can never attempt delivery if pt is seizing), 3rd aggressive prompt vaginal delivery c oxytocin and hydralazine to decrease BP.
401. HELLP syndrome – hemolysis (schistocytes), elevated LFTs, low platelets. No CNS or renal problems (r/o TTP), no h/o URI/GI infection (r/o HUS). Tx c steroids and prompt delivery.
402. Never recommend termination of pregnancy, unless: 1 – pulmonary HTN in mom, 2 – Marfan’s syndrome c an aortic aneurysm >4cm, 3 – Eisenmengers syndrome (pulm HTN c bidirectional shunt, 4 – peripartum cardiomyopathy.
403. If they ask about rheumatic heart disease in the context of pregnancy, know about mitral valve stenosis management (diuretics 1st, vasodilators, then balloon vulvoplasty). Management of cardiac disease in pregnancy is bed rest, decreased physical activity, decrease weight, correct anemia, analgesics, vacuum delivery.
404. Management of hyperthyroid disease in pregnancy is to stay on PTU to prevent thyroid storm, but warn mom that baby might be mentally retarded or have IUGR.
405. DM in pregnancy – a/w fetal NTD (most common fetal anomaly), hypoglycemia (d/t maternal insulin, tx c IV glucose), hypocalcemia (failure of PTH synthesis after birth), polycythemia (d/t increased erythropoietin from intrauterine hypoxia), respiratory distress (to check lung maturity, phosphatidylglycerol is a better choice than L:C ratio), hyperbilirubinemia.
406. Prolonged latent phase – cervical dilatation 20hrs in primipara, >14rhs in multipara). MCC is analgesia, so tx is bedrest and sedations.
407. Prolonged active phase – cervical dilatation >3cm, but slow/no rate (40,000, brain/liver mets, >6 months of D&C) do radiation and chemotherapy (MAC: MTX, Adenomycin, Cytotoxin). If good prognosis, give MTX only and f/u hCG every week for 3months while on OCPs.
430. Uterine prolapse – loss of uterine support due to cardinal ligament dysfunction. MCC is childbirth. Best tx is vaginal hysterectomy c ant/post repair (yes, first!), but if pt refuses surgery, do Kegel exercises, estrogen HRT and pessaries.
431. Stress Incontinence – weak pelvic floor causes you to urinate whenever you sneeze/cough, none at night. Dx c Q-tip test. Tx c Kegel exercises, then surgery (Marshall-Marcheli-Kranz procedure).
432. Urge Incontinence – involuntary detrusor contractions causing spurts of urine to fall at any time. Dx c cystometric studies. Tx c anticholinergics (Ditropan)
433. Overflow Incontinence – denervated bladder (DM, MS, CVA) causes bladder to keep filling up, thus high residual volume even after urination. Tx c cholinergics (bethanecol).
434. Endometriosis – dymenorrhea, dyspareunia, infertility, uterosacral ligament nodularityin the cul-de-sac, chocolate cysts. Dx c laparoscopy. Tx: 1st OCP, 2nd Danazol and Leuprolide (best tx, but not 1st because of side-effects), 3rd surgical resection, 4th pregnancy (however hard, d/t infertility), 5th TAH/SBO. If endometriosis is present, and pt has no s/s, do nothing.
435. Chancroid – painful chancre (H. ducreyi – you cry c ducreyi) c ragged, rolled edges. Tx c Azithromycin
436. LGV – painless ulcer that heals and then forms painful nodes. Tx c erythromycin.
437. Granuloma inguinale – painless, beefy-red ulcer. Dx c Donovan-bodies on smear. Tx c Azithromycin.
438. Chlamydia – MC bacterial STD, can be asymptomatic or mild mucopurulent cervical discharge c or w/o cervical motion tenderness (CMT), (+) Cx/Ab test, (-) stain. Tx c azithromycin (1 dose) or oral doxycycline (7 days).
439. Gonorrhea – Lower GU causes d/c, itching, burning, dysuria; Upper GU causes abdo/pelvic pain. Disseminated when there is dermatitis, polyarthritis or tenosynovitis. Pt has vulvovaginitis c mucopurulent d/c c CMT on bimanual exam. Dx c chocolate agar, Gram (-) diplococci on stain. Tx (for GC and Chlamydia) Ceftriaxone + Doxycycline.
440. PID – lower abdominal pain, adnexal tenderness, CMT and fever 1 week after menses in a sexually active female. Cervicitis (only vaginal D/C, no pain – tx G/C), Salpingo-oophoritis (b/l lower abdo/pelvic pain c CVA tenderness – tx G/C), Tubo-ovarian abscess (pt will look septic, severe pain, n/v, dyschezia, fever – tx c Ampicillin, Gentamycin and Flagyl. If ruptured, ex-lap is done). Tx for G/C in these cases are: outpatient: ceftriaxone + doxycycline, inpatient: clindamycin + gentamycin
441. Gardnerella Vaginosis – fishy odor on whiff test, pH 6, clue cells, tx c metronidazole (clindamycin if pregnant in 1st TM)
442. Trichomonas vaginalis – frothy, green smelly discharge c strawberry cervix, pH 5. Tx c metronidazole for pt and partner (if pt pregnant, tx c vaginal betadine).
443. Candida yeast infection – itchy, burning, dyspareunia, cottage-cheese discharge, that sticks to the vaginal wall, pseudohyphae, pH 4, tx c nystatin or Amp B.
444. Contraception: remember effects of estrogen (increases BP, cholelithiasis, LFTs, HDL, art/venous thrombosis and decreases LDL) and progesterone (affects mood, increase weight, acne, increase LDL, decrease HDL). Absolute CI: pregnancy (causes VACTERL), liver dz, vascular dz (DVT, SLE, CVA) and hormonally-dependent cancers like breast). Benefits include decreased risk of ovarian/endometrial cancer, decreased dysmenorrhea/DUB/PID/ectopics.
445. IUD – put it in 1 week after menses and f/u in 1 week. Does not affect risk of STDs. Absolute contraindications include pregnancy, pelvic cancer, salpingitis, steroid use (pt on Crohns, asthma), h/o PID. Increased risk of ectopics and PID when placed.
446. Abnormal vaginal bleeding: Pre-menarchal (52yo - endometrial cancer). A neonate c vaginal bleeding is normal due to maternal estrogen, thus reassure mom.
447. Precocious Puberty – normally: breast development @ 9yo, pubic/axillary hair @ 10yo, growth @ 11yo, menarche @ 12yo. If only 1 stage occurs early, this is Incomplete isosexual precocious puberty, next step is CT brain/abdo/pelvis. If all stages occur early, this is complete isosexual precocious puberty, next step is tx c constant GnRH stimulation (to decrease estrogen). If pt has bone lesions and café-au-lait spots, pt has McCune-Albright Syndrome. If pt has high estrogen c a pelvic mass, they have a granulose-theca cell tumor, tx c surgery.
448. Dysfunctional Uterine Bleeding – MCC is anovulation d/t unopposed estrogen, so no secretory phase (d/t lack of progesterone) c unstable endometrial thickening. Pt will have h/o irregular, unpredictable menstrual bleeding without cramps. Next step is endometrial biopsy to r/o cancer. Tx c NSAIDS if she desires children, cyclic progestin therapy or daily combined OCPs if she doesn’t desire children or has menorrhagia.
449. Primary Amenorrhea – 1st step is pregnancy test (whether she says she is sexually active or doesn’t), 2nd step is physical exam: (+) breasts and (+) uterus -> check prolactin, if normal r/o imporferate hymen (cyclic menstrul pain c bulging hymen, predisposition to endometriosis, tx c surgery) and tx c progesterone. (+) breasts and (-) uterus -> get karyotype, if 46 XY, pt has Androgen Insensitivity Syndrome (Testicular Feminization, no pubic hair, next step is remove testes from abdm), if 46XX, pt has Rokitanky-Hausen syndrome (she will have pubic hair). (-) breast and (+) uterus -> gonadal dysgenisis, so next step is get karyotype to r/o Turners syndrome (45XO, webbed neck, far spaced nipples, streak ovaries, premature ovarian failure, needs estrogen).
450. Secondary Amenorrhea – 1st step is r/o pregnancy, 2nd r/o prolactinoma (if prolactin level is high, next step is MRI of head. If abnormal, pt has pituitary tumor, if normal, pt has drug-induced prolactinoma) and hypothyroidism, 3rd progesterone challenge test. If pt bleeds after 2 weeks (estrogen is adequate), check LH. If elevated pt has PCOS, if normal/low check TSH/prolactin again. If pt does not bleed after 2 weeks (inadequate estrogen) check FSH, if high pt has premature ovarian failure (next step is karyotype to r/o Turners vs Ovarian failure due to congenital adrenal hyperplasia), if normal/low pt has craniopharyngioma, next step is MRI. If MRI is insufficient, pt has Ahsermann’s syndrome (scarring due to prior D&C/D&E. Tx by surgically removing scarred tissue then giving high-dose estrogen for 1 month to regenerate lining). Again, if LH/FSH are high, next step is karyotype. If XO, pt has turners, if XX pt has ovarian failure (now r/o autoimmune dz versus CAH). If LH/FSH are normal or low, next step is MRI of head. If abnormal pt has pituitary tumor/destruction or hypothalamic dz (may be a/w Kallman’s syndrome (anosmia, amenorrhea), anorexia, exercise, tx c estrogen). If normal, pt has Asherman’s syndrome.
451. Breast mass in a female intraductal papilloma. Next step is galactogram-guided excision.
454. Polycystic Ovarian Synd – female, hirsutism, amenorrhea, infertility (MCC of infertility in women 1yo, tet spells, boot-shaped heart
488. Transposition of great vessels – cyanosis in 1st 24hrs, aorta from RV, pulm artery from LV, egg on a string heart, tx c balloon atrial septostomy, then arterial switch
489. Total anomalous pulm venous return – pulmonary veins drain into systemic venous circulation (partial or total), snowman heart. 1st medications, 2nd surgery
490. Truncus arteriosis – single great artery is origin of aorta and pulm arter and coronary artery, listen for truncal valve click. Tx: 1st treat CHF, 2nd surgery
491. VSD – MC CHD, holosystolic murmur at 1-2months, tx c subacute bacterial endocarditis prophylaxis
492. ASD – pulmonary ejection murmur plus wide, fixed split S2, no SBE prophylaxis, usually presents after infancy
493. Coarctation of aorta – HTN in UE, low BP in LE, poor femoral pusles, Turners synd, rib notching on CXR, tx c balloon angioplasty
494. PDA – premature babies, congenital rubella, continuous machinery murmur c wide pulse pressure.
495. Hypoplastic left heart – underdeveloped LV and aorta, vascular collapse in 1st week of life, ductus dependent, tx: 1st prostaglandin E, 2nd Norwood or transplant
496. Hydrocephalus – communicating (obstruction of arachnoid villi) or noncommunicating (Aqueduct of Sylvius stenosis, Chiari malformation at cerebellar tonsils or Dandy-walker cyst of 4th ventricle). Baby c rapid increase in head circumference, split sutures, bulging anterior fontanelle, setting-sun sign (of eyes), 6th nerve palse, papilledema, dx c CT scan (do not do LP in risk of herniation). Tx: 1st hyperventilate and elevate head, 2nd mannitol, 3rd ventriculoperitoneal shunt
497. Congenital cataracts – rubella, CMV, toxo, galactosemia, tx c surgery right away to prevent permanent visual impairment.
498. Congenital glaucoma – tearing, corneal clouding, photophobia, sturge-weber synd (facial port-wine stain, seizures, CNS calcifications), neurofibromatosis, rubella, tx c surgery.
499. Congenital deafness – Alports (nephritis c deafness), CMV, rubella, maternal drugs.
500. Osteogenesis Imperfecta – brittle bones cause multiple fractures in a kid, blue sclera, osteoporosis, family history, type I collagen disorder, teeth deformities.
501. Developmental Dysplasia of the Hip – subloaxation of femoral head from the acetabulum, causing asymmetric thigh creases, clicking sound, + Ortolani sign (hip reducibility), + Barlow sign (hip dislocatability), dx c ultrasound. Tx c harness, then closed reduction, then open reduction (>6mo age) if closed reduction failed.
502. Talipus Equinovarus – toes face medially, forefoot adduction. Tx c manipulative casting, then surgery if needed.
503. Transesophageal Fistula – dx c failure to pass nasal catheter to stomach, AXR shows air-distended proximal esophagus. Tx: 1st NGT, 2nd surgery
504. Duodenal atresia – bilious projectile emesis, a/w Downs syndrome, abdominal distention, double bubble on AXR (air-distended stomach and proximal duodenum). Tx: 1st correct fluids/electrolytes, 2nd surgery
505. Pyloric stenosis – nonbilious projectile emesis, olive-shaped RUQ mass, dehydration c hypochloremic alkalosis. Tx:1st fluid/electrolyte correction, 2nd pyloromyotomy
506. Meckel’s Diverticulum – 2yo c painless rectal bleeding and abdm pain. Dx c technetium-labeled nuclear scan (Meckel’s scan), tx: 1st correct life-threatening anemia, 2nd surgical excision.
507. Hirschsprung’s Disease – congenital megacolon causing obstruction, absense of Auerbach’s and Messner’s plexus, failure to pass meconium in 1st week, dx c 1st barium enema (shows transitional zone) rectal biopsy (aganglionosis). Tx: 1st fluid/electrolyte correction, 2nd Abx if enterocolitis suspected, 3rd surgical excision of ganglionic segment.
508. Hyaline membrane Disease – RSD, surfactant insufficiency, early onset (hours after birth) baby has tachypnea, grunting, nasal flaring and retractions. Early problems include breathing difficulty, metabolic disturbances and infection. Late problems include broncopulmonary dysplasia. Risk factors include prematurity, maternal DM and multiple pregnancies. Dx: 1st CXR (shows fine reticular granularity in b/l lungs), 2nd L:S ratio (should be >2:1) and phosphatridylglycerol. Tx: Prevention is the best tx (prevent prematurity, give maternal steroids 48-72 hours antepartum if 5yo c inappropriate voiding of urine. Tx: 1st behavioral techniques (bell, buzzer, bed time fluid restriction), 2nd Imipramine (last resort).
522. Dementia vs Delerium: Delerium (rapid onset, fluctuating consciousness, often reversible, perceptual disturbances, incoherent speech). Dementia (insidious onset, clear consciousness (until late in course), irreversible).
523. Alzheimer’s vs Vascular (Multi-Infarct) Dementia: Alzheimers dementia (women, older, chrom 21, linear/progressive, no focal defecits (key), supportive tx). Vascular dementia (men, younger than alzheimers, HTN, stepwise/patchy pattern, (+) focal deficits (key), tx underlying condition).
524. Alcohol – intoxication includes slurred speech, ataxia, disinhibition, impaired judgement, coma and blackouts. Withdrawal includes tremor, agitation, irritability, n/v, fever, seizures, delirium tremens (onset of delirium, vivid auditory/tactile/visual hallucinations, paranoid delusions 2-3 days post cessation of long-term heavy use). Tx intoxication supportively. Tx withdrawal c vital sign/electrolytes/Mg/thiamine/vit B12/folate/glucose monitoring. 2nd Hydration c thiamine before glucose (prevent Wernicke), 3rd benzodiazepine (chlordiazepoxide). Tx dependence c confrontation of denial and rehab (AA). Specific managements: Alcohol hallucinations (chlordiazepoxide, IVF, haloperidol), Wernicke’s encephalopathy (sudden ataxia, confusion, nystagmus, lateral rectus palsy from thiamine deficiency. Tx c thiamine) Korsakoff’s syndrome (severe anterograde/retrograde amnesia, confabulations and polyneuritis from thiamine defiency).
525. Opioids – intoxication includes euphoria, analgesia, hypoactivity, anorexia, drowsiness, n/v, constipation, pin-point pupils, hypotension and bradycardia. Overdose includes CNS/respiratory depression, pinpoint pupils, pulm edema, seizure, coma and death. Withdrawal includes (not deadly) rhinorrhea, yawning, diarrhea, sweating, dilated pupils, tachycardia and HTN. Tx overdose c naloxone. Tx dependence c abstinence through methadoes titration.
526. Stimulants – amphetamines/cocaine, rapid dependence of tolerance, IVDA risks, paranoid psychosis. Intoxication includes euphoria, alertness, increased energy, anxiety, talkativeness, mydriasis, tactile hallucinations (crawling bugs), HTN and tachycardia. Withdrawal includes (non-deadly) fatigue, hypersomnia, anxiety, dysphoria, suicidal ideation, craving. Tx intoxication symptomatically (antiarrhythmic, benzo for agitation, haloperidol). Tx withdrawal supportively (observe for suicidality). Tx dependence c rehab.
527. Sedatives – benzo/barbs – intoxication causes slurred speech, drowsiness, impaired attention, disinhibition. (Flumetrazepam is the date-rape drug). Overdose c barbs for suicide, (not so much benzo b/c of high therapeutic index, unless taken with another drug or alcohol). Both cause resp depression, coma, death. Withdrawal causes anxiety and insomnia. Severe withdrawal is a medical emergency (n/v, autonomic hyperactivity, photophobia, tremor, hyperthermia, delerium, seizures, death) most severe c short-acting drugs. Overdose benzo c flumazenil (does not reverse resp depression), barbs c charcoal, gastric lavage. Tx barbiturate withdrawal c pentobarbital challenge test to get daily dose, and taper off. Tx benzo withdrawal c long-acting benzo (diazepam, clonazepam) and gradually withdraw.
528. Nicotine – acetylcholine (nicotinic) agonist. Withdrawal causes irritability, wt gain, and difficulty c concentration. Tx: 1st obtain specific date to stop, 2nd educate/counsel.
529. PCP – paranoia, assaultiveness, impulsiveness, vertical and/or horizontal nystagmus (dead give-away), diaphoresis, resp depression, seizures, normal size pupils. Tx symptomatically
530. Hallucinogens – LSD, Ecstacy – sympathomimetic effects (mydriasis, tachycardia, sweating, diarrhea, urination), panic reactions, illusions, paranoia. Later on, pt may not be using drug anymore and reexperience intoxication (flashback).
531. Cannabinoids – Marijuana/THC – intoxication has euphoria, bad judgement, slowed reactions, dry mouth, conjunctival injection (dead give-away). Chronic use causes amotivational syndrome and memory impairment.
532. Hallucination is a disturbed sensory perception (visual, tactile, auditory). Delusion is a fixed, false belief (even if people prove to you otherwise). Psychosis is inability to judge boundary between real and unreal.
533. Schizophrenia – presence of >2 s/s of the following for >6months: delusions, hallucinations (generally auditory, link visual c alcohol withdrawal), disorganized speech/behavior, negative s/s (flat affect, no speech, no motivation, anhedonia). Better prognosis (NBME 3 question) if acute, late onset, good social/occupation hx, positive s/s, medication compliance, married, female gender. Symptoms due to altered dopamine activity (newer antipsychotics affect serotonin also). Negative s/s have enlargement of cerebral ventricles and hypoactive frontal lobe. Tx: 1st assess if pt needs hospitalization (protect self/others), 2nd Antipsychotics (Risperidone), 3rd Psychosocial tx. [Timeline: 6mo = schizophrenia]
534. Delusional (Paranoid) Disorder – persistent, nonbizarre, well-systematized delusion. Erotomanic (on is loved by a famous other, NBME 3 TQ), grandiose (one possesses great talent), jealous (conviction that lover is unfaithful), persecutory (one is conspired against, MC), somatic (one has a physical abnormality like odor). Tx: 1st hospitalization for inability to control suicidal/homicidal impulses or danger a/w delusions, 2nd psychotherapy, 3rd antipsychotics/antidepressants.
535. Schizophreniform – schizophrenia 4days.
540. Major depression disorder (MDD)– 2 of SIGECAPS in >2wks– sleep changes (delayed sleep onset, decreased REM. Note the difference: Anxiety has increased REM latency, depression and narcolepsy have decreased REM latency), interest loss, guilt, energy loss, concentration decreased, appetite (up or down), psychomotor (retardation or agitation), suicidality. Decreased serotonergic activity a/w violence and suicide. Tx: Hospitalize if suicide risk, 2nd Antidepressant (SSRI 1st) for 6-12 months (not that it takes 4-6wks to start effects), 3rd ECT (rapid response in pregnancy, elderly, medically ill), 4th psychotherapy, 5th antipscyhotic + antidepressant for psychotic pts, 5th Phototherapy if depression is seasonal, 6th treat comorbid psychopathology (anxiety, substance abuse, personality d/o, ADHD).
541. Depression vs Bereavement – Depression (mood pervasive/unremitting, constant low self-esteem/worthlessness, suicidal, sustained psychotic s/s, no improvement c treatment, social withdrawal). Bereavement (mood fluctuates, self-reproach regarding deceased, not suicidal, transient visual/auditory hallucinations or deceased, s/s improve c time and usually gone by 6 months, often welcomes social support). It is normal to have an illusion or hallucination about the deceased, but a normal grieving person knows that it is an illusion or hallucination, while an MDD pt thinks its real. Other clues to MDD that are not normal are feeling of worthlessness, suicidality and psychomotor retardation.
542. Bipolar Disorders: Type I is full-blown mania c MDD. Type II is hypomania c MDD. Tx: 1st assess risk of suicide, assaultiveness, dangerous poor judgement. 2nd For acute mania give mood stabilizer (lithium). For depression – modd stabilizer c or w/o antidepressant if necessary.
543. Cyclothymia – numerous hypomanic episodes c depressive episodes for >2yrs. (Cyclo is a psycho, while dysthymia is just depression for >2yrs).
544. Panic Disorder – minutes to hours of unexpected, sudden intense anxiety, dyspnea, parasthesia, CP, fear of dying. A/w agoraphobia (fear of places where escape is difficult such as bridges, public transportation, large crowds, traveling). Tx: 1st If acute, emergent case, give reassurance and benzo (alprazolam, clonazepam). 2nd R/o MI, PE, CVA, hypoglycemia, 3rd Antidepressants (SSRI is tx of choice for long-term management), 4th Cognitive-behavioral therapy (CBT) for agoraphobia.
545. Obsessive-Compulsive Disorder – recurrent intrusive images, impulses, thoughts (obsessions) and ritualistic behaviors (compulsions) that produce anxiety and affect way of life. A/w Tourette syndrome. Abnormality is serotonin system. Tx c SSRIs (fluvoxamine), but if you only see TCA’s pick clomipramine.
546. Specific Phobia – irrational, excessive fear and avoidance of a specific object or situation. Tx: Systemic desensitization.
547. Social Phobia – fear of embarrassment, scrutiny of others (public speaking, eating in public, public bathrooms). Tx: 1st CBT, 2nd BB (propranolol) for stage fright, 3rd Antidepressants (not TCAs) and high-potency benzodiazepines.
548. Posttraumatic Stress Disorder – >1 month, must have 3: reexperiencing (flashbacks), emotional numbing (avoidance), autonomic arousal (insomnia, irritability). Tx: 1st hospitalize for acute suicide, violence risk. 2nd CBT, 3rd Antidepressants.
549. Acute Stress Disorder - 6months. Muscle tension, restlessness, poor concentration, fatiguability, irritability, loss of sleep. Tx: 1st psychotherapy, 2nd Antidepressants (Buspirone).
551. Somatorofrm Disorders – unlike factitious disorder and malingering, the symptoms are not intentionally produced but are strongly linked to psychological factors. Examples include somatization disorder (multiple somatic complaints, tx c regularly scheduled visits c PMD), conversion disorder (neurologic s/s), pain disorder (pain in absence of adequate physical findings, tx c psychotherapy), hypochondriasis (fear of specific disease, tx c regular medical visits), and body dysmorphic disorder (preoccupation c defect in appearance, tx c psychotherapy and SSRI’s after you assess suicide risk).
552. Factitious disorder – “Munchausen syndrome.” Intentional production of s/s for unconscious psychological reasons (need to assume sick role) usually in someone in medical occuption or c history of illness. If s/s produced by parent, this is Munchausen’s by proxy. Tx c psychiatric consult, confrontation may be helful.
553. Malingering – intentional production of symptoms for a recognized gain (money, drugs, avoid work/military/prison).
554. Dissociative Identity disorder – multiple personalities, which take over life and pt may or may not be aware of each other. Tx c intensive psychotherapy.
555. Amnestic Disorder – 2 types: psychogenic fugue (sudden, unexpected travel c amnesia of old identity and assumption of new identity that lasts hours to months, pt is unaware of loss) and psychogenic amnesia (sudden inability to recall important personal information of a traumatic or stressful event, but aware of loss). Recovery usually returns spontaneously. If not, try hynosis, amobarbital or psychotherapy.
556. Depersonalization disorder – recurrent feeling of detachment from one’s body or self (feel like you’re in an outside world).
557. Anorexia Nervosa – must have 3: amenorrhea, minimal normal body weight, fear of gaining weight. Tx: 1st hospitalize for dehydration, starvation, hypotension, electrolyte, hypothermia, suicide risk. 2nd treatment contract for wt gain, 3rd CBT.
558. Bulimia Nervosa – binge eating, normal weight, overconcerned c wt/diet/exercise, self-induced vomiting, laxatives/diuretics, a/w kleptomania. Tx: 1st hospitalize for ECG (hypokalemia-induced arrhythmia is MCCOD), electrolytes, amylase, LFTs, esophageal/gastric rupture, suicide risk. 2nd psychotherapy, nutritional counseling, SSRI for binging (do not give buproprion for risk of seizures).
559. Old, classic USMLE TQ: Mom finds her son having sex c another boy, is this normal or homosexuality? Normal (unless they say he enjoys it). Another TQ is a man, who knows he is a man and likes women, dresses up like a woman and acts like a woman, what is his sexual orientation? Heterosexual (b/c he likes women).
560. Projection – attributing your own wishes to someone else. A/w paranoid personality d/o (p for p – paranoia c projection)
561. Denial – if they deny having a disease, next step is do nothing! (because it usually does not interfere c treatment, but if it does, next step is confront the pt).
562. Splitting – all is good or bad. a/w borderline d/o. If they only say all is good, its idealization. If they only say all is bad, its devaluation. Splitting must have both.
563. Regression – look for h/o bedwetting in a kid >5yo (3.0 is a medical emergency that needs IV saline or hemodialysis.
580. Electroconvulsive therapy – increases serotonin for conditions like MDD, mania and schizophrenia. No absolute contraindications. Only relative CI is high intracranial pressure (brain tumors). Who gets it? Suicidal pt (tx of choice), those who don’t respond to meds, pregnancy, h/o benefit c ECT, medication complications. MC adverse effect is memory loss.
581. Benzodiazepines – all work on CP450 exams OTL (Oxazepam, Temazepam, Lorazepam), so remember OTL for Outside The Liver.
582. Suicide – if pt mentions it, next step is to ask more questions (attempt, ideations), then admit. Risks: h/o att
Reply
#2
emergency orders:
head of bed elevation
PEFR
finger stick blood glucose
pulse ox
oxygen
ekg
iv access
ivf
npo
cardiac monitor
bp monitor
drugs:
morphine
phenargan
tylenol
thiamine
glucose
naloxone

trauma:
airway and C spine
breathing
circulation
and then all emergency orders


labs for both office/ER:
cbc
bmp/cmp/lft
abg
preg test
UA/urine and blood culture
stool studies
iron studies
csf studies
urine drug screen
cardiac enz
lipase/amylase/ldh
esr/RF/ANA
echo
cxr/axr
usg
ct/mri
Pre op tests-PT/INR/PTT and type and cross
tumor markers

admit orders: pneumonic ADCVAANDIMPLE
A-admit to/location change(very impt as we always forget)
Diagnosis
Condition
Vitals every 2/4 hours
Activity-bed rest with BRP or ambulate etc
antibiotics-incl pre op antibiotics
nursing-intake and output/foley/NGT etc
Diet-NPO/Cardiac/diabetic etc
IVF
Meds-Remember to discontinue or keep home meds here
Labs-daily monitoring of labs or labs every 8 hours orpulse ox every hour etc
Extra-counsel, consult, vaccinations esp in kids and elderly, rehab, suicide contract, blood transfusion etc...






























uncomplicated sinus infection amoxicilin 40 -50 mg/kg
septic arthiritis in child nafcilin iv and 3th. Generation cephalosporin
conjuntivitis gonococal ceftriaxone
chlamydia conjuntivitis erytromicin
gestational listeria ampi/genta
meningitis profilaxis rifampine, ceftraiaxone, ciprofloxacine
endometritis 9ana,neg and positive clinda micin, gentamicin
actinomyces oral cavity abscess penicillin
lyme less than 9 y/old amoxicilin 40 -50 mg/kg/dia tid for 21 days
lyme more than 9 years old doxicilin 100 mg bid for 21 days
lyme in children can not take amoxi or doxi cefuroxime or eritromicin 21 days/azytro. Early 1st stage
lyme stage 3 ceftriaxone of penicilin G for 14 - 21 days IV
HIV pneumocistis carini pneumonia trimetropin sulfamethoxazole
RMSF in adults and children doxycycline. Some prefer cloramphenicol for children
rhizopus infection. Mucormicosis amphotericin B plus surgical debridemnt ( both)
sifilis penicilin
sifilis patient allergic to peniciiin tetracicline or doxyxycxline
sifilis patient who is pregnant erytromycin
cat scrach…B. henselae 5 days azytromicin
cat scrach…B. henselae alternate tx; claritro, rifam. Smx,tmp 5 to 7 days
actinomyces israeli in diabetic patient high dose of penicilin or erythromycin 6 - 12 week
chemoprophylaxis of tuberculosis isoniazide plus pyridoxin for 9 months
alternative chemoprophylaxis fo Tb pyrazinamide plus rifampicin for 2 months
alternate chemo tb pt can not tolerate pzn rifampicine alone for 4 months
cytomegalovirus cholitis in HIV patient ganciclovir..alternativae is foscarnet
HIV lung cavity with nocardia (weak acid fast branch) trimetropin sulfamethoxazole
chlamydia in pregnant azytromicin one single dose
gonorrhea in pregnant same as in non pregnant: ceftriaxone plus eritro
HIV candida esophagitis ( disphagia) fluconazol 1- 2 weeks. Resistant strain: itraconzaol
HIV CMV esophagitis ( odynophagia) ganciclovir IV…alternative is forscarnet
ehrlichiosis doxycilin
RMSF in pregnant cloramphenicol
HIV profilaxis MAC if CD4 less than 50 azytromycin or claritromicin.Alt: rifabutin
HIV profilaxis histoplasm. in endemic and lss 100 CD4 itraconazol
febrile neutropenia. Usually pseudomona monotherapy: ceftazidime,cefepime,imipenem, mero
lyme prophylaxis in pregnant oral amoxicilin
gonorrhea pharingeal ciprofloxacin, ofloxacin, ceftriaxone (if not allerg to ceph)
cryptococal meningitidis amphotericine and flucitocine
chlamydia uretritis in men azithromycin (1 dose) or doxycylin
lyme. Early localized doxycilin due to risk of ehrlichiosis concomitat
lyme. Early diseminate or late lyme ceftraixone or cefotaxime
meningitis acute in older than 60 and inmunocompr. ceftraixone + vanco (pneumoresitant) + ampici (listeria)
meningitis in hospitalized or after neurosurgery staph aureus (vancomycin),pseudomona(ceftazidime)
nocardia Tx. and prevention: TMP SMX. Minocilin 2nd. Choice
malaria: general chemoprophylaxis cloroquine
malaria india pakistan banglad resistan to cloroquine mefloquine use as prophylaxis
malaria vivax and oval primaquine
malria treatment in areas resistant to cloroquine fansidar
nosocomial neumonias or neumonias in ventilated cephalosp. 4th gen: cefepime,ceftazidime. Pseudomone
pseudomone nosocomial infection dime, pime, imip/cilastin,cirpofloxa, aztreonam, tobra
nosocomial MRSA vancomycin
nosocomial anaerobs clindamycin
human bite amoxicilin/clavulanate
endocarditis in IV abuser: empiric treatment vancomycin and gentamicin
endocarditis in pat that is not iv abuser: empiric tx nafcilin iv and 3th. Generation cephalosporin
lyme pregnant,lactating or children amoxicilin
mycobacterium avium complex profilax if cd less 40 azytromycin
mycobacterium avium complex treatment claritro y etambutol
limb threating infection/ulcer in diabetic patient empiric:cefotetan or ampi/sub, or clinda+fluorquinolone
limb threating infection/ulcer in diabetic that no heal if empiric tx fail do vano (MRSA) and ticarcilin (pseudom)
babesiosis 2 regimens: quinine/clinda and atovaquone azytromicin
HIV diseminated histoplasmosis iv amphotericin B and lifelong itraconazole
shyphilis in pt allergic to penicilin docycilin or tetraciclin for 14 days
shyphilis pregnant patient allergic to penicilin eritromycin
neuroshypliis IV aqueous crystalline penicillin
pneumonia in cystic fibrosis children(severe=pseudom) penicilin antiseudomone (piperacilin) +aminogluc (genta)
pneumonia in cystic fibrosis children alternative to penicilin is cefalosporine (pime or dime)
pneumonia in adults suspectin pseudomona fluorquinolone: ciprofloxacine can be used istead aminog
pneumonia: cystic fibrosis children (no severeConfusedtaph) antiestaphilococal medication ( pseudomone and stphylo
HIV patient with cryptococal infection in SNC amphotericin B (IV) plus flucytosine….later: fluconazole
influenza a and b oseltamivir and zanamivir
influenza a and b in pt with asthma avoid zanamivir
influenza a amantadina and rimantidina
HIV and hepatitis B together lamivudine, efavirenz, tenofovir
pseudomembranous cholitis plus ileo rectal vancomycin plus oral vanco and iv metron
pneumonia in pt w/ bronchiectasia. Think pseudomone cefepime/dime
RSV infection in children rivabirin
CMV gastric ulcers in transplanted patient ganciclovir valganciclovir
meningitis for streptococcus pneumoniae vancomycin plus ceftriaxone and dexamethasone
african tick bite fever (mc ricketsial infection in human doxycilin
meningitis by listeria ampicilin
meningitis by listeria in pt allergic to penicilin smx. Tmp
necrotizing fasciitis by MRSA vancomycin
tuberculosis latent alternate treatments rifam and piraz for 2 months or rifamp for 4 months
tuberculosis latent . Recommended treatment isoniazide for 9 months
meningitis after neurosurgical procedures staphylo and pseudomone vancomycin and cefepime
brain abscess after otitis media.contiguous spread multibugs: ceftriaxone and metronidazol (empiric tx)
strep or staphilo shock toxic (foreign bodies) vancomicine mrsa and clindamycin
septic arthritis in rheumatic patiet (>90%staph aureus) nafci if sensitive or vanco if mrsna
osteomyelitis in sickle cell. Empiric treatment vancomycne and ceftriaxone (vanco for stap and cef sallmonella)
neuroshyphilis penicilin, if allergic, desensitization is mandatory
pneumonia: community aquiredd. Empiric tx ceftraixone plus azytromicin
gonococal infection diseminated: empiric tx ceftriaxone (pacific rim and hawaii strains are resistant to cirpfloxa
pneumonia by strep neumonia resistant to penicilin fluoroquinolones, also vanco and linezolid.
HIV pat with sudden lost of vision and pain. acute reitnal necrosis: herpez or varicella virus: intravenous acyclovir
cryptococal infection in transplanted patients flucytosine and ampho b are the best choices
candidemias (pt with catehters chronically) caspofungines
dermatophytosis terbinafine
clenched-fist injury of the hand( punch a mouth) ampicilin sulbactam (anaerob strepto, eikenella corrodens) IV
herpes simplex v. infection w/o systemic complication oral valacyclovir
herpez infection complicated with meningitis iv acyclovir
vancomycin resistant enterococus bacteria linezolid: risk of severe trombocitopenia
pyelonephritis empiric treatment in ambulatory pt oral levofloxacin if pt is compliant, if not compliant iv levofloxacin
cystitis uncomplicated tmp.smx, .Avoid tmp/smx in pyelonephritis due to resistance to tmp.
cat bite in a pt allergic to penicilin tmp. Sulfamethoxazole plus clindamycin (covaer past.strep and staph
cholangitis piperacilin/tazo or ampi/sulbact or ticar/clavul
cholangitis in allergic to penicilin patient aztreonam + metro …or cephalosp 3th generation +metronidazol
spontaneous bacterial peritonitis: e coli or strept cefotaxime doc, also piper/tazo or ampi/sulbact
divertriculitis ciprofloxacin plus metronidazol
dermatophytosis miconazol, ketokonazol, ekonazol
dermatophytosis ( nails and hair) itraconaqzol or terbinafine
dermatophytosis (nail and harir ) in children griseofulvine, since itra or terbinafien is not FDA approved for child
lyme: meningitis encephalitis ceftriaxone intravenous
lyme: only facial palsy can be treated with oral doxycilin
lyme; cardiac involvement ceftraiaxone intravenous
lyme arthritis: doxyciclin
brain abscess empiric tx. Cover gram +, gram -, anaero penicilin, cefalosp 3 th generation and metronidazol
brain abscess post traumatic or post neurosurgery has to cover staph aureus:oxa/nafc , if allergy or resistant; vanco
brain abscces in drug abuse or pt with shunts: staph resistant (MRSA) or staph epidermitis: go with vanco as 1s.choice
brain abscess with edema and midline shift beside antibiotic, use manitol and steroids
Reply
#3
Record # 1

Question/Fact:

A 39-year-old Polish man comes to the clinic for painful calves after walking long distances and for discoloration of the fingers with changes in temperature. He says his symptoms started two months ago, and he gets no relief from the ibuprofen. He has previously been healthy. He currently smokes a pack a day and drinks socially. He has no history of drug abuse. On physical examination, his blood pressure is 140/90 mm Hg, heart rate is 68/min, and he is afebrile. Examination of the hands reveals distal digital ischemia and trophic changes in the nails of both hands. Radial pulses are absent bilaterally, but all other pulses are present. His right calf shows evidence of a superficial thrombophlebitis. Laboratory studies show: white cell count 9,600/mm3, hematocrit 38.6%, MCV 89 μm3, ESR 40 mm/h, and C-ANCA as negative. The rheumatoid factor and ANA are negative. Which of the following should be done next for this patient?

(A) Heparin
(B) Prednisone
© Arterial bypass
(D) Cyclophosphamide
(E) Abstention from tobacco

Answer:

(E) Abstention from tobacco

Explanation:

This patient has thromboangiitis obliterans (Buerger's disease), which is an inflammatory occlusive disorder involving small and medium-sized arteries and veins in the distal and upper extremities. The prevalence is highest in men of Eastern European descent under the age of 40. Although the cause is unknown, there is a definite relationship to cigarette smoking and an increased incidence of HLA-B5 and -A9 antigens in patients with this disorder. Clinical features of thromboangiitis obliterans often include a triad of claudication of the affected extremity, Raynaud's phenomenon, and migratory superficial thrombophlebitis. Claudication is confined to the lower calves and feet or forearms and hands because this disorder primarily affects the distal vessels. Hand examination can reveal severe digital ischemia, trophic nail changes, ulceration, and gangrene at the tips of the fingers. Brachial and popliteal pulses are usually present, but radial, ulnar, and/or tibial pulses may be absent. Smooth, tapering, segmental lesions in the distal vessels are present on angiography. The diagnosis can be confirmed by excisional biopsy of an involved vessel. There is no specific treatment, except abstention from tobacco. The prognosis is worse in those who continue to smoke, but results are relatively good in those who stop. C-ANCA antibodies are usually found in Wegener's granulomatosis. Arterial bypass may be indicated in disease confined to larger vessels. The hand abnormalities effectively exclude peripheral vascular disease. If these measures fail, amputation may be required. Cyclophosphamide and prednisone do not help. Again, the management is to stop smoking.
==============

Topic:

Rheumatology

Record # 2

Question/Fact:

A 25-year-old woman with Crohn's disease presents to your office with recurrent abdominal pain and diarrhea. She has been taking mesalamine 4 grams per day for the last year. Last fall, after developing diarrhea and pain, she was placed on prednisone 60 mg daily. She had a complete remission and, after a 3-month tapering of the prednisone, suffered a relapse. Prednisone was restarted 2 months ago at 60 mg daily, and now as the dose has decreased to 20 mg per day, the diarrhea has recurred. She is having 6 to 8 water stools per day, crampy pain, and some weight loss. What would be the best next step?

(A) Restart the prednisone and plan to maintain the dose at 40-60 mg indefinitely
(B) Restart the prednisone with 6-mercaptopurine and plan on prednisone taper in 2 months
© Stop the prednisone and add cyclosporine
(D) Admit to the hospital and give high-dose intravenous steroids to induce remission
(E) Stop the mesalamine and add methotrexate

Answer:

(B) Restart the prednisone with 6-mercaptopurine and plan on prednisone taper in 2 months

Explanation:

Prednisone is effective in treating active Crohn's disease for short durations (3-6 months). Long-term use for maintenance is not indicated. 6-Mercaptopurine and azathioprine are steroid-sparing medications used to limit the need for prednisone. Prednisone, like other corticosteroids, has numerous side effects and should only be used for treating active flares of disease, not maintenance of remission. Cyclosporine and methotrexate have limited roles in the management of Crohn's disease.


Topic:

Gastroenterology

Record # 3

Question/Fact:

A 65-year-old man presents to the emergency room with complaints of weakness, generalized swelling in his extremities, and right leg pain. At the time of presentation, he appears to be in moderate distress from the leg pain. The patient states that his symptoms started two days ago. The patient also has frequent urination and increased thirst. He states that he has felt weak for the past few months. Physical examination reveals a tender, erythematous, and swollen right calf. He also has 2+ pitting edema in all extremities. Blood pressure is 107/55 mm Hg, and temperature is 100.3 F. Venous ultrasound is positive for lower extremity deep vein thrombosis. Laboratory studies reveal:

White cell count 11,000/mm3; hematocrit 32.3%; platelets 105,000/mm3; K 4.0 mEq/L; BUN 24 mg/dL; creatinine 1.7 mg/dL. The PT/PTT are normal.

Total bilirubin 0.4 mg/dL, AST 28 U/L, albumin l.9 g/dL, cholesterol 326 mg/dL; triglycerides 425 mg/dL.

Urine dipstick shows protein 3+, hemoglobin 1+, white cells 1+; 24-hour urine shows 6.2 grams of protein.

What is the next step in the treatment of this patient?

(A) Renal biopsy
(B) Plasmapheresis
© Anticoagulation
(D) Cyclophosphamide
(E) Prednisone

Answer:

© Anticoagulation

Explanation:

This patient has nephrotic syndrome based on the presence of edema, hyperproteinuria, hypoproteinemia, and hyperlipidemia. Such patients are predisposed to developing a hypercoagulable state secondary to the renal losses of proteins C and S and antithrombin III, as well as increased platelet activation. Patients with evidence of venous thrombosis should be anticoagulated for at least 6 months. Recurrent thrombosis and renal vein thrombosis warrant lifelong anticoagulation.

Although he may need a renal biopsy, he needs to have his thrombus treated first as the "next" step. The same is true of using cyclophosphamide and prednisone. This patient most likely has membranous glomerulonephritis simply because he is an adult with nephrotic syndrome, and this is the most common cause in adults. Colonoscopy should also be done in a patient like this because there is a strong association of glomerulonephritis with solid tumors, such as colon and breast cancer.
Topic:

Nephrology

Record # 4

Question/Fact:

A 42-year-old man from Vietnam, who had been a bus driver in Thailand, presents to the emergency department after having shortness of breath while playing soccer with his son this morning. Over the last several months, he has been having several episodes of shortness of breath. Several of the episodes were associated with chest pain. He denies any significant medical history. He has a 25-pack-year use of tobacco, and he has a sedentary lifestyle. His father had a myocardial infarction at the age of 59. His heart rate is 72/min, blood pressure is 140/66 mm Hg, and respiratory rate is 14/min. His examination shows mild jugulovenous distention with a collapsing carotid arterial pulse. His cardiac examination reveals a point of maximal impulse that is displaced laterally and inferiorly and a mild diastolic blowing murmur at the base while he sits up. His sensory examination shows loss of vibration sense in all extremities, and an abnormal Romberg test. EKG shows normal sinus rhythm with left axis deviation and ST-segment depression and T-wave inversion in leads I, aVL, V5, and V6. The chest x-ray shows an enlarged heart with dilatation of the proximal aorta. The CBC, chemistries, and cardiac enzymes are negative. The echocardiogram shows an ejection fraction of 60%. What is the next best step in the management of this patient?
(A) Treat with digitalis
(B) Exercise stress test
© Cardiac catheterization
(D) VDRL and lumbar puncture, followed by penicillin therapy
(E) Aortic valve replacement

Answer:

(D) VDRL and lumbar puncture, followed by penicillin therapy

Explanation:

This patient has a murmur of aortic regurgitation (AR) and an abnormal neurological examination, suggesting syphilis. Therefore, this patient needs a VDRL and a lumbar puncture. Syphilis of the aorta involves the intima of the coronary arteries and may narrow the coronary ostia, leading to myocardial ischemia. There is also destruction of the medial muscle layers of the aorta, leading to aortic dilation. Myocardial ischemia in AR happens because oxygen requirements are elevated secondary to left ventricular (LV) dilatation and elevated LV systolic wall tension. Coronary blood flow is normally during diastole when the diastolic arterial pressure is subnormal. This leads to decreased coronary perfusion pressure.

Nifedipine or ACE inhibitors are only used once the patient develops severe AR. Digoxin is of very limited use at any time. An exercise stress test is not indicated because of the baseline EKG abnormalities. You normally detect the presence of ischemia on a stress test by looking for the development of ST-segment depression. This patient already has baseline ST-segment depression. A thallium or sestamibi scan would be required in a case like this. If you were investigating for ischemia, surgical treatment does not restore normal LV function. Patients with AR and normal LV function are followed until surgery is indicated. This is when the patient has LV dysfunction but before the development of symptomatic congestive failure. Valve replacement is also indicated in asymptomatic patients when the ejection fraction falls to 55 mL/m2. Although catheterization may be useful before surgery, it would not be done before a specific diagnosis of syphilitic aortitis has been confirmed and treatment with penicillin has been given.

Topic:

Cardiology

Record # 5

Question/Fact:

A 40-year-old woman is brought to the emergency department by her daughter who states that she found her mother at home several hours ago, confused, lethargic, and unable to get up from her chair or speak. Her mother has a seizure disorder for which takes an antiseizure medication. She also has a history of alcohol abuse in the remote past. For the past several weeks, her mother has been complaining of difficulty sleeping and anxiety. The patient is stuporous and unresponsive to verbal stimuli. Her blood pressure is 100/60 mm Hg, heart rate is 50/min, and respiratory rate is 9/min. The pupils are pinpoint, and there is horizontal nystagmus. Asterixis is present.

Laboratory examinations reveal: white cell count 9,800/mm3, sodium 150 mEq/L, BUN 18 mg/dL, creatinine 0.9 mg/dL, glucose 50 mg/dL, calcium 5 mg/dL, ammonia 100 μg/dL, albumin 3.0 g/dL, AST 100 U/L, ALT 80 U/L. The urinalysis and lumbar puncture are normal. A CT scan of the brain shows cerebral edema. Arterial blood gas shows a pH of 7.20, a pCO2 of 46 mm Hg, and a pO2 of 79 mm Hg. Osmolar gap is zero. The toxicology screen is negative for benzodiazepines and opioids. What is the most likely substance that this patient overdosed on?

(A) Phenytoin
(B) Carbamazepine
© Valproic acid
(D) Ethanol
(E) Valium

Answer:

© Valproic acid

Explanation:

This patient most likely is intoxicated with valproic acid. This drug is widely used in the management of seizure and mood disorders. Valproic-acid intoxication produces a unique syndrome consisting of hypernatremia, metabolic acidosis, hypocalcemia, elevated serum ammonia, and mild liver aminotransferase elevation. Hypoglycemia may occur as a result of hepatic metabolic dysfunction. Coma with small pupils may be seen, and this can mimic opioid poisoning. Encephalopathy and cerebral edema can occur.

Phenytoin and carbamazepine are also commonly used antiseizure medications. Phenytoin intoxication can occur with only slightly increased doses. The overdose syndrome is usually mild. The most common manifestations are ataxia, nystagmus, and drowsiness. Hepatic encephalopathy would be unusual. Choreoathetoid movements are occasionally seen. Carbamazepine is a first-line agent for temporal lobe epilepsy, as well as trigeminal neuralgia. Intoxication causes drowsiness, stupor, coma, or seizures. However, dilated pupils and tachycardia are more common.

Signs of ethanol intoxication are similar to the signs of anticonvulsant medication. In addition, it causes a high osmolar gap. Valium is an unlikely cause of intoxication because this patient's blood benzodiazepine levels are negative.

Topic:

Poisoning

Record # 6

Question/Fact:

A 52-year-old woman presents to the emergency department with fever, weakness, and abdominal pain for the past three days. It has been associated with nausea and three episodes of vomiting. Her husband states that her temperatures have been as high as 103.5 F and that she has not been herself lately, appearing confused and lethargic. She has a history of hypothyroidism and migraine headaches. She appears lethargic, dehydrated, and is oriented only to person. Her blood pressure is 75/50 mm Hg, temperature is 102.9 F, and pulse is 108/min. She has dry oral mucosa and hyperpigmented areas of her skin spread diffusely over the posterior neck, hands, and knuckles. Rales are heard over the right lower lung field, and the chest x-ray shows a right lower lobe infiltrate. The EKG is normal. The patient is placed on intravenous hydration. Laboratory studies show a white cell count of 6,300/mm3, and the differential shows 82% neutrophils, 7% lymphocytes, and 9% eosinophils. The sodium level is 112 mEq/L, with a potassium of 5.9 mEq/L and a chloride of 92 mEq/L. Bicarbonate level is 20 mg/dL, and BUN is 32 mg/dL. The creatinine level is normal. The glucose level is 60 mg/dL, and the urinalysis is normal. What is the best initial test to diagnose this disorder?

(A) Immediate cortisol and assess ACTH level
(B) Metyrapone stimulation test
© Early morning cortisol
(D) A cosyntropin stimulation test
(E) 24-hour urine cortisol

Answer:

(A) Immediate cortisol and assess ACTH level

Explanation:

In the context of acute adrenal crisis, the most appropriate initial diagnostic test is to obtain a random cortisol level before initiating treatment with intravenous hydrocortisone. In a patient who is hypotensive and hemodynamically unstable, it is inappropriate to perform any diagnostic maneuvers that require several steps to obtain a diagnosis. (The metyrapone stimulation and the cosyntropin stimulation are such tests.) The early-morning cortisol is diagnostically useful if it is very low, which confirms adrenal insufficiency, or very high, which excludes adrenal insufficiency. A 24-hour urine for cortisol is a test used to confirm the diagnosis of the hypersecretion of cortisol, also known as Cushing's syndrome, which is the opposite of adrenal insufficiency.

Topic:

Endocrinology

Record # 7

Question/Fact:

A 45-year-old woman presents to your office after developing a pruritic rash and a fever. She first noticed it on her wrists two weeks ago but states that it has now spread to her feet as well. Her past medical history is significant for a seizure disorder following the removal of a meningioma. She has been treated with Dilantin. Physical examination is significant for icteric sclera. There are polygonal, flat-topped, violaceous papules limited to her wrists and her ankles. A white, reticulated, lacy lesion is also evident on examination of her buccal mucosa. Her liver is enlarged and is nontender to palpation. Laboratory analysis reveals: PT 11 seconds, albumin 3.6 g/dL, alkaline phosphatase 160 U/L, AST 700 U/L, ALT 960 U/L, ANA 1:160. Anti-hepatitis C virus (second generation) is negative; anti-hepatitis-B surface antibody (HBs) is positive; and anti-hepatitis-B core antibody (Hbc)is negative. She has an erythrocyte sedimentation rate of 20 mm/h and a cholesterol of 160 mg/dL. Anti-smooth muscle antibody test is negative, and an ultrasound of the abdomen is normal. What would you do next?

(A) Start prednisone
-2b therapya(B) Initiate interferon-
© Administer N-acetylcysteine
(D) Stop Dilantin
(E) Start methotrexate

Answer:

(D) Stop Dilantin

Explanation:

The patient has Dilantin-induced hepatitis. Drug-induced hepatitis may resemble autoimmune hepatitis, including the presence of hypergammaglobulinemia and positive antinuclear antibodies (ANAs). This can result in a false-positive anti-HCV ELISA test. The liver biopsy confirms the picture of drug-induced cholestatic hepatitis. Prednisone and/or azathioprine are the initial treatments of choice for autoimmune hepatitis. Although this patient had a positive ANA, additional tests, such as anti-smooth muscle antibody and anti-LKM (liver, kidney, microsomes), are needed to confirm the diagnosis of autoimmune hepatitis.

Topic:

Gastroenterology

Record # 8

Question/Fact:

A 28-year-old female comes to the emergency department with a headache and fever. She has not had any recent infections, nor has she been exposed to any drugs. Her medical history is unremarkable. On examination, the patient appears lethargic. Her temperature is 100.5 F, pulse is 100/minute, blood pressure is 130/85 mm Hg, and respirations are 18/min. Her conjunctivae are yellowish, and scattered petechiae are noted on the lower extremities. The liver and spleen are not enlarged.

Laboratory studies show the following results: WBC 12,000/mm3; hematocrit 27%; platelets 14,000/mm3; bilirubin 4.5 mg/dL; direct bilirubin 0.5 mg/dL; BUN 40 mg/dL; creatinine 3.5 mg/dL. PT, fibrinogen, and PTT are all normal. Her peripheral blood smear shows fragmented red blood cells.

What is the most effective treatment for this patient?

(A) Splenectomy
(B) Glucocorticoids
© Plasmapheresis
(D) Intravenous immunoglobulins
(E) Platelet transfusion

Answer:

© Plasmapheresis

Explanation:

This woman has a combination of hemolytic anemia with fragmented RBCs on peripheral smear; thrombocytopenia; fever; neurologic symptoms; and renal dysfunction -- a classic pentad of symptoms that characterizes thrombotic thrombocytopenic purpura (TTP). Approximately 90% of patients will respond to plasmapheresis. Patient should be emergently treated with large-volume plasmapheresis. Sixty to 80 mL/kg of plasma should be removed and replaced with fresh-frozen plasma. Treatment should be continued daily until the patient is in complete remission. Platelet transfusions in patients with TTP are contraindicated and can be associated with acute clinical deterioration. Antiplatelet agents, splenectomy, intravenous immunoglobulin, and immunosuppressive agents have not been of reliable benefit to patients with TTP. Each is less effective than plasmapheresis. Glucocorticoids are useful in patients if plasmapheresis does not work.

Topic:

Hemotology-Oncology

Record # 9

Question/Fact:

A 58-year-old woman comes to your office. She is currently in atrial fibrillation and is asymptomatic. Her rate is 70/min. She denies hypertension, diabetes, and congestive failure. There is no other past medical history. What is the most appropriate management of this patient?

(A) Warfarin and clopidogrel
(B) Heparin followed by warfarin
© Low-molecular-weight heparin
(D) Aspirin (325 mg) daily
(E) Warfarin to maintain an INR of 2 to 3

Answer:

(D) Aspirin (325 mg) daily

Explanation:

This is a young patient who has an episode of atrial fibrillation in the absence of other preexisting conditions. The American College of Chest Physicians has established guidelines for anticoagulation in nonrheumatic atrial fibrillation. Patients with risk factors for the formation of thrombi such as a previous stroke, transient ischemic attack, systemic thromboembolism, left ventricular dysfunction, recent congestive heart failure, systemic hypertension, or diabetes should be placed on warfarin to an INR of 2 to 3. Patients with no risk factors who are younger than 65 years are considered to be low risk and should take one aspirin daily. Aspirin is also suitable for patients with a contraindication to warfarin therapy. The efficacy of other antiplatelet agents has not been proven in patients with atrial fibrillation.

Topic:

Cardiology

Record # 10

Question/Fact:

A 62-year-old man presents to your clinic complaining of four days of dysuria, frequency, and urgency. He feels slightly feverish and has had dull, lower-back pain for the past few months. He has had several episodes of the dysuria over the last several months. Each time he was given antibiotics for one week, and the symptoms resolved. Currently his temperature is 100.4 F. The genital examination is unremarkable, and the digital rectal examination reveals a nontender prostate, which is normal in size and consistency, with no palpable masses. After gentle massage of the prostate, a small amount of purulent discharge is extruded from the urethral meatus. The urine culture grows 100,000 colonies/mL of E. coli. Urine cultures from his prior symptomatic episodes also grew E. coli but only 10,000 colonies/mL. Which of the following is most appropriate?

(A) Cystoscopy
(B) Ciprofloxacin and azithromycin orally once now
© Trimethoprim/sulfamethoxazole for one week
(D) Renal ultrasound
(E) Ciprofloxacin for 4 to 6 weeks

Answer:

(E) Ciprofloxacin for 4 to 6 weeks

Explanation:

This patient has chronic bacterial prostatitis. Chronic prostatitis can present with lower abdominal pain, perineal pain, or low back pain. There is usually no dysuria unless there is accompanying cystitis. On physical examination, the prostate usually feels normal and is nontender. As in this patient, chronic prostatitis may manifest as a recurrent urinary tract infection (UTI). The key to the diagnosis is culture of urine or urethral discharge. Pathogens for chronic prostatitis in older men are the same as for a UTI, with E. coli being the most common organism identified. One may extrude purulent discharge by massaging the prostate, which will grow the offending organism. One can also culture the urine post massage of the prostate, which should grow ten times more colonies than premassage urine. This patient cultured 10,000 colonies of E. coli in prior cultures, and currently he grew 100,000 colonies postprostatic massage. Ciprofloxacin for 7 days would be appropriate treatment if this were just a UTI. Therapy for one week is not long enough to clear chronic bacterial prostatitis. Most antibiotics don't have good penetration into the prostate, and it takes at least four weeks of therapy with ciprofloxacin to clear the infection. Ciprofloxacin and azithromycin for a single dose would be the treatment for urethritis. This patient does have a urethral discharge, which may be confused with urethritis. However, since the discharge is extruded only on palpation of the prostate, this strongly suggests that the prostate is the source of infection. Cystoscopy would be useful in a patient with recurrent UTIs in whom you suspected a structural malformation of the genitourinary tract. This patient's UTIs are originating from his chronically infected prostate. Trimethoprim/sulfamethoxazole for 12 weeks is an acceptable alternative for treating chronic prostatitis.

Topic:

Infectious Diseases

Record # 11

Question/Fact:

A 29-year old man comes to your office for a routine visit. His only complaint is leg pain after walking a three-block distance. He states that six months ago he was able to walk a longer distance without having to stop. His father died of a heart attack at the age of 44. His mother had diabetes mellitus, and she too died of a heart attack at the age of 47. His older brother, who is now 35 years old, had a stroke and underwent a carotid endarterectomy last year.

The patient presents as a thin individual with a blood pressure of 135/70 mm Hg and a heart rate of 78/min. Physical examination findings are remarkable for the presence of multiple xanthelasmas on the face, chest, and upper back. There is bilateral, irregular, firm, and nodular thickening in the Achilles tendons and extensor tendons of the hands. This patient's medications include atorvastatin, gemfibrozil at maximum doses, and niacin, which was added to the regimen six months ago. He is maintaining a fat-free diet and exercises regularly. Laboratory test results show: total cholesterol 815 mg/dL, triglycerides 515 mg/dL, and HDL 55 mg/dL. The level of total cholesterol has increased by 15% since the last visit.

What would you recommend to this patient?

(A) Nutritionist consult
(B) Stress test for detection of silent ischemia
© Plasmapheresis
(D) Liver transplantation
(E) Increase the dose of statins as long as transaminases are within the normal range

Answer:

© Plasmapheresis

Explanation:

This patient presents with familial hypercholesterolemia (FH), which is a common autosomal dominant disorder due to absent or defective LDL receptors and resulting in a decreased capacity to remove plasma LDL. LDL cholesterol levels are markedly increased. It is associated with characteristic xanthomas in the Achilles, patellar, and extensor tendons of the hands and by the presence of xanthelasma. Corneal arcus is frequently seen. It is frequently associated with early coronary artery disease (CAD), peripheral vascular disease, and cerebral vascular disease. The plasma cholesterol level is generally in the range of 300 to 500 mg/dL, and in some patients homozygous for FH, it can exceed 800 to 1,000 mg/dL. Triglyceride levels are usually normal, but in 10% of patients, they may be mildly elevated.

Because of the risk of CAD, these patients need especially vigorous therapy. A low-fat and low-cholesterol diet should be initiated, although it gives only a moderate result and will not be enough to control the problem by itself. Effective therapy can be achieved with HMG-CoA reductase inhibitors (statins) as first-line therapy. They lower LDL by 20 to 45%. When they are combined with a bile acid-binding resin, levels of LDL may be decreased by 50 to 60%. In some patients, triple therapy with a statin, a bile acid-binding resin, and niacin may be necessary. Patients homozygous for FH may not be responsive to these measures. For them, measures such as plasmapheresis or LDL apheresis are indicated. Liver transplant is the last resort when all else fails as treatment.

This patient is already on maximum doses of statins and bile acid-binding agent. The addition of niacin did not help. There is very little chance that any additional medical therapy will solve this patient's problem; that is why plasmapheresis is indicated.



Topic:

Cardiology

Record # 12

Question/Fact:

A 37-year-old, HIV-positive man comes for evaluation of generalized weakness, diffuse muscle pain, and frequent headaches that began eight weeks after the start of new HIV medications. He has never had any symptoms from his HIV infection, and he has a CD4 of 255/μL and an HIV RNA viral load of 25,000 (by PCR). He was recently started on zidovudine, lamivudine, and ritonavir/lopinavir. His past medical history is significant for hypertension and hypercholesterolemia. His medications include simvastatin and metoprolol. His physical examination is significant for diffuse muscle tenderness of the extremities. The range of motion is decreased because of pain with movement. His potassium level is 5.4 mEq/L, serum bicarbonate is 16 mEq/L, BUN is 35 mg/dL, creatinine is 1.6 mg/dL, and his viral load is RNA 40,000. The genotyping test result is pending. What will you do while waiting for this result?

(A) Switch zidovudine and lamivudine to didanosine and stavudine, and continue ritonavir
(B) Switch zidovudine, lamivudine, and ritonavir/lopinavir to didanosine, stavudine, and indinavir, and stop simvastatin
© Continue all medications but stop simvastatin
(D) Continue zidovudine and lamivudine, and switch ritonavir/lopinavir to efavirenz
(E) Switch to didanosine, stavudine, and efavirenz, and stop simvastatin

Answer:

(E) Switch to didanosine, stavudine, and efavirenz, and stop simvastatin

Explanation:

This patient presents with a drug interaction between the protease inhibitors and the HMG-CoA reductase inhibitor. In this case, it is with ritonavir and simvastatin. This can produce significant toxicity from the statin. Ritonavir can increase the serum concentration of simvastatin, causing severe myalgias, rhabdomyolysis, and potential renal insufficiency. The next necessary step is to stop simvastatin or change the protease inhibitor to a non-nucleoside reverse-transcriptase inhibitor, such as efavirenz. However, in this case, the patient also presents with failure to achieve a reduction in HIV viral load of 1 log after eight weeks of therapy. In the event of inadequate treatment of HIV infection, the best choice would be to start two new nucleoside reverse-transcriptase inhibitors (NRTIs) and use efavirenz instead of ritonavir, in addition to discontinuing the simvastatin. It is not enough to change ritonavir to indinavir because high-level cross-resistance is very likely. Genotyping guides the therapeutic choice of all treatment failures. The best thing to do when treatment is insufficient is to use as least two, and preferably three, new drugs.

Topic:

Infectious Diseases

Record # 13

Question/Fact:

A 55-year-old man presents with abdominal pain and diarrhea for the past 3 months. He has also noticed a weight loss of 10 lb during this period. He denies nausea, vomiting, melena, or hematochezia. He consumes five to six beers each weekend, smokes half a pack of cigarettes a day, but has never used intravenous drugs. The past medical history is significant for osteoarthritis, newly diagnosed diabetes on a trial diet for 2 months, and recurrent duodenal ulcers found on four separate upper endoscopies. He takes diclofenac/misoprostol and famotidine 40 mg bid. Three years ago, he had taken triple antibiotics to treat H. pylori. He also tells you that tumors run in his family. His vital signs are normal. Physical examination is significant for mild epigastric tenderness to deep palpation without radiation. Routine labs ordered show: WBC 8,500/mm3, hemoglobin 13.4 g/dL, hematocrit 40.1%, platelets 256,000/mm3, amylase 155 U/L, sodium 141 mEq/L, potassium 4.2 mEq/L, chloride 106 mEq/L, CO2 23 mm Hg, BUN 15 mg/dL, creatinine 1.0 mg/dL, glucose 188 mg/dL, and calcium 11.2 mg/dL (elevated). What test would you order next?

(A) Serum lipase
(B) Upper endoscopy with biopsy
© Abdominal ultrasound
(D) Fasting serum gastrin level
(E) Liver enzyme studies

Answer:

(D) Fasting serum gastrin level

Explanation:

This patient's history of "tumors in the family" is consistent with MEN-1 (hyperparathyroidism, gastrinomas, and pituitary tumors). He presents with symptoms of gastrinoma, such as recurrent ulcer refractory to multiple treatments (H. pylori regimen and high-dose H2 blockers) and diarrhea. He also has an incidental hypercalcemia most likely secondary to his underlying diagnosis of MEN-1. The diagnosis of gastrinoma requires the demonstration of fasting hypergastrinemia and an increased basal gastric output.

Topic:

Gastroenterology

Record # 14

Question/Fact:

A slim, healthy 30-year-old woman is scheduled for a dental prosthodontic procedure and was sent for medical evaluation of a known history of mitral valve prolapse (MVP). The patient is a highly active individual and denies palpitations, chest pain, or shortness of breath. She admits to having a family history of heart disease, notably her father, who had died of a heart attack in his forties, and her mother, who had mitral valve prolapse. On physical examination, the patient is comfortable and has normal vital signs. Auscultation of the heart reveals a normal S1 and S2 and a prominent midsystolic click, which is accentuated in the standing position. No systolic murmur is appreciated. What is your overall assessment and plan for this patient?

(A) Get an echocardiogram to evaluate mitral valve motion and blood flow prior to clearing her for the procedure
(B) Prescribe empiric antibiotics for endocarditis prophylaxis and clear her for the procedure
© Get a cardiology consultation prior to medical clearance because the patient has a significant family history of heart disease
(D) Clear her for the procedure without endocarditis prophylaxis
(E) Clear her for the procedure with endocarditis prophylaxis

Answer:

(D) Clear her for the procedure without endocarditis prophylaxis

Explanation:

Mitral valve prolapse (MVP) is a commonly diagnosed valvular disorder affecting women more often than men in a 3:1 ratio. MVP is most commonly diagnosed in people between the ages of 20 and 40. Most people have no presenting symptoms. There is myxomatous degeneration of the valve leaflets, resulting in a stretching of the leaflets and chordae tendinae. Because of the disproportionate size of the left ventricle and mitral valve, there is uneven closure of the valve during each heartbeat and subsequent prolapse of the leaflets into the left atrium. The prolapse is similar to the opening of a parachute. The prolapse causes the classic mid-to-late systolic click. If there is regurgitation of blood back into the atrium, an apical systolic murmur can often be appreciated upon auscultation.

This patient is generally healthy and has a known history of MVP. On examination, she is found to have the midsystolic click but no systolic murmur. The lack of a murmur indicates that blood is not being regurgitated into the atrium. In this setting, the patient does not require antibiotics for endocarditis prophylaxis prior to the dental procedure. Prophylaxis for patients with MVP is recommended if a murmur is present or if evidence of nontrivial mitral regurgitation is found on the echocardiogram. Because the patient has a known history of MVP, she would not require a cardiology consultation or echocardiogram to reconfirm the diagnosis. In fact, an echocardiogram is not a required study to diagnose MVP because dynamic auscultation can be more reliable. Furthermore, the fact that she has remained symptom- and complaint-free would indicate that her condition is stable, and so no study should be warranted at this time. Besides all this, dental prosthodontic procedures do not need antibiotic prophylaxis.

Topic:

Cardiology

Record # 15

Question/Fact:

A 40-year-old man comes to the office because of pain in his right knee for the past three days. The patient denies fever, vomiting, or dysuria. He has no history of trauma but admits to prior episodes of pain, especially after binge drinking. It usually occurs in the knee, ankle, or big toe and is relieved somewhat by ibuprofen. He takes no medications and has no allergies. He has a 25-pack-year smoking history and drinks about half a case of beer when hanging out with friends. His mother developed the same symptoms at the age of 50. On examination, the right knee appears swollen, red, and tender to palpation and has a limited range of motion. You decide to aspirate the knee joint. Which of the following is most consistent with his diagnosis?

(A) Positively birefringent, rhomboid-shaped crystals and 200 white cells/μL
(B) Bipyramidal crystals and 2,000 white cells/mL
© Negatively birefringent, rhomboid-shaped crystals and 20,000 white cells/μL
(D) Cloudy and watery fluid with weakly positive birefringent crystals and 20,000 white cells/μL
(E) Watery fluid with strongly negative birefringent crystals and 20,000 white cells/μL

Answer:

(E) Watery fluid with strongly negative birefringent crystals and 20,000 white cells/μL

Explanation:

Gout is a metabolic disease that most often occurs in men at middle age or older. It rarely occurs in women until they are postmenopausal. The acute gouty episode typically happens at night and is brought on by excessive alcohol use, trauma, surgery, dietary excess, or glucocorticoid withdrawal. The joint fluid aspirate appears cloudy because of the numerous white cells. They typically range in number from 5,000 to 50,000/μL. The cell count in this range can be found in any kind of inflammatory arthritis, such as gout, pseudogout, or rheumatoid arthritis. Crystal analysis is required to distinguish them. Gout will have negatively birefringent, needle-shaped crystals, whereas pseudogout will have weakly positive, rhomboid-shaped crystals. Rheumatoid arthritis should have no crystals. Septic arthritis from infection usually gives >50,000/μL white cells in the synovial fluid. The inflammatory process causes breakdown of hyaluronate in the joint fluid and makes it become watery.

Topic:

Rheumatology

Record # 16

Question/Fact:

A 35-year-old man comes to the hospital after an episode of syncope. There were no preceding symptoms, and the patient recovered rapidly and completely with no residual effects. The patient did not have seizure activity during the episode. There is no history of heart disease and no previous episodes of syncope. The patient lives in rural Connecticut. His only previous medical problem was bilateral facial palsy several months ago. Currently, the physical examination is normal, except for a heart rate of 52/min. His blood pressure is normal. An EKG shows a sinus rhythm with Mobitz II second-degree heart block with a PR interval of 0.34 seconds. Echocardiogram is normal. He has a positive VDRL and a negative FTA. What is the most appropriate management of this patient?

(A) Doxycycline in addition to electrophysiological studies
(B) Ceftriaxone in addition to pacemaker
© Ceftriaxone in addition to prednisone
(D) Ceftriaxone
(E) Doxycycline in addition to permanent pacemaker

Answer:

(B) Ceftriaxone in addition to pacemaker

Explanation:

This patient seems to have second-degree heart block secondary to Lyme disease. He lives in Connecticut, which is an endemic area. (The city of Lyme is in Connecticut.) Facial palsy is the most common neurological manifestation of Lyme disease. The false positive VDRL is characteristic as well. Besides, the patient is very young and has no other reason to have heart block, such as ischemic heart disease. In Lyme disease, high-grade AV block with a PR interval of >0.3 seconds is an indication for intravenous therapy with either ceftriaxone or penicillin. A pacemaker should be placed at least temporarily in those with a Mobitz II heart block because of the risk of progressing on to third-degree block. This patient is also severely symptomatic from his heart block and has had syncope. Prednisone was used in the past but is inferior to an antibiotic alone. Steroids would only be used in those for whom the heart block does not improve with antibiotics. More minor forms of Lyme disease can treated with oral doxycycline. Doxycycline can be used with those who have just the rash, joint symptoms, facial palsy, or first-degree heart block.

Topic:

Cardiology

Record # 17

Question/Fact:

A 21-year-old man with no significant past medical history presents to office with complaints of blood in his urine and mucosal bleeding while brushing his teeth. The patient complains of intermittent "ringing in the ears." He denies any drug or alcohol use. He has no family history of bleeding disorders. Petechiae are noted in the oral cavity, as is dried blood in the nostrils.

Laboratory studies show the following:

Hematocrit 32%; white blood cell count 8,000/mm3 with 60% neutrophils; platelet count 13,000; PT 13 seconds; PTT 28 seconds; LDH 1,200 U/L; elevated indirect bilirubin.

Coombs' test is positive; abdominal examination is normal; and the peripheral smear shows spherocytes.

What is the most likely diagnosis?

(A) Alport's syndrome
(B) Bernard-Soulier syndrome
© Felty's syndrome
(D) Thrombotic thrombocytopenic purpura
(E) Evans' syndrome
(F) Idiopathic thrombocytopenic purpura (ITP)

Answer:

(E) Evans' syndrome

Explanation:

Evans' syndrome is the association of autoimmune hemolysis with autoimmune thrombocytopenia. It is treated initially with steroids and may occasionally need splenectomy to control the disease.

Alport's syndrome is the congenital association of glomerulonephritis with sensorineural hearing loss and ocular problems.

Bernard-Soulier syndrome is a functional platelet disorder presenting with platelet-related bleeding with a normal platelet count.

Felty's syndrome is the association of rheumatoid arthritis with neutropenia and splenomegaly. It is occasionally associated with thrombocytopenia. This patient has no history of rheumatoid arthritis, and the spleen and neutrophil count are normal.

ITP would not give the evidence of hemolysis that is present here, such as an increased bilirubin, positive Coombs' test, high LDH, or anemia. This patient does not have the renal failure or fever associated with TTP. In addition, TTP should give fragmented red cells on peripheral smear.

Topic:

Hemotology-Oncology

Record # 18

Question/Fact:

A 31-year-old woman presents to the emergency department with three hours of shortness of breath. She had been walking her dog this afternoon and had not been outside for more than a few minutes before she began to feel chest tightness, wheezing, and a cough. She has not had any relief from her bronchodilators or steroid inhalers that she uses daily. She states that her daily activities have become affected by frequent episodes of shortness of breath that recur a few times during each week. These attacks can last days at a time, and she is afraid that her current medications are no longer of assistance to her. On physical examination, she has a temperature of 98.8 F, a pulse of 98/min, a blood pressure of 136/90 mm Hg, and a respiratory rate of 23/min. There is some evidence of hyperemia and secretions in the nasal passages bilaterally. She is using her accessory muscles to breathe, and wheezing is audible. Pulmonary function testing reveals an FEV1 of 68% of predicted, with a reduced FEV1/FVC ratio. This increases by 14% after high-dose bronchodilators are administered. Her peak expiratory flow was 158 L/min before bronchodilators were given. Arterial blood gases on room air are: pH 7.36, pCO2 48 mm Hg, and pO2 60 mm Hg. Chest x-ray shows evidence of hyperinflated lungs. The severity of this patient's clinical condition corresponds with which of the following classifications of asthma?

(A) Moderate intermittent
(B) Severe intermittent
© Mild persistent
(D) Moderate persistent
(E) Severe persistent

Answer:

(D) Moderate persistent

Explanation:

This patient presents with an acute attack of asthma, likely precipitated by allergens from the environment. Her symptoms are suggestive of moderate persistent asthma, as she requires the daily use of an inhaled short-acting β2-agonist, the exacerbations are affecting her daily activities, and they recur at a frequency of more than twice per week, lasting days at a time. Other parameters consistent with moderate persistent asthma are the occurrence of nocturnal symptoms more than once per week. Her FEV1 value of 68% is consistent with the criteria for the FEV1 to fall between 60 and 80% of predicted, a reduced ratio of FEV1/FVC to 80%
2. Mild persistent -- symptoms greater than 2×/week but less than l×/day with FEV1 >80%
3. Moderate persistent -- daily symptoms greater than 2×/week with FEV1 >60 and 90%. Tissue examination is far more sensitive than pleural-fluid culture. A PPD would be completely nonspecific in a man from Russia who would almost certainly have a positive test anyway. Adenosine deaminase is elevated in third-space fluid collections from tuberculosis, such as the pleural, peritoneum, and pericardium. It is not as sensitive or specific as the pleural biopsy.

Topic:

Pulmonary

Record # 72

Question/Fact:

A 56-year-old Caucasian woman presents to your office complaining of progressive, right upper quadrant abdominal pain of one week's duration. The pain is accompanied by nausea, fatigue, joint pain, and dyspnea on exertion over the past month. She has a past medical history of type 2 diabetes mellitus and chronic renal insufficiency. She denies fever, chills, or diarrhea. On physical examination she appears thin and has a bronze coloration of her skin. Vital signs are normal. Examination of her neck reveals a steadily rising jugular venous pressure during inspiration. Cardiac examination is remarkable for an S3 gallop. The lungs are clear to auscultation, and the liver is nontender and palpable 3 cm below the costal margin. She has a bilateral 1+ pitting edema of the extremities. Chest x-ray reveals pulmonary congestion with an enlarged heart. The EKG shows a normal sinus rhythm with a rate of 86/min, nonspecific ST-T wave abnormalities, and low QRS voltage in all leads. Echocardiogram reveals mildly reduced left ventricular function. Laboratory studies show: amylase 34 U/L, AST 98 U/L, ALT 60 U/L, total bilirubin 1.0 mg/dL, direct bilirubin 0.2 mg/dL, and glucose 260 mg/dL. What would be the next appropriate diagnostic test to order?

(A) CT scan of the chest to evaluate the pericardium
(B) Ferritin, total iron-binding capacity (TIBC)
© Persantine thallium test
(D) Cardiac catheterization

Answer:

(B) Ferritin, total iron-binding capacity (TIBC)

Explanation:

This patient most likely has cardiomyopathy from hemochromatosis. The patient is presenting with signs of both liver dysfunction and congestive heart failure. Her liver is enlarged but nontender with a mild elevation in her transaminases. In addition, she has signs of biventricular cardiac dysfunction. The signs of right-sided heart failure, beside the dyspnea, are the edema and an elevation of jugular venous pressure on inspiration, or Kussmaul's sign. The left ventricle has diminished function on the echocardiogram, as well as an S3 gallop, and congestion is seen on the chest x-ray. Hemochromatosis leads to the accumulation of iron in the liver, heart, joints, pancreas, pituitary gland, skin, and gonads. Although hemochromatosis leads to restrictive cardiomyopathy, the most common cause of death is from cirrhosis and hepatic carcinoma. Iron also accumulates in the pancreas, leading to diabetes. The term "bronze diabetes" is highly suggestive of hemochromatosis.

Although a CT scan of the chest can be used to detect constrictive pericarditis, this disorder would not lead to diminished left ventricular dysfunction on the echocardiogram. A Persantine thallium test is excellent for detecting ischemia in a patient who is unable to exercise. The information obtained by this test in a patient with cardiomyopathy is nonspecific. Cardiac catheterization for biopsy is not necessary because other tests of iron stores are far less invasive and strongly suggest the diagnosis; therefore, the ferritin and total iron-binding capacity (TIBC) are most useful to order next. You should expect to see a transferring saturation rate of >50% and an elevated serum ferritin. A liver biopsy looking for iron stores would be the most specific test.

Topic:

Cardiology

Record # 73

Question/Fact:

A 28-year-old woman comes in for routine management of her hypothyroidism, which has been controlled with levothyroxine 100 μg per day. She does not use either alcohol or tobacco products. She feels fine but thinks she is pregnant. The physical examination is unremarkable, and the urine pregnancy test is positive. What is the next best step in the management of this patient?

(A) Increase her levothyroxine to 150 μg/day
(B) Check the free T4 and thyroid-stimulating hormone (TSH) levels
© Maintain the same dose of levothyroxine throughout the pregnancy
(D) Decrease the levothyroxine to 50 μg/day

Answer:

(B) Check the free T4 and thyroid-stimulating hormone (TSH) levels

Explanation:

This question focuses on the relationship between hypothyroidism and pregnancy. When a woman with hypothyroidism becomes pregnant, her requirement for the hormone increases. Usually, one may have to increase the dose of levothyroxine by 50%. You should then adjust it according to the levels of thyroid-stimulating hormone (TSH). You should first check a baseline level of the free T4 and TSH in order to know where you stand at baseline to guide your therapy. In pregnancy, there is an increase in the level of thyroid-binding globulin, which can artificially elevate the level of T4; this is why we should use the free T4 level to evaluate the true function of the gland. Postpartum, all dose requirements usually return to their prepregnancy levels.

Topic:

Endocrinology

Record # 74

Question/Fact:

A 30-year-old woman comes to the clinic because of an inflamed and painful right eye. She states that she was fine until three days ago, when she noticed that her vision was blurry. She usually wears contact lenses and thought they may be the source of her complaints. Physical examination shows a very teary, uncomfortable person. There is some inflammation of the conjunctiva and chemosis. Fluorescein staining reveals a corneal dendritic ulcer. What is the most appropriate therapy?

(A) Topical steroids
(B) Switch brands of contact lens cleaning solution
© Systemic steroids
(D) Topical trifluridine
(E) Topical polymyxin

Answer:

(D) Topical trifluridine

Explanation:

This patient presents with a dendritic ulcer, which is almost always caused by a herpes infection. Sometimes they can be caused by corneal abrasions or excessive use of contact lenses. If this were simply a reaction to the contact lens solution, there would not be a dendritic pattern visible on fluorescein staining. A herpetic, dendritic ulcer usually presents with a painful eye, visual blurring, and conjunctival inflammation. The treatment is with ophthalmic trifluridine and acyclovir for about 10 days. Corticosteroids are absolutely contraindicated because they may worsen it. These patients should be referred to an ophthalmologist.

Topic:

Infectious Diseases

Record # 75

Question/Fact:

A 38-year-old, HIV-positive woman finds out that someone at her workplace has tuberculosis. Her PPD at employee health is negative. Her chest x-ray is now normal, and her PPD last year was negative. What should you do next for her?

(A) Nothing further is required
(B) Repeat the PPD in three months
© Start isoniazid and stop in three months if the tuberculosis skin test (PPD) is negative
(D) Start isoniazid for a full nine months
(E) Yearly chest x-rays

Answer:

(D) Start isoniazid for a full nine months

Explanation:

If a patient has been exposed to tuberculosis (TB), and the tuberculosis skin test (PPD) is negative, you do not have to do a chest x-ray. The PPD should be repeated at three months. Most patients who have been exposed to TB and who are going to develop a positive PPD will do so within three months of the exposure, and the PPD should be repeated at 12 weeks. So, she should have the repeat test. However, it is more important to start treatment with isoniazid. There is no reason to give isoniazid to most patients if the PPD is negative. The only exception to this rule is in small children under two years of age who can have a serious exposure but still have a negative PPD because of immature T cells. The other exception is an HIV-positive patient with profound immunosupression and a very low T-cell count who has a serious exposure, such as someone in their own household. There is no routine indication for yearly chest x-rays in any population. Any person with a newly positive PPD should get a chest x-ray. This person's test, however, was negative.

Topic:

Pulmonary

Record # 76

Question/Fact:

A 50-year-old man comes in for management of gout that was originally diagnosed six months ago He has gouty attacks approximately once a month in his left great toe. He was started on daily colchicine several months ago and was told to avoid high-purine foods. He has a history of insulin-dependent diabetes for 20 years, which has been well maintained. Laboratory studies show: BUN 42 mg/dL; creatinine 2.6 mg/dL; uric acid level 8 mg/dL (normal 2.5-7.5 mg/dL) one month ago. What additional medication may be indicated in light of his history?

(A) Probenecid
(B) Allopurinol
© Methotrexate
(D) Ibuprofen
(E) Prednisone

Answer:

(B) Allopurinol

Explanation:

Allopurinol is a xanthine-oxidase inhibitor that promptly lowers plasma-urate and urinary uric-acid concentrations. It is of special value in uric acid overproducers, tophaceous gout, and in patients unresponsive to uricosuric agents. Allopurinol is also helpful in patients with uric-acid renal stones. It should be used in low doses in patients with renal insufficiency. The patient is already on low-dose colchicine on a daily basis. He should be continued on this medicine because in an older patient with occasional attacks, this may be all that is needed to prevent recurrences. Colchicine should be used once a day in moderate renal insufficiency, such as in this patient's case.

Probenecid is a uricosuric drug that is contraindicated in this patient because the creatinine level is elevated above 2 mg/dL. Probenecid and sulfinpyrazone are two uricosuric drugs that may be used with normal renal function. NSAIDs are used for the treatment of an acute attack and are relatively contraindicated with impaired renal function, as in this case. Steroids should be used in the case of an acute attack for those patients unable to take NSAIDs. Methotrexate does not help at all.

Topic:

Rheumatology

Record # 77

Question/Fact:

A 29-year-old man comes to see you because of difficulty with drooling and a unilateral dry eye. On physical examination, he has unilateral facial palsy. He lives in Massachusetts and frequently goes trekking in the mountains. His serologic test is positive for an IgM antibody to Borrelia burgdorferi. What is the most appropriate management?

(A) Repeat the serology in four weeks
(B) Perform a lumbar puncture
© Oral doxycycline for three weeks
(D) Intravenous ceftriaxone

Answer:

© Oral doxycycline for three weeks

Explanation:

The patient has facial palsy because of Lyme disease. Facial palsy is adequately treated with oral doxycycline. The positive IgM antibody test for Borrelia burgdorferi has sufficient specificity in this case to indicate the need for therapy. A repeat test is not necessary. In the absence of other neurologic abnormalities, a lumbar puncture is not necessary. Intravenous therapy does not give a greater efficacy when compared with oral doxycycline.

Topic:

Infectious Diseases

Record # 78

Question/Fact:

A 62-year-old woman with a 10-week history of rheumatoid arthritis presents with persistent pain and swelling of her hands and knees. She also has generalized fatigue and weakness. She reports a mild improvement of her symptoms after starting rofecoxib, prednisone, and physical therapy but still has more than 1 hour of stiffness upon awakening each morning. She has a history of macular degeneration and peptic ulcer disease. On physical examination, she has tenderness and soft-tissue proliferation of the proximal interphalangeal and metacarpophalangeal joints. This is symmetrical bilaterally, with limited flexion and extension of both wrists. There is fluid in each knee and soft-tissue swelling. Laboratory tests show a hemoglobin concentration of 10.2 g/dL, and the erythrocyte sedimentation rate is 45 mm/h. Kidney and liver function tests are normal. What therapy should be started in this patient?

(A) Naproxen
(B) Methotrexate
© Hydroxychloroquine
(D) Infliximab
(E) Intra-articular glucocorticoids

Answer:

(B) Methotrexate

Explanation:

The first step in medical management of rheumatoid arthritis is the use of aspirin and other nonsteroidal anti-inflammatory drugs (NSAIDs) to control the symptoms and signs of the local inflammatory process. COX-2-specific inhibitors, such as celecoxib and rofecoxib, suppress the signs and symptoms of rheumatoid arthritis as effectively as does classic COX-nonspecific NSAIDs, such as naproxen or ibuprofen, but are associated with a significantly reduced incidence of gastrointestinal ulceration. This suggests that COX-2-specific inhibitors might be considered instead of classic COX-nonspecific NSAIDs in persons with an increased risk of NSAID-induced upper gastrointestinal side effects, such as those with a history of peptic ulcer disease and persons receiving glucocorticoids or anticoagulants. The patient in this question has a history of peptic ulcer disease, and that is why she is on rofecoxib.

Disease-modifying agents appear to have the capacity to decrease elevated levels of acute-phase reactants and, therefore, are thought to modify the inflammatory component of rheumatoid arthritis. Most rheumatologists recommend the use of methotrexate as the initial DMARD in individuals with evidence of aggressive rheumatoid arthritis. Aggressive disease is characterized by fever, weight loss, or joint erosions on x-rays. Cyclophosphamide appears to be no more effective than other DMARDs and causes a variety of toxic side effects. Cyclophosphamide also appears to predispose the patient to the development of a malignant neoplasm. It is used only for patients who have clearly failed therapy with the other DMARDs (i.e., methotrexate or hydroxychloroquine). Infliximab is also remarkably effective at controlling the signs and symptoms of rheumatoid arthritis in patients who failed DMARD therapy, and it has far less adverse effects than cyclophosphamide. Intra-articular glucocorticoids can often provide transient symptomatic relief when systemic medical therapy has failed to resolve inflammation. Hydroxychloroquine can be used as a DMARD as well. It should not be used in this specific patient because she has severe disease and a history of macular degeneration. Hydroxychloroquine can cause additional retinal lesions.

Topic:

Rheumatology

Record # 79

Question/Fact:

A 76-year-old woman comes to the emergency department after falling in her house. The daughter witnessed the episode and states that her mother fell on her left side but did not pass out or hit her head. Her mother had a hard time getting to her feet and had to be helped up. The patient denies dizziness, palpitations, or loss of consciousness. She also has hypertension, diabetes with gastroparesis, and peripheral vascular disease. She has had occasional urinary incontinence and memory loss. Her medications are aspirin, atenolol, lisinopril, glyburide, metformin, metoclopramide, and cilostazol. The patient is alert but has decreased concentration and a markedly impaired memory. She has normal language function. The cranial nerves are intact, and there is increased tone in the lower extremities bilaterally with 4/5 motor strength in all muscle groups. The deep-tendon reflexes and cerebellar function are normal. She has a broad-based stance, hesitant initiation of walking, and a shuffling, ataxic gait. What is the next step in the management of this patient?

(A) Discontinue metoclopramide
(B) CT scan of the head
© Lumbar puncture
(D) Start levodopa/carbidopa (Sinemet)
(E) Donepezil
(F) Ventriculo-peritoneal shunt

Answer:

(B) CT scan of the head

Explanation:

Even if you have no clue about the diagnosis, the first step in managing a patient with severe CNS-related abnormalities is a CT scan of the head in virtually any question you encounter on the boards. This patient has normal-pressure hydrocephalus (NPH). The gait disorder is usually characteristic and is the most reliable feature. Typically, the family describes the subacute onset of progressive intellectual deterioration accompanied by slowness and restriction of movement, particularly of the gait. There should also be the presence of bladder incontinence. In a way, it can be thought of as Parkinson's disease of the lower extremities. The disease is slowly progressive over weeks, months, or sometimes years. Parkinson's disease has many clinical features that are not present in NPH, such bradykinesia, rigidity, rest tremor, freezing, and postural instability. All of these are absent in this patient. In addition, NPH will not have a response to Sinemet. Although metoclopramide can cause a secondary parkinsonism, it should not cause cognitive decay or urinary incontinence. And even though donepezil may be useful for Alzheimer's disease, it will not help the memory loss of NPH. Ventriculo-peritoneal shunting would not be appropriate, unless a CT scan of the head is performed first. In short, don't start disease-specific therapies until you have confirmed a specific disease. Lumbar puncture is not a useful prognostic test. The only proof of shunt efficacy is to perform the shunt. Success is more likely if the shunt is done before the onset of severe cognitive problems.

Topic:

Neurology

Record # 80

Question/Fact:

A 43-year-old obese man is referred to you by his corporate masters for a cardiac evaluation prior to beginning an exercise program that mostly consists of playing intense games of racquetball with the boss. He denies any cardiac risk factors and has no history of coronary disease. His parents are robustly healthy, and he does not smoke. You find a blood pressure of 110/70 mg/dL. Laboratory tests show an LDL of only 140 mg/dL. His EKG is normal. In order to clear him for exercise, you order a thallium stress test, which shows a small reversible defect in his inferior wall. How would you manage him?

(A) Clear him for racquetball
(B) Tell him he will have no problem as long as he loses 10% of body weight prior to beginning exercise
© Start aspirin alone
(D) Start statins

Answer:

(D) Start statins

Explanation:

Although he has no cardiac risk factors, he has an LDL above 130 and a sestress test showing ischemia. Once you have the presence of coronary disease, risk factors such as hypertension, tobacco smoking, low HDL, family history, and the patient's age become irrelevant. Although obesity is certainly a risk for an increase in all-cause mortality, obesity is not specifically a risk factor in the evaluation of who needs lipid-lowering therapy. Statin therapy would be combined with a dietary restriction on fat intake, as well as weight loss.



Topic:

Cardiology




=====================================================
practical exam 2
=====================================================
brainX Digital Learning System
Study Session of KnowledgeBase: Practice Exam 2

Record # 1

Question/Fact:

A 55-year-old man with a past medical history significant for diabetes for 15 years presents to your office complaining of increasing shortness of breath over the past few months. Although he is pain-free today, he has had angina-like chest pain over the last several months. There is no radiation of the pain or nausea, vomiting, or diaphoresis. The patient's medications consist of metformin, glyburide, and lisinopril. He denies alcohol, tobacco, or illicit drug use.

On physical examination, the patient appears as an age-appropriate obese male. Blood pressure is 130/170 mm Hg, and heart rate is 66/min. Jugulovenous distention is present. There is an S3 gallop with lateral displacement of the point of maximal impulse and some minimal rales at the lung bases. There is no peripheral edema. An EKG reveals a normal sinus rhythm at a rate of 64/min with no ST elevation and no T wave inversions. Anterior and inferior leads have QS waves. An echocardiogram reveals four chamber dilatation, global hypokinesis, and an ejection fraction of 35%.

What is the next diagnostic step for this patient?

(A) Coronary angiography
(B) 24-hour Holter monitor
© Transesophageal echocardiogram
(D) Thallium stress test
(E) Endomyocardial biopsy

Answer:

(A) Coronary angiography

Explanation:

In the patient described, the physical examination and history paint a picture of dilated cardiomyopathy, although one should not come to a precise diagnosis until the EKG and echocardiogram are done. The patient denies any previous alcohol use, and there is no medical history suggesting the use of cardiotoxic drugs. The QS waves on the EKG probably represent previous ischemic events, such as a myocardial infarction. These might have gone unnoticed because of the patient's diabetes, leading to a "silent" myocardial infarction. Because this patient is symptomatic with anginal pain and dyspnea, the next diagnostic step
Reply
#4
1. Upper gastrointestinal endoscopy is the diagnostic study of choice when a patient presents with acute alkali ingestion.
2. Quantitative CT of the spine is the most sensitive diagnostic test for osteopenia but it is not a gold standard because of its poor reproducibility. The test of choice in most settings is dual-energy X-ray absorptiometry of spine
3. Despite of other risks, HRT is still the best form of therapy for osteoporosis in postmenopausal women
4. Pap smear: If 3 consecutive pap smears are normal, screening may be performed less frequently (every three years) in a low-risk patient. Screening is usually started at 18 and stopped at 60-75 years.
5. If a woman has undergone a hysterectomy for cancer or cervical dysplasia she needs yearly Pap smear screening of the vaginal epithelium and inspection of the vulvar and perianal epithelium.
6. Learn the Hepatitis-B prevention. Know when to give vaccine and when to give HBIG. If someone is exposed to HBV and has a documented response to HBV vaccination, he does not need anything else but reassurance
7. Human studies have demonstrated a significant association between type A personality and exaggerated cardiovascular response.
8. Pneumococcal vaccine contains capsular polysaccharides and it produces T cell independent B cell response
9. Racemic epinephrine decreases the need for intubation in patients with croup and should always be tried before any invasive procedure
10. Adolescent onset of hirsutism and virilism, with normal menstruations and elevated 17-hydroxyprogesterone, are diagnostic of congenital adrenal hyperplasia

11. Spontaneous hemarthrosis raises the suspicion for hemophilia for which factor VIII assay is diagnostic
12. Decreased tympanic membrane mobility is specific for the diagnosis of acute otitis media.
13. Tetralogy of Fallot, the most common cyanotic congenital disease in children of less than 4 years of age, presents with cyanotic spells and pansystolic murmur on examination.
14. Neonatal sepsis is suspected when a baby develops jaundice after the 3rd day and within the first week, especially if he is feeding poorly and lethargic; work-up with blood cultures and lumbar puncture is the next step.
15. Rubella is characterized by low-grade fever, lymphadenopathy (sub-occipital and posterior auricular) and rash
16. The prodrome of measles may show Koplik sign, a pathognomonic finding, which appears as bluish white lesions on the erythematous buccal mucosa opposite to the first and second upper molars, and sometimes on the inner conjunctivae and vaginal mucosa.
17. An epiglottis rolling in from side to side is the diagnostic finding of laryngomalacia, and all such babies should be fed in the upright position.
18. Bedwetting is a normal phenomenon until the age of 5.
19. Severe dehydration in a neonate in the presence of hyponatremia, hyperkalemia, hypoglycemia and metabolic acidosis suggests the diagnosis of congenital adrenal hyperplasia.


20. First step in the management of an infant with suspected congenital diaphragmatic hernia is the placement of orogastric tube.
21. Kawasaki’s disease is a disease of blood vessel walls leading to aneurysm formation. It is usually self-limited though it can be fatal because of aneurysm formation, thrombosis or rupture of the coronary arteries, leading to myocardial infarction.
22. Moderate to severe degree of slipped capital femoral epiphysis requires surgery
23. Ileo-ileal intussusceptions, which may be caused by Henoch-Schonlein purpura, require surgical reduction.
24. Friedreich Ataxia is an autosomal recessive condition and genetic counseling is recommended for prenatal diagnosis for parents with one affected child.
25. Displaced anterior fat pad is a radiographic sign of supracondylar fracture, which may be complicated by Volkmann’s ischemic contracture
26. Fragile X syndrome is characterized by low to normal IQ with learning disabilities, generalized language disability, short attention span, autism, large head, prominent jaw, large low set ears and macroorchidism.
27. Meningitis may be complicated by regression of developmental milestones
28. Cerebral anoxia is the most common cause of cerebral palsy
29. Fetal alcohol syndrome is characterized by irritability, mild to moderate mental retardation, hypoplastic maxilla, long philtrum, thin upper lip border, and microcephaly
30. Deficiency of sphingomyelinase causes Niemann-Pick's disease, which is characterized by cherry red macula, protruding abdomen, hepatosplenomegaly, lymphadenopathy, and regression of developmental milestones.

31. Bed rest with hip joint in the position of comfort is the treatment of choice for transient synovitis of the hip
32. Arthrocentesis and empiric treatment with IV nafcillin are the most appropriate measures for the management of suspected septic arthritis in a child
33. Aspirin therapy with monitoring of liver enzymes are the mainstay of the treatment of systemic juvenile rheumatoid arthritis
34. Presence of dermatitis herpetiformis and chronic non-bloody diarrhea in a child of 12-15 months, suggests the diagnosis of celiac disease
35. Positive anti-Smith antibodies and/or anti-double stranded DNA antibodies is specific and confirmatory for the diagnosis of systemic lupus erythematosus
36. Duodenal atresia presents with bilious vomiting, and its typical radiographic finding is the ‘double bubble sign’
37. Sudden onset of respiratory distress may be caused by foreign body aspiration, and rigid bronchoscopy is the procedure of choice, both for diagnostic and therapeutic purposes
38. Recurrent self-limiting episodes of vomiting and nausea in children, in the absence of any apparent cause, suggest the diagnosis of cyclical vomiting.
39. The 24-hour esophageal pH monitoring is the gold standard for the diagnosis of gastroesophageal reflux disease
40. Mast cell stabilizers are the drugs of choice for asthmatic patients who also have evidence of other allergic disorders


41. Increased gastric residues in a preterm neonate are highly suspicious for necrotizing enterocolitis.
42. Unilateral flank mass in a child more than 3 years is most likely due to Wilm’s tumor, which arises from the metanephros.
43. Nodular swellings in the irises of patients with neurofibromatosis are hamartomas.
44. The investigation of choice, for the diagnosis as well as the follow-up, in cases of Sturge-Weber syndrome is CT scan of the head. The cerebral lesions in Sturge-Weber syndrome are histologically similar to the facial lesions.
45. Turner's syndrome with 46 XY karyotype is associated with a higher incidence of gonadoblastoma, and hence, prophylactic bilateral gonadectomy is indicated.
46. Revise the management of hyponatremia from SIADH.
Mild (asymptomatic with sodium 120-130 meq/L) = Fluid restriction
Moderate (asymptomatic with sodium 110-120 meq/L) = Loop diuretic + normal saline
Severe (symptomatic) = hypertonic saline
47. NSAIDs can cause SIADH, which results in euvolemic, hypotonic, hyponatremia
48. Radioactive Iodine is the treatment of choice for Grave’s disease
49. Propylthiouracil can cause agranulocytosis, which should be suspected in patients presenting with sore throat and fever while on the drug
50. Hashimoto’s may rarely cause hyperthyroidism but the radioactive Iodine uptake is low thus distinguishing it from Grave’s disease


51. Indication for parathyroidectomy in asymptomatic patients with hyperparathyroidism are S. calcium 1 mg/dL above upper limit with urine calcium excretion over 50mg/24hr, urine calcium excretion over 400mg/24hr, cortical bone density more than 2SD below normal, under age 50-60, pregnancy, follow up difficulty.
52. Primary adrenocortical deficiency is associated with hyperpigmentation of the skin due to increased levels of ACTH that helps to differentiate from secondary adrenal insufficiency, which is due to pituitary failure.
53.
When Addison’ s disease is suspected clinically, the short cosyntropin (ACTH) stimulation test is performed to make a diagnosis
54. Failure to suppress 24-hour urine cortisol level following high dose dexamethasone suppression test suggests ectopic ACTH production rather than Cushing’s syndrome.

55. Know how to differentiate between primary and secondary hyperaldosteronism. Plasma rennin activity is low in primary hyperaldosteronism and is high in secondary hyperaldosteronism
56. Suspect DKA in stuperous patients with rapid breathing and history of weight loss, polydypsia and polyuria.
57. The single best screening test for virilizing neoplasm is to obtain serum testosterone and DHEA
58. Know the importance of fluid replacement in the treatment of acute hypercalcemia. Loop diuretics are indicated only after adequate rehydration.
59. Dehydration in Diabetes insipidus is managed with isotonic saline until the volume deficit is restored
60. Know that detection of microalbuminuria is the best screening test for diabetic nephropathy
61. Management of DKA includes restoration of intravascular volume with normal saline and correction of ketosis with regular insulin
62. In non-ketotic hyperglycemic hyperosmolar coma: ketones are not found in the serum; blood glucose levels are very high (greater than 600 mg/dl); there is no acidosis; anion gap is not elevated and serum bicarbonate level is either normal or mildly decreased; neurological manifestations including coma are often present; serum osmolality is greater than 310 mOsm/kg
63. Fluid replacement is the most important step in the management of non-ketotic hyperglycemic coma
64. Waning of insulin dose results in hyperglycemia at 3 AM and 7 AM whereas in Dawn phenomena the hyperglycemia occurs only at 7 AM.
65. Insulinomas secrete excessive amounts of insulin, C peptide, and pro insulin
66. Arterial pH or anion gap is the most reliable indicator of metabolic recovery in patients with diabetic ketoacidosis
67. Know the management of diabetic nephropathy. Protein restriction to 0.8 g/kg of body weight has a beneficial effect on the course of diabetic nephropathy.
68.
Thyroiditis is associated with hyperthyroidism and low radioactive Iodine uptake
69. In the management of type 2 diabetes mellitus, if one oral hypoglycemic agent becomes ineffective then often another agent from a different group is added to achieve better glycemic control.

70. Treatment of choice for streptococcal pharyngitis is single injection of IM Benzathine penicillin G.


71. TMJ syndrome often presents with unilateral and chronic pain in the muscles of mastication. Pain is described as a dull ache worsened by chewing. Recognise the radiation to the ear, jaw, and posterior cervical region.

72. Know the clinical features and management of peritonsillar abscess or quinsy, which is a suppurative complication of acute tonsillitis
• Contralateral deviation of the uvula
• Throat pain markedly more severe on the affected side and occasionally referred to the ipsilateral ear.
• Exudates on the tonsil AND Erythema of the tonsil



73. Fibreoptic laryngoscopy establishes the diagnosis of epiglottitis but it must be performed in the operating room with preparations already made to perform endotracheal intubation
74. While breaking bad news, physicians have to begin with an appropriate initial sentence, which would help the patient feel at ease
75. D-xylose absorption is abnormal both in bacterial overgrowth and Whipple’s disease. However, with bacterial overgrowth, the test becomes normal after antibiotic treatment.
76. When a motility disorder of esophagus is suggested by contrast studies, the next step is usually esophagoscopy to exclude mechanical causes of dysphagia like stricture or esophageal cancer
77
Esophagoscopy is indicated when a patient with gastroesophageal reflux disease (GERD) fails to respond to empiric treatment or when patient has features of complicated disease.

78. Hepatitis B virus immune globulin plus lamivudine treatment is the most effective treatment to prevent recurrent HBV infection after liver transplantation.
79. When diarrhea caused by clostridium difficile is severe, administration of metronidazole is also needed along with discontinuation of the offending antibiotic
80. Increased intragastric pressure during vomiting could cause tears in the mucosa of the cardia and sometimes of the distal esophagus- called Mallory-Weiss tears.
81. CT scan is the best test for the diagnosis of diverticulitis in acute setting
82. Fulminant colitis is a serious complication of ulcerative colitis and plain radiography shows distended colon. Proctosigmoidoscopy with biopsy establishes the diagnosis of ulcerative colitis and it can be performed in acute settings.
83. Always suspect Crohn’s disease as a cause for chronic diarrhea in a young patient
84. Migratory thrombophlebitis and atypical venous thromboses are suggestive for chronic DIC, most probably due to some visceral malignancy.
85. Abdominal CT scan is the next diagnostic test when abdominal ultrasound does not explain cholestatic jaundice
86. Gastro-intestinal (GI) blood loss without GI complaints in an elderly patient is most probably caused by colon cancer and needs a colonloscopy
87. Recurrent peptic ulcers with hypercalcemia are best explained by MEN type 1
88. Barium swallow is the initial test of choice for all patients with dysphagia
89. Manometry establishes the diagnosis of diffuse esophageal spasm
90. Recognize Zenker’s diverticulum. Barium esophagogram is the investigation of choice for confirmation


91. Know how to manage severe
symptomatic bradycardia. IV atropine is usually the first step.

92. Recognize the hemodynamic features of mediastinal hemorrhage and know how to differentiate it from pericardial tamponade
93. Recognize mesenteric arterial stenosis in a patient that presents with abdominal angina
94. Recognize the clinical presentation of mesenteric thrombosis. Severe abdominal pain out of proportion to the physical findings along with bloody diarrhea should make you think about this.
95. Think of PE in a postoperative patient with JVD and new onset RBBB.
96
. Recognize the classic presentation of a tension pneumothorax. Remember the hypotension shortness of breath jugular venous distension and decreased breath sounds

97. Recognize the clinical presentation of pulmonary edema. Iatrogenic fluid overload is one of the common causes of pulmonary edema in perioperative patients
98. MI is one of the differential diagnoses of acute abdominal pain and should be ruled out in patients with risk factors.
99.
Learn the clinical presentation of mesenteric artery thrombosis.Always consider abdominal angina in a patient with risk factors for atherosclerotic vascular disease. Postprandial pain typically shortly after 20-30 mins Pain Last 60- 90 mins. Wt loss because fear of food might cause pain. Abdo pain and blood in stool. Sudden onset. Tx. Laparotomy and prophylaxis antibiotics.



100. Antihypertensive management should be the first step in patients with aortic dissection with hypertension
101. Thiazide diuretics are the initial antihypertensive of choice in patients with osteoporosis.
102. Learn how to differentiate ischemic angina from aortic stenosis angina. Echocardiogram should be the next step in suspected AS.
103. Know the natural history and complications of Marfan's syndrome in pregnancy. Every woman with Marfan’s syndrome who is contemplating pregnancy must be screened by echocardiography to determine the size of her aortic root.
104. Know how to recognize and treat right ventricular infarction. IV NS to increase the outflow from right ventricle should be considered in these patients
105. Sudden onset of shortness of breath (SOB), bibasilar rales, and an 'apical' murmur radiating to the axilla are quite characteristic of new onset mitral regurgitation. Papillary muscle dysfunction, or rupture, is the most common cause of MR in this setting
106. Choose the appropriate initial antihypertensive therapy in a patient with asthma. HCTZ is the initial drug of choice in patients with chronic persistent asthma
107. IV adenosine is the drug of choice for paroxysmal SVT. Know how to recognize the rhythm on ECG
108. Any patient who presents with sudden onset of chest pain, SOB, and has evidence of hypoxia and the new onset right bundle branch block should be considered as having a PE until proven otherwise. Therefore, the next best step from the choices offered is to get a V/Q scan
109. IV nafcillin is the treatment of choice for cellulitis with systemic signs.
110. Lithium precipitates and exacerbates psoriasis which is characterized by sharply defined erythematous areas with whitish, silvery and scaly plaques
111. Look for electrolyte abnormalities and correct them in patients with arrhythmias
112. Know how to differentiate between primary hyperparathyroidism and familial hypocalciuric hypercalcemia in a patient with increased serum calcium, decreased serum phosphorus, and increased PTH levels. Calcium excretion is decreased in familial hypocalciuric hypercalcemia and is normal or elevated in primary hyperparathyroidism
113. In a patient with hypochloremic metabolic alkalosis due to gastric outlet obstruction, potassium is usually low and needs to be supplemented
114. Parathyroidectomy is the only effective treatment for primary hyperparathyroidism
115. Plummer-Vinson syndrome is characterized by atrophic glossitis, stomatitis, and upper esophageal webs associated with iron-deficiency anemia. Usually these webs don’t regress with conservative treatment and require dilatation
116. Paracentesis provides useful diagnostic information in patients with ascites
117. Slow tapping of ascitic fluid balanced with infusion of albumin per liter tapped, is the final conservative measure for the treatment of acsites secondary to cirrhosis that has to be attempted before surgery.
118. Hypotension, hyponatremia, azotemia, and oliguria with normal urinalysis in a patient with severe liver disease are suggestive of hepato-renal syndrome. Measures are taken to withhold all possible precipitating factors and initial management includes careful volume load.
119. Patients with carcinoid syndrome are at risk of developing niacin deficiency owing to increased formation of serotonin from tryptophan.
120. Varicoceles that fails to empty when the patient is recumbent raises the suspicion for renal cell carcinoma for which abdominal CT scan is the investigation of choice.

--------------------------------
121. Suspect medullary cystic disease in adults with recurrent UTI or renal stones and contrast filled cysts demonstrated by IVP
122. Know the different causes of nephrotic syndrome and their likelihood in a given patient. Focal segmental glomerulosclerosis (FSGS) is the most common cause of nephrotic syndrome in African American adults. It also occurs in patients with HIV and IV drug abuse.
123. Indications for dialysis includes uremic symptoms such as pericarditis, encephalopathy, coagulopathy, and fluid overload unresponsive to diuresis, hyperkalemia not amenable to standard therapy, and pH less than 7.20
124. Know the factors that have been shown to delay the progression of chronic renal disease. Protein restriction and use of ACE inhibitors are two such factors.
125. Acute pyelonephritis presents with fever, chills, flank pain, pyuria, and bacteriuria. CVA tenderness may be present. Sometimes WBC casts may be seen.
126. Hydration is the cornerstone of therapy for renal stone disease. A detailed metabolic evaluation is not needed when a patient presents with their first renal stone.
127. Colicky flank pain with radiation to groin indicates renal colic. Small bowel disease such as surgical resection or chronic diarrhea, leading to the malabsorption of fatty acids and bile salts can predispose calcium oxalate stones
128. Know the differential diagnosis of microscopic hematuria, and know to differentiate myoglobinuria from hematuria. Alcoholism can cause rhabdomyolysis; the resulting myoglobinuria causes a positive dipstick analysis for blood and a negative microscopic examination for RBC‘s.
129. IgA nephropathy is the most common cause of glomerulonephritis in adults. Suspect IgA nephropathy in patients with gross hematuria associated with upper respiratory infection, GI symptoms, or flu-like illness
130. Any patient with recurrent sinusitis, cavitary lung lesions, pulmonary hemorrhages and glomerulonephritis will most likely have Wegener’s glomerulonephritis
131. The common presentation of cryoglobulinemia includes palpable purpura, glomerulonephritis, non-specific systemic symptoms, arthralgias, hepatosplenomegaly, peripheral neuropathy, and hypocomplementemia. Most of these patients have evidence of Hepatitis C.
132. Suspect Alport’s syndrome in patients with recurrent episodes of hematuria, sensorineural deafness and family history of renal failure.
133. Oral trimethoprim-sulfamethoxazole is the preferred empiric treatment for acute uncomplicated cystitis.
134. When isolated proteinuria occurs, the evaluation of the patient should begin by testing the urine on at least two other occasions
135. Chlamydial urethritis is suggested by the presence of mucopurulent urethral discharge, sexual history with multiple partners, and absence of bacteriuria on urinalysis
136. Patients with prostatodynia are afebrile and have irritative voiding symptoms. Expressed prostatic secretions show normal number of leukocytes and culture of these secretions is negative for bacteria.
137. Ultrasound or other imaging techniques should be done in patients with pyelonephritis whose symptoms do not respond 48 -72 hours after appropriate antibiotics
138. Acute epididymitis in younger patients is usually caused by sexually transmitted organisms such as C. trachomatis or N. gonorrhea. In older men it is usually non-sexually transmitted and is caused by gram-negative rods.
139. Know how to recognize and treat nonbacterial prostatitis. Rule out bladder cancer in all elderly patients with irritative voiding symptoms and have negative urine culture.
140. Acute post streptococcal glomerulonephritis occurs 10-20 days after streptococcal throat or skin infection. It presents with hematuria, hypertension, red cell casts, and mild proteinuria.
141. Renal vein thrombosis is an important complication of nephrotic syndrome, which is the most commonly caused by membranous glomerulonephritis in adults.
142. 70% of cases with interstitial nephritis are caused by drugs such as cephalosporins, penicillins, sulfonamides, sulfonamide containing diuretics, NSAID’s, Rifampin, phenytoin, and allopurinol. Discontinuing the offending drug is the treatment of drug-induced interstitial nephritis.
143. Patients with nephrotic syndrome are at increased risk for developing hypercholesterolemia, hypertriglyceridemia, and hypercoagulable state.
144. Acalculous cholecystitis occurs in critically ill patients and imaging studies show diagnostic findings of thickening of the gall bladder wall and presence of pericholecystic fluid.
145. Primary Polydypsia (psychogenic polydypsia) is characterized by primary increase in water intake. Patients taking phenothiazines have this problem due to dry mouth caused by anticholinergic action of phenothiazines
146. Accidental ingestion of battery into esophagus warrants its immediate removal.
147. Warfarin induced skin necrosis is a complication seen in patients with protein C or S deficiency especially when it is started in high doses without heparin coverage
148. Gentamicin causes vestibular toxicity
149. Spherocytosis is due to RBC membrane defect mostly from spectrin deficiency
150. Aortic dissection is an acute emergency and its medical management includes prompt blood pressure lowering with IV nitroprusside and short-acting beta-blocker.
151. Vitamin A and its derivatives taken in large doses can lead to pseudotumor cerebri as a complication. The presentaion includes headache, nausea, vomiting, papilledema, and elevated CSF opening pressures, which is characteristic of pseudotumor cerebri.
152. Patients who are on high doses of vitamin D and calcium should have a close eye on serum calcium levels. Signs of toxicity include nause, vomiting, constipation, polyuria, and mental status changes.
153. Wernicke’s encephalopathy is a syndrome characterized by ataxia, ophthalmoplegia, nystagmus and altered mental status. Wernicke’s encephalopathy is seen in chronic alcoholics with thiamine deficiency
154. Hypercarotenemia is commonly seen in patients with anorexia, diabetes & hypothyroidism.
155. Excessive use of vitamin C in patients with renal insuffiency can cause oxalate stones
156. Pellagra, a niacin deficiency syndrome is characterized by a triad of 3 Ds of Diarrhea, Dermatitis, and Dementia and if untreated eventually leading to Death (4th D).
157. Patients who eat heated food all the time, such as toast and tea are more likley to develop folic acid deficiency.
158. Vitamin K deficiency is usually manifested by prolonged PT, normal platelet count and normal bleeding time. PTT is variable.
159. Vitamin B 12 deficiency: Subacute combined degeneration of the dorsal and lateral spinal columns is the classic neurologic manifestation. Symmetrical neuropathy, ataxia with loss of vibration and position sense is the clinical clues.
160. Penile fracture is a medical emergency and needs prompt surgical repair, but it should always be preceded by retrograde urethrogram to rule out urethral injury which is very common with penile fracture
161. Removal of impacted esophageal foreign body is mandatory
162. Pressure sores are common over the sacrum, heels and hips. Frequent turning of the patient is the only best method to prevent these
163. Heat stroke is a life threatening catastrophic medical emergency due to failure of thermoregulatory center leading to severe hyperthermia with body temperature generally greater than 105 F. Patients will have CNS symptoms
164. Heat exhaustion, also known as heat prostration is a syndrome characterized by volume depletion under the conditions of heat stress. It can be distinguished from heat stroke by T< 40 C and lack of severe CNS symptoms.
165. Rapid cooling of the patent is the cornerstone of management of heat stroke and should be done in adjunction of stabilizing treatment. Evaporation cooling is the preferred modality.
166. Think of Conversion disorder in a female patient with sudden onset neurological symptoms preceded by an obvious stressor. The treatment of choice would be Psychotherapy
167. Zinc deficiency is associated with alopecia, mental changes, diarrhea, and dysgeusia, smell abnormalities, maculopapular rash around the mouth and eyes, and impaired wound healing.
168. Severe, excruciating pain should be treated with IV opioids even if the patient has a history of drug abuse
169. Lithium toxicity presents with tremulousness, headache, confusion, GI distress, fatigue and, in extreme cases, with seizures, coma, hyperreflexia and opisthotonus
170. Hypothyroidism should always be considered in patients with an unexplained elevation of serum CK concentration and myopathy.
171. For frostbite injuries, the best treatment is rapid re-warming with warm water. Whenever frostbite or cold injuries are diagnosed, no attempt should be made to débride any tissue initially. Rapid re-warming with dry heat (like a fan) is not effective for frostbite.
172. Beta blockers decrease the risk of perioperative myocardial infarction, and thus do decrease mortality in perioperative patients with vascular disease undergoing noncardiac surgery. Beta 1 selective agents are the drugs of choice for this purpose.
173. Wiskott Aldrich syndrome is one of the immunodeficiency syndromes, characterized by recurrent pyogenic infections, atopic eczema and thrombocytopenia. IgM levels will be decreased, but IgA, & IgG will be within normal limits
174. In a HIV patient, bilateral interstitial pneumonia is most likley due to Pneumocystis carinii infection.
175. Flumazenil, a benzodiazepine antagonist, is antidote for acute benzodiazepine intoxication.
176. Glucagon is used to treat beta-blocker toxicity
177. Mucormycosis requires aggressive surgical debridement plus early systemic chemotherapy with amphotericin B.
178. Association of diabetes mellitus and Mucor-mycosis is frequently tested in the examination.
179. Empiric antibiotic therapy with either ceftazidime or cefepime should be started in febrile neutropenic patients
180. Invasive aspergillosis occurs in immunocompromised patients like those with neutropenia, those who are taking cytotoxic drugs like cyclosporine or those who are taking very high doses of glucocorticoids. Invasive pulmonary disease presents with fever, cough, dyspnea or hemoptysis. Chest x-ray may show cavitary lesion. CT scan shows pulmonary nodules with halo sign or lesions with air crescent.
181. Treatment of disseminated Histoplasmosis in HIV patients is IV amphotericin B followed by lifelong treatment with itraconazole
182. Lymphocutaneous Sporotrichosis is the most common form and presents with painless papule at the site of inoculation which later on ulcerates.
183. Always remember Blastomycosis is big-based budding yeast. Blastomycosis is endemic in south central and north central USA.
184. Know the epidemiology of the fungal infections. Patient from Arizona should make you think of Coccidioidomycosis
185. Whenever there is suspicion of HSV encephalitis, IV acyclovir should be started without delay
186. Always suspect Infectious mononucleosis in a young patient presenting with sore throat and nonspecific spectrum of symptoms. Recognise the role of steroids in treating complications.
187. In both infants and children, Staphylococcus aureus is the most common organism responsible for osteomyelitis.
188. S. Epidermidis is a frequent cause of osteomyelitis associated with prosthetic devices.
189. Gram-negative rods like Klebsiella or Pseudomonas cause osteomyelitis when patients have a history of urinary tract infection (UTI) or urinary instrumentation
190. Three phase technetium scan is the test of choice for acute osteomyelitis and it is very sensitive.
191. Learn the clinical presentation, pattern of organ involvement and the complications in a case of mumps. Orchitis is one of the most frequent complications of mumps in adults.
192. Always consider the risk of splenic rupture in case of infectious mononucleosis and advise the patient to avoid contact sports to prevent this hazard.
193. Amoxicillin-clavulanate is the antibiotic of choice for prophylaxis against infections caused by human bite
194. Cat-scratch disease that usually occurs after scratch or bite by a cat and is caused by Bartonella.
195. Strep viridans is a frequent case of infective endocarditis in patients with damaged heart valves
196. Failure to respond to herpes simplex with acyclovir indicates resistance. The patient most likely requires foscarnet.
197. Babesiosis should be suspected in any patient from an endemic area who presents with a tick bite. Remember 'splenectomized patient'
198. The herpes simplex virus (HSV) esophagitis ulcers are usually multiple, well circumscribed and have a "volcano" (small and deep) like appearance, whereas the ulcers seen in CMV infection tend to be large, shallow and superficial.
199. Skin lesions of Kaposi sarcoma usually occur on the trunk, face and the extremities. Usually these lesions are papules that later on become plaques or nodules. The color changes from light brown to pink to dark violet.
200. So far there has been no case reports of congenital rubella syndrome in women inadvertently vaccinated during early pregnancy.
201. Pyridoxine (Vitamin B6) is an antidote for isoniazid
202. Duration of QRS complex is the best measure for assessment of severity of tricyclic antidepressant toxicity.
203. All chemical injuries to eyes should be immediately washed with plain water or saline.
204. Impaired concentration and conjunctival injection are important features of marijuana use.
205. Calcium is useful in reversing cardiac effect of calcium channel blocker.
206. Tyramine contained in aged cheese reacts with monoamine oxidase inhibitors to produce hypertensive crisis
207. Dextromethorphan interacts with monoamine oxidase inhibitors and can produce severe hyperthermia.
208. Decision of use of N acetyl cysteine as an antidote for acetaminophen overdose is generally based on 4 hour post-ingestion acetaminophen levels
209. Bromocriptine is used in the treatment of neuroleptic malignant syndrome, which is characterized by marked hyperthermia, muscular rigidity, tremors, altered mental status, and diaphoresis.
210. First step in the management of pesticide poisoning is to remove the source of poison.
211. Phenothiazines cause hypothermia by causing vasodilatation and by inhibition of shivering
212. Sodium bicarbonate prevents the development of arrhythmia in patients with tricyclic antidepressant toxicity by alleviating cardio-depressant action on sodium channels
213. Recognize the affect of alcohol intake on acetaminophen induced hepatic injury
214. In case of suspected osteomyelitis, blood cultures and x-rays should be taken and the patient should be started on intravenous antibiotics. If the x-rays are negative three-phase technetium bone scan should be considered. Needle biopsy can be done to identify the organism if the blood cultures are negative.
215. Cervicofacial actinomycosis classically presents as slowly progressive, non-tender, indurated mass, which evolves into multiple abscesses, fistulae, and draining sinus tracts with sulfur granules, which appear yellow. Treatment is high dose penicillin for 6-12 weeks.
216. Cauda equina is an acute compression syndrome presenting with acute motor and sensory loss, loss of rectal tone and urinary retention. It has to be treated urgently
217. Neurofibromatosis is a neurocutaneous syndrome, which has a tendency to form tumors of the central nervous system (CNS), peripheral nervous system, skin and viscera
218. CT scan showing hyperdense (White) area suggests hemorrhage. The infarcts on CT are seen as hypodense (Black) parenchymal areas.
219. Asymptomatic patients with carotid artery stenoses of 60 to 99 percent are considered to have a proven indication for CEA. Complete occlusion (100 percent stenosis) of the carotid artery is a contraindication to surgery
220. When a patient presents with painless visual loss lasting a few seconds, a duplex ultrasound of the carotids should be the first test of choice.
221. Petit mal seizures may present with a sudden cessation of mental activity, which occur repeatedly. There are not associated with complex automatisms or tonic clonic activity. Diagnosis can be best confirmed by EEG studies
222. Primidone is an anticonvulsant agent, which can be used to treat benign essential tremor. However, it can also precipitate acute intermittent porphyria. Diagnosis of porphyria is made by assessing urine for porphobilinogen.
223. Itch, redness and fissuring in swimmers is typical of athlete’s foot. The condition is best treated with antifungal creams such as tolnaftate.
224. In a child who has been to the ER several times and has bruises of unknown origin on his/her body, child abuse should always be suspected and reported to the appropriate authorities.
225. Gonorrhea is a common organism, which causes sexually transmitted disease. It is also a common cause of pharyngitis, generally acquired from oral sex.
226. Postherpetic neuralgia can cause significant pain along the nerve dermatomal distribution. The best treatment is with the anti viral agent, acyclovir
227. Urethral cultures have higher yield than synovial or blood culture in cases of suspected gonococcal purulent arthritis.
228. Always suspect endocarditis whenever a patient is febrile and has other constitutional features in the presence of a new heart murmur.
229. Patients with trichinosis, presents with gastrointestinal (GI) complaints followed by muscle pain, swelling, and weakness. Presence of subungual splinter hemorrhages, conjunctival and retinal hemorrhages, periorbital edema and chemosis should make you think about trichinosis.
230. Always suspect rubella when a febrile patient with rash has occipital or posterior cervical lymphadenopathy and arthritis.
231. Vibrio parahaemolyticus is usually transmitted by ingestion of seafood. Patients having symptoms of food poisoning after intake of seafood should be suspected for this.
232. Group A beta-hemolytic streptococcus is the most frequent cause of erysipelas and cellulitis.
233. Proteus is the most likely cause of urinary tract infection in patients with alkaline urine.
234. Facial nerve palsy and classical Erythema migrans (EM) indicate Lyme disease, which is a tick borne disease.
235. Pseudomonas aeruginosa is the most frequent cause for malignant otitis externa.
236. Mefloquine is the drug of choice for chemoprophylaxis of chloroquine resistant malaria
237. Amoxicillin is used for treatment of pregnant/lactating patients with early-localized Lyme disease
238. Doxycycline is used to treat primary syphilis in penicillin allergic patients
239. Pneumococcal vaccine is recommended in all HIV infected patients whose CD4 count is > 200 cells/micro-L.
240. IV amphotericin plus flucytosine is the antibiotic regimen of choice for central nervous system cryptococcal infection in AIDS patients.
241. Bacillary angiomatosis, a great mimicker of Kaposi sarcoma, is caused by gram-negative rod Bartonella and is successfully cured with antibiotic treatment. They are highly vascular, initially are smooth and in the form of small papules that gradually become enlarged and pedunculated or nodular.
242. Azithromycin or clarithromycin is the drug of choice for MAC prophylaxis in HIV patients with CD4 cell count below 50/microL.
243. Whenever a health care worker is exposed to HIV, baseline HIV testing should be performed immediately and postexposure prophylaxis with combination of two or three antiretroviral drugs should be started without any delay.
244. Any patient from southwestern region with history of tick bite developing systemic symptoms along with leukopenia and thrombocytopenia should make you think about Ehrlichiosis.
245. HIV patients are at a high risk for tuberculosis and require prophylactic treatment to prevent an active disease. PPD is done to verify this. Skin induration of greater than 5 mm with PPD testing in HIV patients requires prophylaxis with isoniazid for 9 months
246. Listeria monocytogenes is a frequent cause of meningitis in patients older than 55 and therefore in such patients ampicillin should be added to the empiric antibiotic regimen.
247. Presence of erythema chronicum migrans in a patient with tick bite history warrants immediate treatment without waiting for serology of Lyme's disease.
248. Lyme disease prophylaxis is given in pregnant patients who have history of tick bite and have anxiety about acquiring disease.
249. Methanol intoxication is associated with visual loss.
250. Patients with hemochromatosis are vulnerable to listeria monocytogenes infections and some other bacterial infections. Know the various bugs that are likely to jeopardize such patients.
251. Pinpoint pupils and respiratory depression are the hallmark features of acute opioid toxicity for which naloxone is the drug of choice
252. Heterophil antibody test is sensitive and specific for diagnosis of IM. EBV specific antibody test is used in patients with high suspicion for IM and have negative heterophil antibody test.
253. Albendazole or mebendazole is the first line of treatment for E. vermicularis infection. Pyrantel palmate is an alternative.
254. Consider CMV pneumonitis as a late complication in post BMT recipients with dyspnea and cough.
255. Serotonin agonists (triptans) are the drugs of choice to abort acute attacks of migraine when simple analgesics or NSAIDs fail.
256. Pregnancy testing should be considered in patients with high risk of being pregnant prior to starting treatment with sumatriptan
257. The choice of prophylactic agent for migraine depends upon any associated medical condition. In patients with coexistent depression, antidepressants should be used for prophylaxis of migraine.
258. Rupture of saccular aneurysm is the most frequent cause of non-traumatic subarachnoid hemorrhage (SAH). Know the CT appearance of SAH.
259. Campylobacter jejuni is the most frequent precipitant of Guillain-Barré Syndrome (GBS).
260. Atrophy of the caudate nucleus is a characteristic feature of Huntington’s chorea. Recognize the classic clinical presentation of Huntington’s chorea.
261. Acetazolamide is the first line medical treatment for idiopathic benign intracranial hypertension.
262. Acute exacerbations of MS are treated with corticosteroids. Beta-interferon or Glatiramer acetate is used to decrease the frequency of exacerbations in patients with relapsing-remitting or secondary progressive form of MS.
263. Oligoclonal bands are present in 85-90% of cases of multiple sclerosis. CSF pressure, protein and cell count is grossly normal in patients with MS.
264. Initial presentation of multiple sclerosis with optic neuritis or sensory symptoms carries a good prognosis.
265. Cutaneous larva migrans is a common cause of dermatological disease in travelers from tropical regions, and is characterized by pruritic elevated serpiginous lesion on the skin.
266. Always consider malaria in patients from endemic areas with high-grade periodic fever and chills. Anemia and splenomegaly are the clinical clues.
267. Condyloma Lata and bilaterally symmetrical maculopapular rash involving the entire trunk and extremities are characteristic of second stage of syphilis. Remember the rash is present on palms and soles. Serological tests are positive in secondary syphilis.
268. It is recommended that all patients with atherothrombotic TIA should receive an antiplatelet agent if there is no contraindication to its use and antiplatelet agent of initial choice for this purpose is aspirin.
269. Anticoagulation is considered when TIA is due to emboli arising from the heart.
270. Anticholinergics are useful treatment for Parkinsonism patients who are younger than 70 with disturbing tremor and minimal bradykinesia. Parkinsonism tremor is a resting tremor.
271. Oral anticholinesterase is usually the initial treatment of choice for myasthenia gravis and all patients should have evaluation for thymectomy
272. First step in all patients suspected of having stroke is CT scan without contrast. If they have evidence of ischemic stroke, then carotid Doppler and TEE are performed to evaluate the possible source of embolism.
273. Presence of cord cavity is the most characteristic feature of syringomyelia that typically presents with areflexic weakness in the upper extremities and dissociated anesthesia in a "cape" distribution. Other features like caudal displacement of fourth ventricle or cerebellar tonsils or focal cord enlargement may or may not be present.
274. CT/MRI picture of heterogenous and serpiginous contrast enhancement is typical of high-grade astrocytoma. Recognize the classic butterfly appearance.
275. Patient with cerebellar hemorrhage presents with ataxia, vomiting, occipital headache, gaze palsy, and facial weakness. There is no hemiparesis. It is crucial to make early and correct diagnosis, as urgent surgical decompression may be life saving in such cases.
276. Treatment of myasthenia crisis consists of endotracheal intubation and withdrawal of anticholinesterases for many days.
277. The most common site of hypertensive hemorrhage is putamen (35%). Internal capsule that lies adjacent to putamen is almost always involved leading to hemiparesis.
278. All those patients who are on warfarin treatment and develop serious bleeding need rapid reversal of anticoagulation and fresh-frozen plasma is the most appropriate first choice for this purpose.
279. Untreated LGV may progress to a severe and chronic disease causing ulceration, proctocolitis, rectal stricture, rectovaginal fistulas and elephantiasis.
280. Riluzole is a glutamate inhibitor and is currently approved for its use in amyotrophic lateral sclerosis.
281. Intravenous cefotetan, ampicillin/sulbactam, or the combination of clindamycin and a fluoroquinolone is the appropriate empirical treatment for limb-threatening infections in diabetics, whereas mild, or non-limb-threatening, infections can be treated with oral antibiotics like cephalosporin, clindamycin, amoxicillin/clavulanate and fluoroquinolones.
282. The majority of human infections with E. granulosus are asymptomatic. Hydatid cysts can be found in almost any part of the body but liver (MC) is involved in two-thirds of patients and the lungs are involved in 25 percent of patients. It is most commonly seen in areas where sheep are raised.
283. Neurocysticercosis is the most common parasitic infection of the brain and highest prevalence seen in rural areas where pigs are raised and there are poor sanitary conditions
284. Streptococcus bovis endocarditis is associated with colorectal cancer and colonoscopy is advisable in such patients.
285. Condylomata acuminata are skin colored or pink, verrucous and papilliform skin lesions present around the anus and podophyllin is one of the available treatment options.
286. Eaton–Lambert syndrome is associated with small cell carcinoma of the lung and results from autoantibodies directed against the voltage-gated calcium channels in the presynaptic motor nerve terminal.
287. Immunocomplex disease is primarily responsible for glomerulonephritis, Roth spots and Osler’s nodes. Janeway lesions result from septic embolism.
288. MRI is the test of choice to support the clinical diagnosis of multiple sclerosis
289. Multiple observational studies have demonstrated that patients with hypertension have about four times the risk of stroke when compared to non-hypertensive subjects
290. Understand the concept of lead-time bias in screening tests. The concept of lead-time bias includes prolongation of apparent survival in patients to whom this test was applied, without changing prognosis for the disease.
291. Confounding is referred to the bias that can result when the exposure-disease relationship is mixed up with the effect of extraneous factors called confounders.
292. Know the concept of positive predictive value; positive predictive value depends on the prevalence of the disease of interest in the population to which the test is applied. PPV increases with an increase in prevelance of disease in the study population. For NPV, the inverse is true.
293. Thymectomy may induce remission in patients with myasthenia gravis and should be considered in all patients who are between puberty and 60 and whose disease is not confined only to the extraocular muscles.
294. Incidence is the measure of new cases, the rapidity with which they are diagnosed. Prevalence is the measure of the total number of cases at a particular point of time.
295. 68% - within 1 standard deviation from the mean
95% - within 2 standard deviations from the mean
296. 99.7% - within 3 standard deviation from the mean
297. Observer bias occurs when the decision as to whether outcome is present is adversely affected by the knowledge of the exposure status.
298. Any HIV-positive patient with bloody diarrhea and normal stool examination should have a colonoscopy and biopsy done to look for CMV colitis. CMV colitis is characterized by bloody diarrhea with abdominal pain, multiple ulcers and mucosal erosions on colonoscopy; biopsy shows characteristic cytomegalic cells with inclusion bodies.
299. Dihydropyridine Ca-channel antagonists can cause peripheral edema and should always be considered in the differential diagnosis of this condition, along with other causes, such as heart failure, renal disease and venous insufficiency.
300. Elderly patients with depression often present with memory loss (Pseudodementia.), which can mimic other causes of dementia, including Alzheimer’s dementia, while the CT will be normal in these patients, the DST may be of value.
301. In cross-sectional study, exposure and outcome are measured simultaneously at a particular point of time (you can remember it as a ‘snapshot’ study). In other study designs, a certain time period separates exposure and outcome.
302. Situational syncope should be considered in the differential diagnosis of syncopal episodes. The typical scenario would include a middle age or older male, who loses his consciousness immediately after urination, or a man who loses his consciousness during coughing fits.
303. Recognize normal pressure hydrocephalus by the triad of gait disturbance, dementia and urinary incontinence. Other features are normal CSF pressures on lumbar puncture and enlarged ventricles on MRI
304. VDRL testing, PPD skin testing, Hepatitis A and B serology and antibody titer for Toxoplasma are indicated as a part of initial work-up in all newly diagnosed HIV-positive patients.
305. Primary polydypsia is a disorder in which the patient drinks fluids in excess of 5 L per day and both plasma and urine are diluted.
306. SIADH results in hyponatremia, low serum osmolality and inappropriately high urine osmolality.
307. Diabetes insipidus presents with polyuria and polydypsia and urine osmolality is less than serum osmolality.
308. Several well-organized and well-controlled clinical trials demonstrated that ACE inhibitors could slow the progression of diabetic nephropathy and reduce albuminuria
309. Somogyi effect refers to early morning hyperglycemia caused by counter regulatory hormones following nocturnal hypoglycemia
310. Acute respiratory alkalosis (increase in extracellular pH) causes a fall in ionized plasma calcium by increasing the affinity of albumin to calcium
311. There are various mechanisms by which malignancy produces hypercalcemia. These are: the production of cytokines, parathyroid hormone related peptide, calcitriol and ectopic PTH
312. Remember when albumin is low, serum calcium concentration falls in the ratio of 0.8-1 mg of calcium to 1 g of albumin.
313. Always suspect surreptitious vomiting as a cause of hypokalemic alkalosis in a normotensive patient and distinguish it from other entities like diuretic abuse and Bartter’s syndrome on the basis of urine chloride concentration
314. Distinguish primary hyperaldosteronism from renovascular hypertension based on low plasma renin activity in the former and high plasma renin activity in the later.
315. Patients with Bartter’s syndrome have hypokalemia, metabolic alkalosis, normal blood pressure and elevated urine chloride concentration.
316. Patients with primary hyperaldosteronism show persistent elevation of BP, hypokalemia, metabolic alkalosis, mild hypernatremia, very low plasma renin activity and absence of edema
317. Measurement of glycosylated hemoglobin provides good evidence of how well the glucose has been controlled over the past 100-120 days (RBC survival time)
318. Hypercholesterolemia is the most frequent lipid abnormality in patients with hypothyroidism.
319. Hyperlipidemia, unexplained hyponatremia and elevated serum muscle enzymes are indications for thyroid function tests
320. Whenever a patient presents with thyroid nodule, first step is assessment of thyroid function.
321. Hyperplastic colloid nodule is the most frequent cause of thyroid nodule. Majority of thyroid nodules are benign, not malignant.
322. Patients with generalized resistance to thyroid hormones have high serum free T4 and T3 and normal or high serum TSH. Patients typically have features of hypothyroidism despite of having elevated free thyroid hormones
323. Agranulocytosis is a serious side effect of propylthiouracil
324. Radioiodine is more likely to cause permanent hypothyroidism in patients with Graves’ disease than multinodular goiter or toxic adenoma
325. Patients with osteomalacia have low or low-normal serum calcium, low serum phosphate and increased serum parathyroid hormone.
326. Osteomalacia is characterized by defective mineralization of bone
327. Always suspect hypophosphatemic rickets in patients of rickets who has normal serum calcium, normal serum alkaline phosphatase and normal 25-OH vitamin D.
328. Dopamine agonists like bromocriptine or cabergoline are the mainstay of treatment for patients with prolactinoma
329. Mucosal neuromas are the most distinctive feature of multiple endocrine neoplasia type 2b and are present in more than 90% of cases
330. Prolactinoma is the most common pituitary tumor
331. The most common thyroid hormone pattern in sick euthyroid patients is a fall in total and free T3 levels and normal levels of T4 and TSH. Remember ‘low T3 syndrome’
332. Antiperoxidase antibodies are the most prevalent antibodies in patients with Hashimoto’s thyroiditis.
333. Patients with Hashimoto’s thyroiditis are at an increased risk for developing lymphoma of thyroid
334. Papillary carcinoma of thyroid is the most common thyroid malignancy. Invasion of tumor capsule and blood vessels is the hallmark feature of follicular carcinoma of thyroid.
335. Subacute lymphocytic thyroiditis presents with painless enlargement of thyroid gland and hyperthyroidism. Duration is less than 2 months, goiter is small, no ophthalmopathy, and radioactive iodine uptake is low in such patients.
336. Asymptomatic patients with Paget’s disease generally don’t require any treatment. Symptomatic patients are best treated with bisphosphonates
337. MEN II consists of a medullary carcinoma of the thyroid, hyperparathyroidism and pheochromocytoma
338. In Leydig cell tumors, the estrogen production can be increased with secondary inhibition of LH and FSH
339. Male secondary hypogonadism is characterized by low levels of testosterone and low normal gonadotropins
340. DHEA and DHEA-S are produced by the adrenal glands and are used as markers for the adrenal tissue
341. 21-hydroxylase deficiency is the most common form of CAH. It presents with virilism, salt wasting, and increased 17-alpha-hydroxyprogesterone levels.
342. Most significant clinical impact of GH excess in acromegaly occurs on cardiovascular system, which accounts for 38-62 percent of deaths in acromegaly patients, CHF being the most common
343. In central DI, urine osmolality becomes greater than that of serum on arginine vasopressin administration while it remains lower than that of plasma when etiology is nephrogenic in origin. Treatment of choice for central DI is desmopressin, which is usually administered intranasally.
344. Fasting blood glucose measurement is now the recommended test for screening high-risk individuals for diabetes mellitus
345. Know the risk factors for development of foot ulcers in diabetics. Neuropathy is found in about 80% of diabetics with foot ulcers
346. Symmetric distal sensorimotor polyneuropathy is the most commonly seen neuropathy in diabetes and is characterized by classic "stocking glove" pattern sensory loss. The most common cranial nerve involved in diabetes is cranial nerve III. The most commonly involved peripheral mononeuropathy is the median nerve.
347. Diabetic cystopathy manifests as overflow incontinence. The pathology involves weak detrusor contraction secondary to autonomic neuropathy. Treatment is usually started with strict voluntary urinary scheduling and bethanechol; if no response, then the use of intermittent catheterization is advised.
348. GI manifestations of diabetic neuropathy include gastroparesis, constipation and diarrhea. Metoclopramide (drug of choice), bethanechol and erythromycin are useful in the management of gastroparesis
349. Diabetic neuropathy can be accompanied by acute or chronic pain. Treatment options for chronic diabetic neuropathic pain are gabapentine and TCAs (amitriptyline or desipramine).
350. Know the management of diabetic ulcers. Good wound care and debridement is key to the management. Antibiotics alone do not cure diabetic ulcers
351. Fasting blood glucose measurement is now the recommended test for screening high-risk individuals for diabetes mellitus. When fasting plasma glucose is 126 mg/dL or greater, repeat it, and if it is still elevated, diagnosis of diabetes mellitus is established and treatment is started.
352. The first test in all patients with unexplained hypertension and hypokalemia is measurement of plasma renin activity and plasma aldosterone concentration. If it is consistent with primary hyperaldosteronism, do an aldosterone suppression test for confirmation; once confirmed, perform a CT scan of the adrenals to look for a mass; but, if there is no mass on CT scan, do adrenal vein sampling
353. Nocturnal penile tumescence helps differentiate psychogenic causes of male erectile dysfunction from organic causes. It is positive in psychogenic causes and negative in organic causes
354. Glasgow coma scale is used to assess the severity of head injury but it does not indicate the presence or absence of increased intracranial pressure (ICP).
355. Patients with mild head injury can be discharged with a “head sheet” if they have a normal CT scan.
356. Pain relief should be the prime objective in management of rib fracture in elderly
357. In reference to neck trauma, neck can be divided into three zones and the treatment depends on the zone involved
358. Any gunshot wound of the abdomen requires exploratory laparotomy. Any gunshot wound below the 4th intercostal space (level of nipple) is considered to involve the abdomen.
359. Hypotension not responsive to fluid administration is suggestive of ongoing blood loss and such patients with abdominal trauma need an exploratory laparotomy.
360. All the patients with acute carbon monoxide poisoning should be treated with 100% oxygen via a facemask.
361. Delayed emergency from anesthesia is characterized by hypoventilation, which is evident by decrease in respiratory rate, hypertension progressing to hypotension, tachycardia progressing to bradycardia, restlessness and pallor/cyanosis.
362. It is important to rule out a fracture or dislocation of cervical spine as the first priority because of grave consequences of missing a cervical spine injury.
363. Hyperventilation helps to prevent and treat intracranial hypertension by causing cerebral vasoconstriction and thus decreasing cerebral blood flow.
364. Orotracheal intubation and surgical cricothyroidectomy are preferred way to establish an airway in apneic patient with head injury
365. Nasoethmoidal fractures may be associated with injuries to ethmoidal roof, lacrimal system, medial canthal tendons, cribriform plate and vasculature of nose. Associated injuries of these structures can give rise to cerebrospinal fluid rhinorrhea, epistaxis, anosmia, epiphoria or telecanthus
366. Beck's triad of hyotension, elevated JVP, and muffled heart sounds confirms the diagnosis of pericardial tamponade
367. Primary resuscitation of any trauma patient is based on establishing airway, breathing and circulation (ABC of resuscitation).
368. An airway is needed in all unconscious patients and in emergency room an orotracheal intubation is the best option. Hemodynamically unstable trauma patients should be started on IV crystalloids immediately.
369. Aortic rupture should be ruled out in all the chest trauma patients with hypotension. Screening for aortic trauma can best be done with a chest x-ray. Confirmatory test is angiography or spiral CT scan.
370. Saphenous vein cut down or percutaneous femoral vein catheterization are alternatives to have an intravenous access in trauma patients with collapsed veins.
371. Expectant therapy is a rule for all patients with uncomplicated basilar skull fracture. Clinical signs of basilar skull fracture includes rhinorrhea, raccoon eyes (black eyes), and ecchymosis behind the ears and otorrhea.
372. Acute subdural hematoma has a classic presentation of unconsciousness followed by a lucid interval followed by gradual deterioration of consciousness. CT scan is diagnostic and it shows a biconvex hematoma.
373. Treatment of acute subdural hematoma is essentially conservative if no midline shift is present on CT scan.
374. CT scan of a diffuse axonal injury shows numerous minute punctuate hemorrhages with blurring of grey-white interface.
375. Hemothorax is most commonly managed by a tube thoracotomy in lower part of chest. Surgery is seldom necessary
376. Anterior cord syndrome is commonly associated with burst fracture of the vertebra and is characterized by total loss of motor function below the level of lesion with loss of pain and temperature on both sides below the lesion and with intact proprioception.
377. Fat embolism presents with dyspnea, confusion and petechiae in the upper part of the body and occurs after multiple fractures of long bones
378. Know the tetanus immunization.
379. An exploratory laparotomy is needed in all the patients with blunt trauma to abdomen with signs of peritoneal irritation.
380. Patient with head injury can never have hemorrhagic shock due to intracranial bleeding
381. USG or DPL are the procedures of choice to diagnose intra abdominal bleeding in an unstable trauma patient.
382. Posterior urethral injury is associated with pelvic fractures and presents with blood at meatus, high riding prostrate, scrotal hematoma and inability to void in spite of sensation to void.
383. Retrograde urethrogram should be the first step in management of suspected posterior urethral injury
384. Intraperitoneal bladder rupture can occur in trauma patient with full bladder
385. Retrograde cystogram with post void film is the investigation of choice for patients with suspected bladder trauma
386. Penile fracture is a medical emergency and needs prompt surgical repair
387. Rule out vascular injuries in case of penetrating wound near the site of important vessels
388. The usual sequence of management of injury to bone, artery and nerve is to stabilize the bone followed by repair of vasculature followed by the nerve repair
389. All the patients with traumatic wound should be assessed for need of Tetanus toxoid and/or tetanus immunoglobulin
390. Pelvic X ray should be routinely done in all patients with trauma to screen for pelvic injury.
391. Vital signs, hemodynamic stability, and need for blood transfusion are important determinant for surgical v/s non-surgical management of patient with splenic trauma.
392. Rhabdomyolysis can occur with severe crush injuries and should be managed with IV fluids, osmotic diuretics and alkalinization of urine
393. Infection is the most common cause of death in burns patients
394. Body surface involved in burn injury is calculated with the help of “Rule of Nines.
395. Burns patients need 4 ml / kg / % of the body area involved of fluids in first 24 hours, half of which is given in the first 8 hours
396. Inhalation injury is common in burns patients and may take several days to manifest. Diagnosis is best done with a bronchoscopy.
397. When circumferential full-thickness burns involving the extremities or chest are present, escharotomy may be necessary
398. Know the injuries suggestive of child abuse and report such cases to child safety services
399. Superficial and erythematous burns while painful do not require any special wound care. Topical or oral analgesics agents can be prescribed.
400. Early excision therapy is indicated for extensive partial-thickness and full-thickness burns, as they do not heal spontaneously. Also, it allows for early skin grafting and lesser complications
401. Know the type of burn injuries and difference between burns and scalds. Scalds are the burn injuries secondary to contact with liquids. The type of burn injury in case of scalds depends on the type of liquid and duration of contact.
402. Psoriatic arthritis (PA) is asymmetrical and oligo-articular and such patients usually have typical features (silvery scales on erythematous plaques over flexural surfaces) of psoriasis present for years. Think of PA when pitting nails is present in the history. Skin rash may not be present all the time.
403. Chronic renal failure is the most common cause of death in systemic lupus erythematosus patients.
404. Inflammatory myopathies are best treated with corticosteroids
405. Presence of fever, inflammatory signs, no crepitus or bullae and signs of overlying skin necrosis all suggest that the patient has cellulitis. This patient also had toe web tinea pedis, which is one of the most common portals of entry for the microorganisms, causing cellulitis.
406. Osteoarthritis is a degenerative bone disease involving mainly the weight bearing joints. Know the pattern of joint involvement, x-ray findings and the differentiating features of this disease.
407. The demonstration of negatively birefringent needle shaped urate crystals in the synovial fluid is suggestive of
Reply
#5
USMLE WORLD NOTES * ENDOCRINE SYSTEM

►DIABETES MELLITUS: Tight glycemic control DECREASES risk of developing NEW peripheral neuropathy

1-DM diagnosis => fasting glucose in 2 separate times > 126
2-Pts with uncontrolled impaired fasting blood glucose [B/w 100 – 126] are at risk for => CAD- CORONARY ARTERY DISEASE [even with a normal lipid profile] and progression to evident DM
3-Fasting blood glucose < 100 is normal
DM: HYPOGLYCEMIA => Glucose < 60
► For the diagnosis of DM = use fasting blood glucose levels [> 126 /x2]
Or= Random blood sugar > 200
♥ RISK OF DM TYPE 1 IN OFFSPRING :
Mother with DM type 1 = RISK is 3%
Father with DM type 1 = RISK is 6%

►DM: Antihypertensive drugs that have been shown to reduce risk of development of type 2 DM:
o ACEIs [reduce insulin resistance] + lifestyle modifications
o Metformin
►Causes of hyperglycemia:
Stress due to any illness can increase in insulin resistance causing a poor glycemic control and aggravating hyperglycemia
Example –acute attack of gout, and not diet or medication compliant

►Polyglandular Autoimmune failure type II [Schmidt’s syndrome] => Addison disease, type I DM, Autoimmune thyroid disease and other autoimmune disorders like: pernicious anemia, premature ovarian failure, vitiligo, celiac disease.

►DM -Type I=> do not have appreciable endogenous insulin production, ketoacidosis when deprived of insulin treatment. [High blood sugar, ketonuria, increased anion gap]

►Sulfonylurea induced hypoglycemia [is severe and prolonged], RX:
-Admission
-Bolus of D50% followed by dextrose infusion + Octeotride
[Because bolus of dextrose lead to elevation in the blood glucose level increasing secretion of insulin->severe rebound hypoglycemia later]
> Add OCTEOTRIDE [somatostatin analogue that inhibits insulin secretion]

►Causes of hypoglycemia with increased plasma insulin level:
1. Insulinoma {Elevated serum Insulin and ↑ C-Peptide
2. Sulfonylurea use {Elevated serum Insulin and ↑ C-Peptide
3. Exogenous insulin [Elevated serum Insulin and LOW ↓ C-Peptide

Diabetes Mellitus/complications-
►Diabetic Retinopathy needs some years to develop => poor glycemic control and blurry vision can be due to swelling in the optical lens secondary to osmotic changes. => Best next step is => Improve glycemic control
►In a patient with Diabetic Retinopathy => how avoid further retinal damage? => Add insulin to the regimen to tighten glucose control.-

►Autonomic neuropathy/delayed gastric emptying -> advice small frequent meals- Metoclopramide, Erythromycin -> Cisapride x long term and severe refractory gastropathy
►Diabetic gastroparesis
Gastroparesis (D.M autonomic neuropathy, delayed gastric emptying -> food remaining in the stomach for a longer period of time than normal.) -symptoms: Postprandial bloating, postural dizziness, abnormal sweating, constipation => Delayed gastric emptying cause postprandial hypoglycemia
♣DX: x rays, manometry, BEST TEST: Nuclear studies to study gastric emptying => Gastric emptying scan
♣RX: Dietary changes (Low-fiber and low-residue diets, restrictions on fat and/or solids), oral medications [pro-kinetic] such as Metoclopramide (Reglan, Maxolon, Clopra). Erythromycin and Domperidone (Motilium); adjustments in insulin dosage for those with diabetes,
♠ For long term management: Cisapride (Propulsid), jejunostomy tube, parenteral nutrition, implanted gastric neurostimulators ("stomach pacemakers"), or botulinum toxin.
♣Complications: Bacterial infection due to overgrowth in undigested food, also, the food can intestinal obstruction; malnutrition,harden into solid masses called bezoars WT loss, diff. controlling glucose.

►DM: Tight glucose control => decreases risk for development of ‘microvascular’ complications [retinopathy, nephropathy, neuropathy]. No effect/control on macrovascular risks: MI, PVD, Stroke, etc
Diabetic patient should maintain their LDL cholesterol < 100 mg/dL -> Statins are the DOC**
►Diabetes Mellitus
►Diabetic poly-neuropathy –affects lower limbs in a stocking-glove pattern, decreased ankle reflexes, ↓ sensitivity to pain or light touch, paresthesias progressing to motor weakness and pain.
BEST screening and diagnostic test x D. Neuropathy is => NERVE conduction studies [axonal pattern of nerve damage]
► Diabetic peripheral neuropathy -> pain management.-
TCAs nortriptyline or amitriptyline [but are contraindicated in pts with HEART disease]
Alternative: Gabapentin

DM management during delivery and labor/surgery
**Insulin administration should not be stopped [to prevent DKA] in a type 1 DM patient even if they are not eating.- Insulin requirement DECREASES following delivery of a pregnant diabetic.
“Take normal insulin dose on the night before surgery and start on insulin drip and infusion of D5% with KCL 40 mEq [to keep blood sugar < 160]

DKA: Most common complication/mistake= incorrect fluid management
-Understand that glucose infusion and potassium replacement are important components in DKA, aside from insulin and NSS

1- Initial fluid: NSS 0.9% until the blood sugar reaches 250 mg/dl + Insulin infusion

2- Then: Decrease Insulin infusion to 1-2 U/hr and change to D5% ½ NSS + KCL
[Dextrose infusion is very important to decrease ketone levels]
3- Switch to SQ Insulin [Rapid acting] [when pt is stable, normal anion gap, HCO3 is > 10, able to tolerate food and precipitating factors are under control] then to oral route feeding.

 DKA – another common mistake: stopping IV Insulin w/o overlapping dose of SQ Insulin
Result= rapid recurrence of DKA
-When change to SQ insulin= this route takes time to be absorbed, so, it should be administered 30-60 minutes before insulin infusion is stopped => fails to do this = rapid recurrence of DKA.

►Diabetes Mellitus / HONKC
**Diabetes type 2 patient presenting with a Hyperosmolar nonketotic coma => hyperglycemic crisis precipitated by infection and steroid use =>
-Requires high doses of IV insulin due to insulin resistance induced by infection, steroid and hyperglycemia.
-Then insulin is reduced once blood sugar and infection are controlled
-SQ insulin is started once the patient starts to EAT consistently:
Regular insulin [rapidly acting] is required to cover meals
And basal insulin (NPH or Glargine) insulin required to cover the period b/w meals
-AVOID oral hypoglycemic agents

►Metformin is especially useful in DM patient OBESE
Metformin, Pioglitazone and Rosiglitazone => Decreases Triglycerides, improves NASH and increases HDL
Insulin, thiazolidinediones, sulfonylureas [glyburide] -> cause WT gain
►Metformin can be continued in patients with PCOD UNTIL PREGNANCY IS DOCUMENTED. Then, advice her to STOP Metformin and prescribe INSULIN

► Metformin is contraindicated in:
 CHF
Alcoholism
Renal failure
USE Insulin for these cases
►Pt with uncontrolled DM and renal failure => Stop Metformin and other sulfonylureas [Stop Glyburide] and Start Insulin [Renal failure is an indication to treat DM with insulin]
• Glyburide is metabolized in the kidney, stop this drug; and Metformin.
• Since Rosiglitazone is metabolized by the liver =. No need to stop Rosiglitazone

► Metformin & lactic acidosis => in the elderly, pt with renal failure, hepatic or heart failure…
=>Other causes of lactic acidosis: hypoperfusion and tissue hypoxia [sepsis, hypovolemic shock, and hypoxemia
=> Look always the anion gap.- AG= Na – (CL + HCO4) Normal: 8-12

High Anion gap metabolic acidosis: Lactic acidosis [metformin], renal failure, DKA, intoxications, ASA, ethylene glycol or methanol.

►Patients with type 1 DM or any autoimmune disease are in risk or predisposed to get other autoimmune disorders such as: Addison disease, hypothyroidism, pernicious anemia, atrophic gastritis; etc [MEN or PAS].
 If a patient with DM type 1 has symptoms of Addison disease; or adrenal failure [WT loss, asthenia, eosinophilia, hyperkalemia, borderline sodium levels, mild anion gap acidosis, prerenal azotemia, low blood glucose].
THE next step is: Check Cosyntropin stimulated cortisol levels

►Effects of Exercise on glycemic control – [principally DM type 2]
-“Hypoglycemic effect of exercise” for hrs after completion of exercise=> Exercise decreases blood glucose by improving insulin sensitivity and increasing non-insulin mediated uptake of glucose by the muscles => serious problems in glycemic management [can lower blood glucose in the presence of high insulin or increase blood glucose when insulin is low]
-Pt needs to check blood glucose before exercise, advice to eat a snack if glucose is in low normal range
-Most reasonable option is: => Start patient on insulin pump [free to vary the timing of their exercise and food intake]
►The role of exercise control in DM type 1 [is not as clearly defined as for the DM type 2]
**Type 1 DM achieve glycemic control with a lower dose of insulin when they exercise regularly or participate in sports
Make a appropriate schedule in order to make appropriate insulin adjustments
Should decrease dose of insulin prior to their most active time of the day or when participating in sports
**Exercise in early afternoon=> morning dose of NPH should be reduced
**Patient on insulin pumps => has more flexibility for changing their insulin dose and may participate in unplanned activities.
**AVOID exercise when blood sugar is > 250 => because risk of DKA

►Glucagonoma
---Pancreatic tumor associated to mild Diabetes and a classic skin rash “Necrolytic migratory Erythema”
--High Glucagon level in serum--Can have metastasis, mainly in liver at the time of diagnosis
--Can secrete other peptides: VIP, calcitonin, GLP1
--RX: Surgery
►Pancreas-
Classic triad of Somatostinoma [tumor, Delta cells of pancreas; majority are cancerous]
Gallstones [by inhibitory effect of somatostatin on gallbladder motility]
Malabsorption [by inhibition of pancreatic secretion]
Diabetes Mellitus [inhibition of insulin secretion by somatostatin]
DX/ Next step: Fasting somatostatin levels [> 160]

Incidentalomas:

►Incidentaloma=unilateral Adrenal mass => require further workup for hormone secretion or malignancy => evaluate hormone production is the first step
►Incidentaloma [Adrenal mass] work up:
Serum electrolytes
Dexamethasone suppression testing
24 hr urine catecholamine, metaneohrine, VMA
17-ketosteroid measurement
All masses should be managed conservatively with SERIAL ABDOMINAL IMAGING
Remove if masses increase in size
All functional mass with radiographic evidence of malignancy or masses > 4 cm in size should be removed

► Pituitary Incidentaloma-
Patients with a small mass [5 mm] or lesion without any hormonal dysfunction can be safely followed by periodic assessment of the pituitary gland with MRI [Repeat MRI of pituitary in 6-12 months]

If first episode of proximal DVT in leg, symptomatic pt => Need Anticoagulation / the primary goal of RX is to prevent clot extension and prevent acute PE!
 RX: Unfractioned Heparin IV /or LMWH x 5 days and add WARFARIN 24 hr after heparin starting. Heparin continued until INR is in 2.0-3.0 for at least 2 days.
If pt is a woman taking HRT => urged to stop HRT and continue WARFARIN for at least 3 months
After a 1st thromboembolic event in the presence of a reversible risk factor = should be treated with anticoagulation for at least 3 months
After a 1st IDIOPATHIC thromboembolic event [no risk factor] or recurrent thromboembolic event => should be treated for a prolonged period of time.

PARATHYROID:
►Indications of parathyroidectomy [pts with 2ry or 3ry hyperparathyroidism]
-Calcium > 10.5 not responding to conservative management
-Moderate/severe hyperphosphatemia, not responding to medical management
-PTH of > 1000 pg/mL
-Intractable bone pain
-Intractable pruritus
-Episode of calciphylaxis -Soft tissue calcification
-It is very important to rule out low bone turnover or adynamic bone disease before a parathyroidectomy => if ALP is very high { indicates that bone turnover is increased, so the surgery is safe.

► Indications of parathyroidectomy in patients with 1ry hyperparathyroidism
-Bone mineral density T score < 2.5 SD
-Complications of 1ry hyperparathyroidism like= kidney stone, evidence of bone disease
-Serum calcium > 1 mg/dl above upper limit of normal
-History of life threatening hypercalcemia
-Urinary Calcium > 400 mg/day
-Creatinine clearance decreased 30%
-younger than 50 y/o

Bone loss after delivery => exaggerated bone loss if predisposing factor like Heparin use combined with increased bone loss during postpartum state. => Discontinuation of breastfeeding during post-partum -> can preserve or improve bone mineral density because decrease calcium loss and decrease in the level of parathyroid hormone rP [PTHrP] which is important in calcium transfer into breast milk, and levels are increased during nursing.

►Celiac disease or gluten induce enteropathy can present w/o signs of malabsorption; in a pt with iron and vitamin D deficiency despite consuming a nutritious diet –and can be suggested by family history of autoimmune diseases and vitiligo.-
Celiac disease screening= Anti-endomysial and Anti-tissue transglutaminase antibody level
Gold standard => Small Intestinal biopsy

Familial Hypocalciuric Hypercalcemia [FHH]
-Can be managed with moderation of calcium intake, adequate hydration and regular physical activity
-Has: => LOW Urine Calcium [ 10
Osteopenic =T score 1.0-2.5
Oteoporosis = T score < 2.5
Optimal intake of calcium and vitamin D

►Excessive bone resorption after prolonged immobilization => leads to hypercalcemia. Seen in patient with a very high turnover [adolescent, older pt with Paget’s] [days to weeks following immobilization
Management: Biphosphonates are useful in decreasing bone resorption

►MEN 1 with 3 “P” = hyperparathyroidism, pancreas tumor [gastrinoma…] and pituitary tumor => Next step in the management => require refer to a surgeon for parathyroidectomy

► Sarcoidosis => hypercalcemia => Overproduction of 1,25-dihydroxyvitamin D => Increased Calcium uptake
Sarcoid produce the enzyme 1-alpha-hydroxylase [convert 25 hydroxyvitamin D to 1,25 dihydroxyvitamin D leading to an increase in GI absorption of calcium. => Resulting hypercalcemia leads a suppressed PTH secretion and an increased urinary calcium excretion.
►Sarcoidosis / hypercalcemia => RX: Glucocorticoids + hydration

►Hypercalcemia
1-Primary hyperparathyroidism
2-Malignancy induced hypercalcemia [calcium is very ↑↑↑]
3-Vitamin D induced hypercalcemia
**In hyoercalcemia due to malignancy [exe; lung tumors, smokers]=> SERUM Calcium levels is much higher than in primary HP.

►Hypercalcemia classification
1-Parathyroid dependent {primary, terciary HP, lithium induced hypercalcemia, familial hypocalciuric hypercalcemia}
2-Parathyroid independent {Malignancies, vitamin D toxicity, granulomatous diseases [sarcoidosis], milk alkali syndrome
Malignancy: due to secretion of PTHrP and other causes
Next step => Serum PTH measurement


►Pseudo-Hypoparathyrodism
Long standing hypocalcemia, hyeprphosphatemia, bilateral catarats and calcification of basal ganglia
Cause: resistance of PTH on its target tissues
Type 1A: Albright hereditary osteodystrophy AHO [short stature, round facies, short 4th – 5th metacarpals and a short neck]
Type 1B: Does not have features of 1A AHO
►Pseudo-Pseudo hypoparathyroidism [PPHP]: do not have hypocalcemia and hyperphosphatemia because resistance to PTH is mild but has features of Albrights

►Following gastric bypass surgery => malabsorption => DX: Measure 25-hydroxyvitamin D serum
RX: need higher doses of vitamin D supplementation to prevent osteomalacia

►Paget’s disease: RX: Biphosphonates
Treatment is indicated when:
Bone pain
Involvement of weightbearing bones
Neurological compromise
Hypercalcemia
 Hypercalciuria
CHF

►When pt does not respond to treatment for osteoporosis [biphosphonates] despite being compliant and has rapid bone loss => suspect secondary causes [MM]*
=> NEXT STEP: Serum Urine protein electrophoresis**

►Long term management for chronic hypoparathyroidism: Vitamin D + Calcium
**Borderline serum calcium and high urinary calcium [due to low PTH levels, PTH increases renal calcium absorption]. => Add a Thiazide diuretic [increase serum calcium and decrease loss of calcium through urine]

ADRENALS

CORTISOL DEFICIENCY = hypotension
=>LONG TERM USE OF GLUCOCORTICOIDS => Patients with chronic suppression of hypothalamic-pituitary-adrenal axis => 2ry or 3ry Adrenal insufficiency => ↓ Cortisol, ↓ ACTH
►Since ACTH does not control secretion of mineralocorticoids from the zona glomerulosa of adrenal gland, patients have can developnormal plasma aldosterone and are normotensive, but IF STRESS vascular collapse under a stressful situation [infection] [ACUTE ADRENAL INSUFFICIENCY] due to decrease of cortisol [hormone responsible for keep vascular tone] and since cortisol promotes synthesis of catecholamines in adrenal medulla, cortisol deficient patients are unable to grow a good pressor response to stress.
♥Hypotension in these pts responds to administration of a stress dose of glucocorticoids and hydration.
♥In secondary and tertiary adrenal failure, there are no significant abnormal mineralocorticoid levels [no need fludrocortisone]

Goal of treatment in ACUTE ADRENAL INSUFFICIENCY

** Reverse hypotension -> Hydration IV NSS 0.9%
**Correct electrolyte abnormalities ->
**Replace cortisol => IV Dexamethasone followed by Cosyntropin Stimulation test

► Dexamethasone is preferred because does not interferes with the measurement of ACTH or Cosyntropin test or with urinary steroids.

Spontaneous hypokalemia, hypertension
-Screen for hyperaldosteronism => Plasma/aldosterone to plasma/renin activity

►Pheochromocytoma = biochemical confirmation is required before imaging is performed to localize tumor => Screening test: - Urinary metanephrine and catecholamine levels.
=>Next step: Alpha blocker {Phenoxybenzamine} and liberal salt/fluid intake to control HTN and restore intravascular volume [for 10-14 days before surgery]
=>Then: Imaging to localize tumor before surgery {CT or MRI
=> Intraoperative hypotension -> IV bolus of NS followed by continuous NS infusion.

Screen for Pheochromocytoma in pts with Medullary thyroid cancer => measure plasma free metanephrine {MEN 2A

USE OF GLUCOCORTICOIDS => HIGH DOSES, > 7.5 MG/D, FOR MORE OF 3 WEEKS => can cause suppression of the hypothalamic-pituitary-adrenal- axis -> Lead to tertiary ADRENAL insufficiency

►Cushing syndrome
**Cushing with ADRENAL etiology is biochemically characterized by a non-suppressible or non-response high dose dexamethasone suppression test and low plasma ACTH levels.=> Next step => Do a CT scan of the adrenal glands

►Young patient with diabetes and HTN, osteoporosis, hypokalemia and metabolic alkalosis => Screening for Cushing syndrome => Overnight Dexamethasone suppression test [or 24 hr urinary free cortisol]

PITUITARY:

Nelson’s syndrome: Bitemporal hemianopsia, hyperpigmentation following bilateral adrenalectomy x Cushing’s disease is suggestive of => Nelson’s syndrome. [pituitary enlargement due to loss of feedback by adrenal glucocorticoids; hyperpigmentation and visual defect after adrenalectomy]
Diagnosis: MRI of the brain = Mass lesion in the sellar and suprasellar region and high plasma ACTH
RX: Surgery and pituitary radiation

►Untreated Acromegaly => Increased RISK of C/V disease [leading cause of death in acromegaly]
50% acromegalic pt has HTN, LV dysfunction, others has asymmetrical septal atrophy, conduction defects, atherosclerosis, etc
Increased risk for COLON cancer

►Hypogonadism, high prolactin and low gonadotropin LH/FSH levels; increased ↑ serum alpha subunits => Classic non-functioning pituitary adenoma
[Arises from gonadotropin secreting cells or gonadotrophs in the pituitary]
►First line of Transphenoidal surgerytherapy












THYROID GLAND:

►Pt taking Amiodarone and with WT gain, swelling on feet, tiredness. =>
Next step => “check TSH in serum” [Amiodarone => Thyroid dysfunction, corneal deposits, skin discoloration, pulmonary fibrosis [lipoid Pneumonitis] and liver toxicity]

►Amiodarone effect on thyroid functions
--Pt on amioradone due to Atrial fibrillation => thyroid side effects
--Causes decrease in conversion of T4 to T3 => in turn decrease T3 levels and elevated T4

►Nodules, Thyroid:
Thyroid nodules > 1 cm should be subjected to FNA and biopsy
Thyroid nodules < 1 cm should be followed by thyroid U/S YEARLY
Radionuclide scan is useful in Toxic adenoma [thyrotoxic state]
► A toxic thyroid nodule is charactherized by signs and symptoms of hyperthyroidism and the RIAU is increased focally [focal uptake]

Thyroid
Papillary thryroid cancer = NTT Near total Thyroidectomy is the TOC. Than use RAI to destroy any residual thyroid -> then total body scan

►Symptomatic tachycardia in hyperthyroid patient => use B-blockers / -> Propranolol

►Subclinical hypothyroidism =mild elevation of TSH [5 – 10] with normal FT4
►Subclinical hypothyroidism + depression and ovulatory dysfunction [irregular heavy painless menstrual cycles] with *only mild elevated TSH
*Next step: => Low dose Levothyroxine [start 50 mcg of levothyroxine or L4]
*Resolves depression and menstrual irregularities
►ASYMPTOMATIC Subclinical hypothyroidism => does not requires treatment
►Subclinical hypothyroidism = Treatment is required if:
Antithyroid antibodies [Anti-TPO]
Abnormal lipid profile
 Symptoms of hypothyroidism
Ovulatory and menstrual dysfunction
When TSH > 10
§ RX: Levothyroxine

►Subclinical hyperthyroidism or EVIDENT hyperthyroidism
HAS increased risk for Chronic ATRIAL FIBRILLATION
Primary goal of treatment: Rate/Rhythm control with B-blockers or CCC
And to prevent systemic embolization with anticoagulation- Heparin
► HRT => increases the risk of DVT venous thromboembolism and for PE [can lead to increased risk of forming blood clots] – It’s not indicated for primary or secondary prevention of CAD. [indicated to prevent osteoporosis and vasomotor symptoms]

►Subclinical hyperthyroidism /or/ thyrotoxicosis: Next step…
Subclinical thyrotoxicosis:When Suppressed TSH levels with normal thyroid hormones [Normal FT4]
MCC: treatment with levothyroxine, nodular thyroid disease, Grave’s disease and thyroiditis
♥ Subclinical thyrotoxicosis induced by levothyroxine RX: reduce the dose
♥ Recheck TSH after 6-8 wks [etiology no determined, TSH becomes normal if repeated in a few weeks]
♥ No treatment is necessary: Patient with no symptoms, mildly decreased TSH, normal bone density

►Calcium carbonate and iron preparations, OTC vitamins => can impair absorption of Levothyroxine
=> Pt with unexplained increased TSH
Pt should be instructed to take L4 on an empty stomach, in the morning, separately from other medications

►Thyroglobulin => Their only source in the body is the thyroid gland, thus the presence of EXOGENOUS Thyroid hormones would suppress the thyroid gland, inhibiting TG release into the circulation
Exogenous Thyroid hormone => “Undetectable levels of Thyroglobulin”
Some OTC weight loss pills contain thyroid hormones and can lead to pill induced thyrotoxicosis but decreased TG


►Thyroid LYMPHOMA [Rare thyroid malignancy in pts with Hashimoto’s]
-Suspected if a patient with longstanding Hashimoto’s thyroiditis presents with a rapid enlarging and compressive symptoms [dysphagia] and a firm thyroid gland or goiter
DX: large fine needle aspiration/biopsy
RX: Radiation and chemotherapy, => the response is good

►Hashimoto’s thyroiditis diagnosis => High titers of anti TPO
Other studies => U/S, Radionuclide thyroid scan, aspiration biopsy

►Thyroid lymphoma => needs => large needle aspiration biopsy

►Primary hypothyroidism => have hyperlipidemia [isolated elevation of LDL or combined elevation in LDL and triglycerides]

PREGNANCY AND THYROID
►Thyroid hormones and contraception or HRT:
Estrogen replacement therapy/contraception => increases requirements for L-thyroxine in hypothyroid patients taking this medication.- [due to induction of liver enzymes, increased level of TBG and increased volume of distribution of the thyroid hormones]
Next step: “Increase dose of levothyroxine”

►Thyroid and pregnancy:
-T4 [Total T4] and total T3 are elevated due to increased levels of TBG => induced by estrogens

**The total serum thyroxine (T4) and triiodothyronine (T3) levels are increased in pregnancy due to high levels of estrogen which, in turn, increase the thyroid hormone-binding protein concentrations
-Free T4, Free T3 and TSH are normal in pregnancy
-In the post-partum period T4 and T3 return to normal [unless postpartum thyroiditis]


►Post partum thyroiditis [Thyrotoxicosis followed by hypothyroidism]
By autoimmunity, has triphasic course=>
1st phase: Thyrotoxic phase= Low Radioactive iodine uptake [1-3 months after delivery] -anxiety, insomnia, palpitations (fast heart rate), fatigue, weight loss, and irritability
2nd phase: Hypothyroid phase = 4-8 months after delivery- fatigue, weight gain, constipation, dry skin, depression and poor exercise tolerance.
3rd phase: Recovery phase [within 12-18 months of the onset]
**Permanent hypothyroidism develops only in 20% cases

►Pregnant pt with thyrotoxicosis
Needs to be treated with PTU => if not response or intolerance => Surgery is indicated
Treatment can’t wait until postpartum period [risk of thyroid storm is high]

►Pregnancy + Hyperthyrodism => Discontinue Methimazole [scalp defect or aplasia cutis in the fetus]
Prescribe: Propylthiouracil

►Pregnant woman with hypothyrodism:
***> Very important to be treated to avoid fetal mental retardation, low IQ
**During the first trimester: INCREASE dose of levothyroxine!

►Most appropriate course of action in GRAVE’S disease in USA:
1- Radioiodine ablation + concurrent prednisone administration
[to avoid worsening of ophthalmopathy]
Contraindications:
* Large retrosternal goiter [due to post-procedure swelling of the gland may compromise airways]

►Thyroid storm
-Diagnosis is med clinically -> Treatment can be started ASAP without waiting for Labs
-Glucocorticoids are used in thyroid storm because they inhibit the conversion of T4 to T3
-Iodine can not be administered before antithyroid drugs because excess iodine can serve as a substrate for the formation of more thyroid hormones, worsening the problem

**Other drugs that inhibit the conversion of T4 to T3=> B-blockers, amiodarone, PTU, iopanoic acid

►THYROID CANCER
**Post-operative patient [total thyroidectomy] of Medullary thyroid cancer=> ↑ persistently of calcitonin => Indicates Residual metastatic thyroid cancer => NEXT step => CT-scan of the neck and chest [with or w/o high resolution U/S]

►Thyroid cancer in remission => Suppressive doses of levothyroxine are needed
-Papillary thyroid cancer in remission => dose of levothyroxine is adjusted to suppress the TSH below normal range (0.1 and 0.3 mU/mL) => Then, the dose can be increased to bring TSH to WNL goal range
--Patient with distant metastasis => even lower levels [complete suppression] is required
--Remember: This “Suppressive doses of levothyroxine’ puts pt at risk of=> Bone loss and Atrial fibrillation

►Important concept:
**Hypothyroidism, even it is severe; it not a contraindication to emergency surgical procedures!
Example-cardiac catheterization


►MEN II:
 Medullary thyroid cancer
Pheochromocytoma
Hyperparathyroidism
MEN II b: Pheochromocytoma and Medullary carcinoma of thyroid + Marfanoid habits with mucocutaneous and GI neuromas

►Patient with medullary thyroid cancer and symptoms of pheochromocytoma like palpitations, headaches and sweating; surgery for thyroid cancer is scheduled => What is the NEXT STEP in Remember MEN II =management? > Alpha blocking agent for at least 10-14 days before ANY surgical procedure + liberal fluid/salt intake to restore intravascular balance

Remember: Suspecting pheochromocytoma and any surgery scheduled-> next step is=> Alpha blocking agent to avoid intraoperative complications with fluids/salt intake to restore intravascular volume

►Iodine induce thyrotoxicosis after angiography; next step-> Thyroid function tests
=>Secondary to iodine administration during coronary angiography => extreme fatigue, shakiness, WT loss, palpitations, firm irregular thyroid [possible multinodular] and tachycardia despite B-Blockers after coronary angiography.—
RX: Self limited disorder [can persist for months, is usually refractory to antithyroid medication]
--For mild symptoms: B-Blockers
--Moderate/severe symptoms: Antithyroid drugs => Refractory cases: Potassium perchlorate
*****Ineffective Radioactive iodine ablation because RAIU is low****

►Subacute thyroiditis: => Very low RAIU***Decreased uptake!!
Classical symptoms of thyrotoxicosis, heat intolerance, palpitations, SOB; thyroid gland enlarged, painful and tender.

►Lymphocytic thyroiditis: => with symptoms of thyrotoxicosis are treated with B-blockers [Propranolol]

►Hashimoto thyroiditis: => High titers of Anti-TPO => hypothyroid state => RX: replacement with levothyroxine

►Central hypothyroidism =>”EVALUATION OF THE ADRENAL STATUS [Cosyntropin stimulation test] is the best next step in patients with central hypothyroidism””
***A patient with Central hypothyroidism has => Borderline elevated TSH, low FT4 ***But this TSH is not biologically active
***-> NEED to rule out concomitant central adrenal insufficiency [Addison disease] that can increase TSH levels without hypothyroidism [due to loss of an inhibitory effect of glucocorticoids on TSH secretion].
***=> Next step: “Measurement of CORTISOL” with Cosyntropin stimulation test
(Treating pt with central hypothyroidism and concomitant adrenal insufficiency or Addison => could precipitate adrenal crisis)

 Combination of elevated T3, T4 and TSH can be seen in:
1- TSH secreting pituitary adenoma and in
2- Thyroid hormone resistant syndrome
The difference: Hyperthyroidism central/pituitary adenoma has ELEVATED Alpha subunits

►TSH secreting pituitary adenoma
**Clinical features of thyrotoxicosis with: ↑ T3 ↑ T4 and ↑ TSH + ↑ serum alpha subunits
►Similar: Thyroid hormone resistant syndrome
**Clinical features of thyrotoxicosis with: ↑ T3 ↑ T4 and ↑ TSH BUT: Normal serum alpha subunits

►Euthyroid sick syndrome [Low T3 syndrome]
-Commonly seen in hospitalized pts due to another condition
-LABS: Decreased total T3 [↓Total T3] with Normal T4 and TSH
-If patient is extremely sick=> can be reduced T4 and TSH too [bad prognosis]
-TSH transiently increases above normal limits when pt recovers
►RX or next step: Repeat thyroid function test in 8 weeks [because recovery phase], do not use thyroid hormone

►Elder people with mental status change => Order Thyroid function tests
MCC of mental status change in elderly:
#1- Medication, polypharmacy
#2- Infections UTI, pneumonia, etc
#3- Metabolic abnormalities=> hypothyroidism, thyrotoxicosis [with depression]

USMLE WORLD STEP-3 INTERNAL MEDICINE
Cardiology
CAD
Multiple risk factors increase the risk of acute CAD.
The risk can be significantly reduced by decreasing LDL cholesterol and, to a lesser degree, by adequately controlling the blood pressure (in diabetic patients, as well as in patients with previous CAD, a blood pressure lower than 130/80 mmHg is recommended). The contribution of other interventions such as smoking cessation, diabetes control, exercise, and HDL increase is also beneficial, but not as critical as the first two measures.
♥In patients with hypercholesterolemia and hypertriglyceridemia ► a statin is the best initial drug of choice. The primary target of therapy is normalizing LDL-cholesterol; the secondary target is normalizing non-HDL cholesterol(total cholesterol – HDL cholesterol).
♥If the statin fails to control hypertriglyceridemia► gemfibrozil or niacin can be added.

In prospective studies of healthy patients, those with higher plasma fibrinogen concentrations developed coronary heart disease more frequently. Fibrinogen levels decrease only 20 years after smoking cessation; while certain drugs such as atorvastatin and lovastatin can produce hyperfibrinogenemia. Even though it is uncertain how dangerous hyperfibrinogenemia can be, it is currently recommended to maintain fibrinogen levels as low as possible and to discontinue any medication that can increase them, unless the benefits outweigh the risks.
Exercise electrocardiography should be the initial test of choice for evaluation of suspected CAD in patients with a normal resting EKG and the ability to exercise.
Exercise stress testing is useful for the diagnosis and risk stratification of patients with stable angina. Exercise testing is useful to identify a high-risk group that will benefit from coronary angiography.

♥Patients at high risk have one of the following findings on exercise testing:
1. failure to increase their blood pressure with exercise
2. Inability to complete stage I of Bruce protocols
3. Appearance of horizontal or down sloping ST segment during exercise

♥Pharmacological stress testing is an alternative to exercise stress testing, and is indicated in the setting of CAD, when the patient is unable to exercise due to some underlying medical disorder (amputation), when an exercise stress test is relatively contraindicated (MI, unstable angina), or in patients in whom interpretation of an EKG is difficult because of preexisting changes, such as left bundle branch block, baseline ST/T changes from left ventricular hypertrophy, etc.
♠Both adenosine and dipyridamole scans are contraindicated in patients with COPD because they can induce bronchospasm.
♥The American College of Cardiology/American Heart Association (ACC/AHA) defines a positive exercise stress test as ►1 mm or more of down sloping or flat ST segment depression during exercise or during the recovery period.
♥Exercise echocardiography is more sensitive than the resting echocardiogram or stress EKG, and is also commonly done in patients with stable angina.
♥Resting echocardiography or EKG is useless in stable angina when the patient has no active chest pain.
-Principles and practice of stress testing are based on the relative development of manifestations of myocardial ischemia as the stress to the myocardium is increased. Regional wall motion defects are the first to appear (detected by stress echocardiography). This is followed by significant perfusion defects (detected by perfusion scans), pulmonary capillary wedge elevation, ST segment changes on electrocardiogram, and finally, anginal pain.
-Stress imaging should therefore be pursued if there are no EKG changes during or after the stress test. It is also the modality of choice in patients with bundle branch block, WPW syndrome, paced ventricular rhythm, and baseline ST changes before exercise (ventricular hypertrophy, digitalis therapy, etc.). A positive stress test result in a patient with symptoms warrants further intervention, preferably with a coronary angiogram.
-Nuclear scans should be performed in subsets of patients with baseline ST segment changes.
-It is important to recognize the advantages and limitations of different available testing modalities for the evaluation of myocardial ischemia.
--Adenosine, along with dipyridamole and dobutamine, is one of the most frequently used pharmacologic agents to induce ischemia in patients with poor functional status.
--Sestamibi test is one of the three available technetium-99m labeled tracers used frequently for radionuclide imaging.
-In patients with angina and a prior history of revascularization (PTCA or CABG), technetium-99m (sestamibi) and thallium perfusion imaging are the two types of radionuclide perfusion imaging modalities used for characterizing the ischemia, establishing the functional effects of lesions, and determining the viability of myocardium.
-The two modalities are also used in patients with complete left bundle branch block (LBBB), electronically-paced ventricular rhythm, preexcitation (WPW) syndromes, > 1 mm ST segment depression at rest, LVH with repolarization changes, and in patients who are unable to exercise.
♥Coronary angiography is indicated in the setting of stable angina pectoris when the angina is refractory to medical treatment, or when exercise testing identifies the patient as high-risk.

►Studies have shown that compared to angioplasty, Coronary artery bypass graft (CABG) improves long-term survival in diabetic patients with multivessel disease and recent Q-wave infarction.
-Good, perioperative control of the blood glucose levels is advocated during CABG. An insulin drip achieves better blood glucose control during CABG, and decreases the incidence of sternotomy wound infections.
-According to the ACC/AHA guidelines, patients who are unable or unfit to exercise should undergo adenosine or dipyridamole myocardial perfusion imaging for the diagnosis and risk stratification of CAD. Adenosine, along with dipyridamole and dobutamine, is one of the most frequently used pharmacologic agents to induce ischemia in patients with poor functional status.

►All patients with chronic stable angina should undergo an exercise stress test for prognostic and risk stratification.

►All patients with coronary artery disease and stable angina should be referred for an exercise stress test for further risk stratification.
♥Patients with a low risk treadmill score (able to exercise more with a normal EKG) have less than 1% annual mortality rate.
♥On the other hand, patients who have a high-risk treadmill score (presence of chest pain or EKG changes with minimal exercise) have greater than 3% annual mortality rate and require more aggressive management.

►Metformin use is contraindicated in patients with renal failure, sepsis, hepatic dysfunction, and severe heart failure. It should also be stopped in patients who are at risk to develop renal failure, such as those who will undergo angiography, a procedure that involves infusion of a high load of contrast agent

►In patients whose chest pain may be ischemic in origin, supplemental oxygen and ASA aspirin should be administered immediately, followed by Nitroglycerin, Morphine, and Beta-blockers (presuming no contraindications exist).

►Calcium channel blockers can be harmful in patients with acute MI.

►Beta blockers, ACE inhibitors, and statins have significant effects on the secondary prevention of coronary artery disease, and should be continued indefinitely unless absolutely contraindicated.

►The only indications of calcium channel blockers in the setting of acute myocardial infarction include:
♥Intolerance to beta blockers, post-infarction angina refractory to beta blockers and nitrates, and rapid atrial fibrillation with contraindications to beta-blocker use (COPD, pulm. disease)
♠Peripheral edema is a well-known side effect of calcium antagonist therapy. It is more common with dihydropyridine agents (e.g. nifedipine, amlodipine), but also occurs in 2 to 15% of patients taking diltiazem [cardizem]. The exact mechanism of calcium antagonist-associated edema is not known; arteriolar dilatation seems to be responsible for increased interstitial fluid accumulation in these patients



Acute pericarditis
Acute pericarditis (infarct associated or infarction pericarditis) can occur within one to four days as a direct complication of a transmural myocardial infarction. The recurrence of chest pain three days after a myocardial infarction, which gets worse with position changes and deep inspiration, is suggestive of infarction pericarditis. It is usually a clinical diagnosis, and is supported by the presence of pericardial friction rub and EKG changes of pericarditis (sinus tachycardia, diffuse ST segment elevations with PR segment depression).
-A pericardial rub is usually heard over the left sternal border and can be present during any of the phases of the cardiac cycle. It is heard as a superficial scratchy or grating sound, which gets more pronounced when the patient leans forward.
-The EKG changes of pericarditis are not always seen, but usually resemble that of an acute myocardial infarction.

Infarction pericarditis
-Is seen less frequently in patients with early and complete reperfusion. It is usually a transient episode and does not affect the management of acute myocardial infarction, unless it is complicated by a large pericardial effusion or tamponade.
>>It should be managed with close clinical observation and adequate pain control.
>>Nonsteroidal anti-inflammatory medications (NSAIDs) are effective in alleviating the pain associated with acute pericardial inflammation, although there are some concerns that they may increase the risk of myocardial rupture after a transmural MI.

D/DX:
1. Chordae tendineae or papillary muscle rupture is a life threatening mechanical complication of an acute myocardial infarction. It usually occurs two to seven days after the infarction and causes acute hemodynamic instability. Patients also develop acute pulmonary edema.

2. Dressler’s syndrome, or post-cardiac injury syndrome, occurs in patients with myocardial infarction and after cardiac surgery. It is an autoimmune mediated syndrome, which usually develops weeks to months after an acute MI. It usually presents with fever, leukocytosis, pleuritic chest pain, and a pericardial rub.

3. A left ventricular aneurysm usually occurs as a late complication of a transmural myocardial infarction (usually a large anterior wall MI). It may result in heart failure, ventricular arrhythmias, or peripheral arterial embolization due to the formation of a left ventricular thrombus. It develops over a longer period of time, and is usually not associated with chest pain. Persistent ST elevation can be present in these patients.

►Sinus bradycardia is commonly seen as a complication of an inferior wall myocardial infarction.
-It is usually present transiently, immediately after or within the first six hours, and resolves within 24 to 48 hours of acute myocardial infarction. Most of the episodes are asymptomatic, and resolve without any therapy or intervention.
>>BUT: Active intervention is required in patients with sinus bradycardia after myocardial infarction, if the patient exhibits signs and symptoms of hemodynamic compromise.
>>The definite treatment is by reperfusion of the infarcted myocardium, either with thrombolysis or percutaneous transluminal coronary angioplasty (PTCA).
>>If bradycardia persists after reperfusion, the patients can still respond to the administration of intravenous Atropine.
>>VERY HD UNSTABLE: Temporary transvenous cardiac pacing is usually required in patients with persistent bradycardia and hemodynamic instability after myocardial infarction, despite the above measures.

►Recent myocardial infarction and atrial fibrillation are two most common causes of arterial thromboembolism.
►The single most important step in the early management of the patients with acute arterial occlusion diagnosed by history and physical examination is immediate IV heparin therapy followed by continuous heparin infusion.
►Heparin therapy will prevent further propagation of thrombus, and inhibit thrombosis distally in the arterial and venous systems due to low flow and stasis.
♥An important issue for many patients after a myocardial infarction is when sexual activity can be safely resumed
-Therefore, this should be addressed in all sexually active patients before hospital discharge.
-A consensus panel concluded that patients resuming sexual activity 6 weeks after an uncomplicated myocardial infarction are at low risk for future myocardial events.
On the other hand, those resuming sexual activity within six weeks are at intermediate risk, and those resuming activity within two weeks are at high risk.
•Patients with complications such as post-infarction chest pain, evidence of arrhythmias, or heart failure are also at intermediate or high risk
•Patients at intermediate risk should receive further evaluation and subsequent reclassification into the low-risk or high-risk category, while patients at high risk should be stabilized with appropriate therapy before resuming sexual activity.
•Consultation with a cardiologist prior to resuming sexual activity is recommended in all high-risk patients

Radionuclide ventriculography [RVG]
-The anthracyclines (doxorubicin, daunorubicin, idarubicin, epirubicin, and mitoxantrone) are the most common cause of cardiovascular complications in cancer patients.
-Patients who are about to receive such cardiotoxic chemotherapy are therefore recommended to undergo baseline testing and serial reevaluation of systolic function to identify early signs of cardiotoxicity.
-Noninvasive serial monitoring of cardiac function with radionuclide ventriculography (RVG) has been the most effective way to detect early toxicity and reduce the risk of anthracycline toxicity.
-Radionuclide ventriculography [also known as radionuclide angiography (RNA) or multiple-gated cardiac blood pool imaging (MUGA scan)] is performed by labeling a patient’s red cells with a radioactive tracer and measuring radioactivity over the anterior chest with a gamma camera.
-It is the ideal and most reproducible test for providing longitudinal quantitative assessment of LV systolic function in patients receiving doxorubicin or other anthracyclines. It should be used when detection of slight changes in the ejection fraction are of importance, such as during chemotherapy, before and after cardiac transplantation, and severe CHF

HTN
-According to the new JNC VII guidelines, thiazide diuretics are the initial drugs of choice for the treatment of essential hypertension.
-Approximately 90% of patients with secondary HTN will have an unidentified cause. Of the remaining, around 3% will be due to renovascular HTN. Other less common causes account for less than 1% each, and are comprised of the following: primary hyperaldosteronism, drug-induced HTN, acute stress, and renal parenchymal disease.
-Abdominal or flank bruits are present in 50% of the patients with renovascular HTN. This makes this clinical finding the most common and expected clinical finding in a patient with suspected secondary HTN.
-Almost all guidelines recommend non-pharmacologic measures that can be preventive, adjunctive or sometimes definitive in the treatment of hypertension.
-Weight loss enhances the effect of anti-hypertensive drugs. It is the single most effective non-pharmacologic measure to decrease blood pressure in overweight individuals, and it is believed to reduce the overall cardiovascular risk in such patients.
♥ Note: Smoking cessation is an important component of lifestyle modification that reduces the risk of future cardiovascular events.
CHF
►Ischemic heart disease is the most common cause of congestive heart failure (CHF), especially dilated cardiomyopathy, in the United States. Approximately 50 to 75% of the patients with heart failure (HF) have coronary disease as the etiology.
-Other known etiologies are: hypertension (13%), valvular disease (10-to12%), renovascular disease, and very rare causes, such as obstructive sleep apnea, myocarditis, alcohol or cocaine abuse, etc.
-In a new case of CHF with unknown etiology, efforts must first be made to rule out the presence of coronary lesions which may be corrected by an angioplasty.
-Congestive heart failure is a clinical syndrome that results from impaired or inadequate ventricular emptying (systolic dysfunction) or impaired ventricular relaxation (diastolic dysfunction).
-It is characterized by specific symptoms that are either due to reduced cardiac output (fatigue, weakness) or excessive fluid retention (peripheral or pedal edema, dyspnea).
-All symptoms are exacerbated by exertion, and the degree of these symptoms helps in assessing the severity of heart failure.
♥Heart failure is primarily a clinical diagnosis and is based on a detailed history and physical examination. A detailed history and physical examination can also provide clues to the specific cause of heart failure.

♥CHF: The major criteria include the presence of paroxysmal nocturnal dyspnea, orthopnea, raised jugular venous pressure, pulmonary rales, presence of third heart sound, increased cardiac silhouette, and pulmonary vascular congestion on chest x-ray.
♥CHF: The minor criteria include the presence of bilateral lower extremity edema, nocturnal cough, dyspnea on exertion, tachycardia, presence of pleural effusion, and hepatomegaly.

►The diagnosis of heart failure is based on the presence of two major criteria or one major and two minor criteria, provided the minor criteria cannot be explained by the presence of any concurrent medical illness.
♥Chest radiograph findings suggestive of congestive heart failure include cardiomegaly, pulmonary vascular congestion (especially in the upper lobes), Kerley B lines and bilateral pleural effusions.
-The presence or absence of such findings will help to differentiate heart failure from dyspnea secondary to a primary pulmonary pathology.
-Plasma B type natriuretic peptide levels are useful at times to differentiate between cardiogenic versus non-cardiogenic causes of dyspnea.

♥Rupture of chordae tendineae should be suspected in healthy individuals who develop flash pulmonary edema (heart failure) associated with an acute mitral regurgitation.
-The differential diagnosis of this condition includes infective endocarditis, papillary muscle rupture secondary to ischemia, and mitral valve rupture secondary to trauma.

♥ACE inhibitors are the main therapy for CHF. ]They are indicated even in the asymptomatic phase. Although there is currently insufficient data to show its benefits to the mortality rate of African-Americans and women, their use in CHF is always recommended.

♥The only reasons for not giving ACE inhibitors are=
(1) Poor tolerance to the drugs (presence of severe adverse effects) and
(2) The presence of contraindications, such as renal failure
(Creatinine greater than or equal to 3 mg/dL) or hyperkalemia)
♥ Hydralazine and isosorbide combination is frequently used in the management of congestive heart failure in patients intolerant of ACE inhibitors.
-Patients should be informed regarding the possible adverse reactions that may result from the use of these drugs, and instructed to stop the medication if they develop any of these reactions.

♣The development of drug-induced lupus-like syndrome
-has been associated with the use of hydralazine.
-Other drugs that have been associated with lupus include procainamide, penicillamine, isoniazid, minocycline, diltiazem, methyldopa, chlorpromazine, and interferon-alfa.
-Patients with drug-induced lupus can develop a variety of flu-like symptoms such as fever, malaise, arthralgias, myalgias, and maculopapular facial rash. They can also develop lymphadenopathy, splenomegaly, pleurisy, and pericarditis. An important immunological marker of drug-induced lupus is the presence of antihistone antibodies.
Remember:
All patients with flash pulmonary edema of unknown etiology should be evaluated with an Echocardiogram.
Patients with acute pulmonary edema are initially managed with oxygen, morphine and loop diuretics (IV furosemide).
The two most commonly used antihypertensive agents in the management of hypertensive crisis complicated by acute pulmonary edema are => IV nitroglycerine and IV nitroprusside.
Marfans
Marfan or Ehlers-Danlos syndrome must be suspected in patients with connective tissue abnormalities and an acute mitral regurgitation secondary to chordae tendineae rupture, although a primary, pre-existing mitral valve prolapse (MVP) is the most common cause.
Some cases may be idiopathic (individuals who experience rupture of the chordae tendineae without previous MVP or connective tissue disease), but an etiology can be found in most occasions.

Cardiac risk factors
-Diabetes mellitus is the single most important predictor of adverse cardiovascular outcomes, especially in women. It is considered to be an equivalent of coronary heart disease (CHD).
-The role of glycemic control in the development of macrovascular disease in patients with type-2 diabetes is not firmly established.
=Hypertension is a well-established risk factor.
=The blood pressure should be kept below 130/85 mmHg in diabetics, unlike non-diabetic individuals whose target blood pressure is below 140/90 mmHg.
=Smoking is an important modifiable risk factor. The risk of cardiovascular events declines rapidly after smoking cessation, and approaches that of non-smokers in several years

MAT
Multifocal or multiform atrial tachycardia (MAT) is characterized by the presence of:
3 or more P waves of different morphologies
The QRS complexes are narrow
While the PR segments and the R-R intervals are variable
The heart rate can reach up to 200 beats per minute.

MAT is usually secondary to the following conditions:
1) hypoxia
2) chronic obstructive pulmonary disease (COPD)
3) hypokalemia
4) hypomagnesemia
5) coronary/ hypertensive/ valvular disease
6) medications (i.e., theophylline, aminophylline, isoproterenol)
Treatment M A T:
#1= Correct cause: Do ABGs because Hypoxia and COPD are the most common etiologies that affect mainly elderly patients. -Therapy is aimed at correcting the underlying cause.
#2-If the patient fails to improve >>electrolyte abnormalities (e.g., hypokalemia or hypomagnesemia) should be wanted.
If therapy is not effective and there are no contraindications, beta-blockers can be used successfully. In patients with asthma or COPD, verapamil is the drug of choice.

-SVT and sinus tachycardia are the most frequent types of tachyarrhythmias.
--Narrow and regular QRS complexes, and absent P waves >> suggest supraventricular tachycardia.
--Narrow and regular QRS complexes, and presence of P waves >> Sinus tachycardia

WPW Syndrome can be identified in around 0.2% of the population.
=It is characterized by a short PR interval (less than 0.12sec), a delta wave at the beginning of the QRS complex, QRS duration of 0.12sec or wider, and non-specific ST segments or T wave abnormalities.
=Although WPW Syndrome is not common, its diagnosis is important because it can present as SVT.
=If verapamil or beta-blockers are given, AV nodal conduction will be slow, and the accessory pathway conduction will increase.
=These can lead to ventricular fibrillation (especially if the patients already have atrial fibrillation) and death.



♦ Blocks:
-In comparison to Mobitz I, second-degree AV Block (Wenckebach’s), the PR does not prolong progressively, {PR is fixed] but QRS complexes are suddenly lost periodically
-Mobitz II AV Block can cause dizziness, episodes of syncope, or transient altered mental status, thus explaining the symptoms of the patient. Because Mobitz II can progress to third-degree AV Block, it needs to be managed with a permanent pacemaker. Even if the patient is asymptomatic, the AHA/ACC (American Heart Association/American College of Cardiology) consensus advises the use of a permanent pacemaker inserted through a venous access.

Nonsustained ventricular tachycardia
-Is defined as the presence of three or more consecutive ventricular beats with a heart rate greater than 120 beats/min, and with the episode lasting for less than 30 seconds. It is seen most commonly in patients with some form of structural heart disease, possibly because structural abnormalities lead to ventricular arrhythmia.
-Some structural abnormalities that can lead to nonsustained ventricular tachycardia include scarred myocardium from prior myocardial infarctions (coronary artery disease), ventricular hypertrophy (LVH or hypertrophic obstructive cardiomyopathy), dilated left ventricle (dilated cardiomyopathy), and valvular abnormalities such as mitral valve prolapse.
Although nonsustained ventricular tachycardia can also be seen in patients without any evidence of structural heart disease, it is important to recognize that structural heart disease is the most common cause, and a 2D-echocardiogram and stress test should be obtained to rule out ischemia.
-Patients with multiple risk factors for developing cardiac arrhythmias need proper evaluation for possible implantable cardiac defibrillator placement to prevent sudden cardiac death.

Atrial fibrillation
◘ Treatment of atrial fibrillation in a hemodynamically stable patient is different from one who is unstable.
1-If the patient is hemodynamically unstable (low BP, patient not responding to commands), the treatment of choice is electrical cardioversion.
2-If the patient is hemodynamically stable, the atrial fibrillation has to be categorized first as either an acute or a chronic process.
►If it is an acute process, the patient may undergo either Cardioversion [electrical or pharmacological with anti arrhythmic drugs class III agents (amiodarone, sotalol, ibutilide, etc.).
] or Rate control [B-blocker or CCB] to convert his arrhythmia to sinus rhythm. [If > 48 hrs, cardioversion need 1 month of anticoagulation in high risk pt]
►If it is a chronic process, then the patient should be managed with Rate control and Anticoagulation –heparin/Warfarin
Cardioversion -can be either electrical or chemical.

►Electrical cardioversion is indicated in hemodynamically unstable patients.

►Synchronized cardioversion (electrical shock administered in synchrony with the ‘R’ wave) is the procedure of choice.
-Electrical cardioversion should not be performed without 3-4 weeks of anticoagulation first in chronic atrial fibrillation (> 48 hours), as the risk of embolization is high. -The required joules are about 100-200 joules.
-Atrial fibrillation is generally more resistant to convert to normal sinus rhythm than atrial flutter, and the energy required is much higher.
-If you choose to do chemical cardioversion, the drugs of choice are class III agents (amiodarone, sotalol, ibutilide, etc.).

Lone atrial fibrillation in a patient younger than 60 years of age and without structural or functional cardiac abnormalities [CHF, valvular disease…] does not increase the risk of stroke. However, small studies suggest that there can be some risk, especially if the patient has other risk factors, such as advanced age, diabetes, or hypertension.
For this reason, ASA Aspirin is recommended as prophylactic therapy. The risk of anticoagulation with Coumadin outweighs the benefits in this setting.

♥Amiodarone interacts with Warfarin, prolonging its effect and increasing the risk of bleeding. Its clinical response is predictable; it is known that the Warfarin dose needs to be decreased about 25% in order to avoid over coagulation.

♥ Strokes and Warfarin use:
-The use of FFP or intravenous vitamin K is only necessary if the patient is bleeding or has an INR greater than 20.
-If the INR is supratherapeutic, but not higher than 5, it is enough to hold the Coumadin or decrease its dose.
-All other cases of excessive anticoagulation with Coumadin can be successfully managed with oral vitamin K

TCA toxicity
-TCA overdose is the leading cause of hospitalization and death. The signs of TCA overdose include hypotension, anticholinergic effects, CNS manifestations, and cardiac arrhythmias.
-Cardiotoxic effects are responsible for most of the mortality in patients with TCA overdose.
-TCAs inhibit fast sodium channels, which result to slowing of the phase 0 depolarization in His-Purkinje tissue and the myocardium.
-This may lead to QRS prolongation and reentrant arrhythmias, like ventricular tachycardia, ventricular fibrillation, and torsades de pointes.
►Sodium bicarbonate is the most effective agent for the management of TCA-induced cardiotoxic effects.
►Lidocaine is the antiarrhythmic drug of choice for TCA-induced ventricular dysrhythmias.

Valvular disorders
-Mitral stenosis is a narrowing of the outflow tract from the left atrium to the left ventricle, resulting in increased pressure in the left atrium, pulmonary vasculature, and right side of the heart.
-It is more common in women, and is most commonly linked to a remote episode of rheumatic fever, though congenital mitral stenosis may be found as well.
-On physical examination, mitral stenosis produces a diastolic thrill palpable over the apex and a low-pitched, rumbling diastolic murmur best heard over the apex when the patient is lying in a left lateral decubitus position.

Mitral regurgitation causes an apical holosystolic murmur, and is most often secondary to myocardial infarction, mitral valve prolapse, rheumatic heart disease, or coronary artery disease.

Aortic stenosis causes a crescendo-decrescendo systolic murmur with a normal S1, a diminished A2, and a paradoxical splitting of S2. It is most often secondary to senile degenerative calcification, or congenital malformation.

An aortic valve area of less than 1.0 cm2 is considered severe stenosis. The onset of symptoms markedly affects the prognosis in patients with aortic stenosis; therefore, prompt intervention is recommended in patients who have cardinal symptoms of aortic stenosis (dyspnea of heart failure, anginal pain, and syncope). Aortic valve replacement is associated with marked reduction in symptoms and mortality in patients with symptomatic aortic stenosis, and is considered the treatment of choice.

Critical aortic stenosis and angina at rest >> Highest risk factors for peri-operative cardiac risk

Aortic insufficiency causes a diastolic murmur best heard adjacent to the sternum in the second to fourth intercostal space. It is most often secondary to infective endocarditis, congenital malformation, connective tissue disorders, or rheumatic heart disease.

Pulmonic stenosis causes a systolic crescendo-decrescendo ejection murmur in the left upper sternal border. It is most often secondary to congenital malformation, rheumatic heart disease, or carcinoid.

COA: This is usually associated with a congenital bicuspid aortic wall.
-In a previously undiagnosed patient, the classic presenting sign is hypertension. Most patients are asymptomatic unless severe hypertension is present, which can lead to headaches, epistaxis, heart failure, or dissecting aneurysm. The major clinical manifestation in children and adults with COA is a difference in the systolic blood pressure between the upper and lower extremities. In most cases, the diastolic blood pressures are similar.
-The classic findings are hypertension in the upper extremities, diminished or delayed femoral pulses, and lower unobtainable arterial blood pressures in the lower extremities.
♦ Magnetic resonance imaging (MRI) is the best noninvasive modality to diagnose CO
Reply
#6
good to know for exam
Reply
#7
Thanks
Reply
#8
Thanks for sharing!

I am just wondering the source of this information.

Once again thank you for posting it on the forum.


-k
Reply
#9
Thanks, only thing I would add is HOCM is assoc with chr 14, otherwise these are very good, Thanks!
Reply
#10
USMLE STEP 3 MATERIALS 2015-16 AVAILABLE
Dear Friend
I have got a lots of important materials necessary for USMLE step 3. All of my materials are completed, updated and high quality. If you are interested then you can check sample. I am offering them at relatively cheap price.

For details, please contact: For G mail chat and Skype chat please add me then i will accept you.
Mail me: usmlesell
G mail Chat: usmlesell
or you can chat with me through gmail ( usmlesell) or skype chate ( usmle.sell)
website address: please google usmlemsell

PAYMENT OPTION: PAYPAL, SKRILL, BITCOIN, WEBMONEY.

Materials are available and you can ask for anything at anytime.
please check for new materials, it is always updating with new materials.

COURSE NAME:
USMLE Step 3 All Materials 2015
Kaplan USMLE step 3 centerprep videos with basic correlation with new curriculum 2015-16
Kaplan USMLE Step 3 Classroom Anywhere Lecture Videos with Lecture notes new 2015-16 with new lecture notes
Med Quest USMLE Step 3 High Yield Lecture Videos with basic correlation by Dr Conrad Fischer 2015-16
Med Quest Internal Medicine Master the Boards by Dr Conrad Fischer 2015
Kaplan USMLE Internal Medicine Master the Boards By Dr Conrad Fischer 2014
Doctors in training usmle step 3 videos and ccs 2015
Archer Online videos 2014
Archer CCS course 2014
Med Study Internal Medicine BOARD REVIEW VIDEOS 2015
USMLE Step 3 Premier Videos 2014
USMLE Step 3 Premier Cases 2014
USMLE WORLD Step 3 Question Banks 2015
usmle world step 3 ccs cases 2015
Kaplan Step 3 Question Banks 2014
USMLE WORLD internal medicine qbank 2014-2015
Reply
« Next Oldest | Next Newest »


Forum Jump: